Vous êtes sur la page 1sur 126

7.

0 Патологічна анатомія
1
Multiple oval ulcers along the intestine were revealed on autopsy of the person, who died
from diffuse of peritonitis in the distant part of the small intestine. Bottom parts of the
ulcers are clear, smooth, formed with muscular or serous covering, edges of ulcers are
flat, rounded. There are perforations up to 0,5 cm in diameter in two ulcers. What diseasis
can be diagnosed?
A Typhoid fever
B Dysentery
C Cholera
D Tuberculosis
E Typhus

2
Patient suffering from trombophlebitis of the deep veins suddenly died. Autopsy has shown
freely lying red friable masses with dim crimped surface in the trunk and bifurcation of the
pulmonary artery. What pathologic process was revealed by morbid anatomist?
A Tromboembolism
B Thrombosis
C Tissue embolism
D Embolism with foreign body
E Fat embolism

3
Examination of a patient revealed a dense, movable skin tumour that is standing out
distinctly from the surrounding tissues. Its section is found to be white and composed of
fibrous tissue. Microscopic examination revealed interlacing collagen fibers and few cells.
What tumour is it?
A Fibroma
B Myoma
C Histiocytoma
D Dermatofibroma
E Desmoid

4
A 50-year-old man has felt vague abdominal discomfort within past 4 months. Physical
examination revealed no lymphadenopathy, and no abdominal masses or organomegaly at
palpation. Bowel sounds are heard. An abdominal CT scan shows a 20 cm retroperitoneal
soft tissue mass obscuring the left psoas muscle. A stool specimen tested for occult blood
is negative. Which of the following neoplasms is this man most likely to have?
A Lipoma
B Melanoma
C Hamartoma
D Adenocarcinoma
E Lymphoma

5
A 40-year-old woman has had a feeling of abdominal discomfort for the past 8 months. On
pelvic examination, there is the right adnexal mass. Abdominal CT scan demonstrates a 7
cm cystic mass involving the right ovary with small areas of calcification. The uterus is
normal in size. The right fallopian tube and ovary have been removed surgically. Grossly,
the mass on sectioning is filled with abundant hair and sebum. Microscopically, the mass
has glandular spaces lined by columnar epithelium, squamous epithelium with hair follicles,
cartilage, and dense connective tissue. What type of tumour is it?

1
A Teratoma
B Squamous cell carcinoma of ovary
C Melanoma
D Sarcoma of ovary
E Metastase of cervical carcinoma

6
A man died 8 days after the beginning of the disease. He was diagnosed with dysentery. At
the autopsy it was found out a thickened wall of the sigma and rectum, fibrinous membrane
on the surface of mucous membrane. Histologically: there is a deep necrosis of mucous
membrane with infiltration of necrotic masses with fibrin. What kind of colitis does
correspond to the changes?
A Diphtheritic
B Catarrhal
C Ulcerative
D Chronic
E Gangrenous

7
A woman suffering from dysfunctional metrorrhagia was made a diagnostic abortion.
Histologically in the scrape there were a lot of small stamped glandulars covered with
multirowed epithelium. The lumens of some glandulars were cystically extended. Choose
the variant of general pathologic process in the endometrium.
A Glandular-cystic hyperplasia of endometrium
B Atrophy of endometrium
C Metaplasia of endometrium
D Neoplasm of endometrium
E Hypertrophic growth

8
A 46 year-old man complains of difficult nose breathing. Mikulich cells, storage of
epithelioid cells, plasmocytes, lymphocytes, hyaline balls are discovered in the biopsy
material of the nose thickening. What is the most likely diagnosis?
A Scleroma
B Virus rhinitis
C Allergic rhinitis
D Rhinovirus infection
E Meningococcal nasopharyngitis

9
Extensive thromboembolic infarction of the left cerebral hemispheres, large septic spleen,
immunocomplex glomerulonephritis, ulcers on the edges of the aortic valves, covered with
polypous thrombus with colonies of staphylococcus were revealed on autopsy of the
young man who died in coma. What disease caused cerebral thromboemboly?
A Septic bacterial endocarditis
B Septicemia
C Acute rheumatic valvulitis
D Septicopyemia
E Rheumatic thromboendocarditis

10
A patient ill with diabetes mellitus felt acute pain in his right foot. Objectively: foot thumb is

2
black, foot tissues are edematous, there are foci of epidermis desquamation, stinking
discharges. What clinicopathological form of necrosis is it?
A Moist gangrene
B Bedsore
C Sequestrum
D Dry gangrene
E Infarction

11
A denaturation of proteins can be found in some substances. Specify the substance that is
used for the incomplete denaturation of hemoglobin:
A Urea
B Toluene
C Sulfuric acid
D Nitric acid
E Sodium hydroxide

12
During surgery in a 17-year-old patient it was revealed the tumour of 4,5х5,0х3,5 sm in
size on the lower surface of the liver with subserose localization, of dark-red color.
On the section tumour has cavities with marked amount of blood. What is preliminary
diagnosis?
A Cavernous hemangioma
B Capillar hemangioma
C Hemangiopericytoma
D Hemangioendothelioma
E Lymphangioma

13
A sick man with high temperature and a lot of tiny wounds on the body has been admitted
to the hospital. Lice have been found in the folds of his clothing. What disease can be
suspected in the patient?
A Epidemic typhus
B Tularemia
C Scabies
D Malaria
E Plague

14
On autopsy it is revealed that kidneys are enlarged, surface is large-granular because of
multiple cavities with smooth wall, which are filled with clear fluid. What kidney disease did
the patient have?
A Polycystic kidney
B Necrotic nephrosis
C Pyelonephritis
D Glomerulonephritis
E Infarction

15
On autopsy it is revealed enlarged dense right lung, fibrin layers on the pleura. Lung
tissue is light green color on incision with muddy liqued exudates. What lung disease are
these symptoms typical for?

3
A Lung-fever
B Bronchopneumonia,

C Interstitial pneumonia
D Pulmonary gangreneі
E Fibrosing alveolitis

16
On autopsy it is revealed: soft arachnoid membrane of the upper parts of cerebral
hemisphere is plethoric, it is of yellowish-green color, soaked with purulent and fibrose
exudate, it lookes like cap. For what disease is it characteristical picture
A Meningococcal meningitis
B Tuberculous meningitis
C Influenza meningitis
D Meningitis at anthrax
E Meningitis at typhus

17
On autopsy of the 58-year-old man it is revealed: mitral valve is deformed, thickened, not
totally closed. Microscopically: centers of collagen fibers are eosinophilic, have positive
fibrin reaction. The most likely it is:
A Fibrinoid swelling
B Fibrinoid inflammation
C Mucoid swelling
D Hyalinosis
E Amyloidosis

18
Chronic inflammation and transformation of the one-layer ciliated epithelium into
multiple-layers flat epithelium was revealed in the thickened mucous membrane of the
bronchus bioptate of the patient with smoke abuse. Which of the processes is the most
likely?
A Metaplasia
B Hyperplasia of the epithelium
C Squamous cancer
D Leucoplacia
E Epithelium hypertrophy

19
For a long time a 49-year-old woman was suffering from glomerulonephritis which caused
death.On autopsy it was revealed that kidneys size was 7х3х2.5 sm, weight is 65,0 g, they
are dence and small-grained. Microscopically: fibrinogenous inflammation of serous and
mucous capsules, dystrophic changes of parenchymatous organs, brain edema. What
complication can cause such changes of serous capsules and inner organs?
A Uraemia
B Anemia
C Sepsis
D DIC-syndrome
E Thrombopenia

4
20
A patient died under conditions of cardiovascular insufficiency. Autopsy results:
postinfarction cardiosclerosis, myocardium hypertrophy and dilatation of its cavities,
especially of its right ventricle. Liver is enlarged, its surface is smooth, incision revealed
that it was plethoric, with dark-red specks against the background of brownish tissue.
Histologically: plethora of central parts of lobules; peritheral parts around portal tracts
contain hepatocytes in a state of adipose degeneration. How are these liver changes
called?
A Nutmeg liver
B Pseudonutmeg liver
C Amyloidosis
D Liver cirrhosis
E Liver steatosis

21
A 59-year-old man has signs of the parenchymatous jaundice and portal hypertension. On
histological examination of the puncture of the liver bioptate, it was revealed: beam-lobule
structure is affected, part of hepatocytes has signs of fat dystrophy, port-portal connective
tissue septa with formation of pseudo-lobules,with periportal lympho-macrophage
infiltrations. What is the most probable diagnosis?
A Liver cirrhosis
B Alcohol hepatitis
C Chronic hepatosis
D Viral hepatitis
E Toxic dystrophy

22
On microscopic examination of the enlarged neck gland of a 14-year-old girl it was
revealed destruction of the tissue structure of the node, absence of the lymph follicles,
sclerotic and necrosis parts, cell constitution of the node is polymorphous, lymphocites,
eosinophiles, atypical cells of the large size with multiple-lobule nuclei
(Beresovsky-Shternberg cells) and onenucleus cells of the large size are present. What is
the most likely diagnosis?
A Lymphogranulomatous
B Acute lympholeucosis
C Chronic lympholeucosis
D Berkitt's lymphoma
E Fungous mycosis

23
A female patient suffering from bronchial asthma had got a viral infection that provoked
status asthmaticus with fatal outcome. Histological examination of lungs revealed spasm
and edema of bronchioles, apparent infiltration of their walls with lymphocytes, eosinophils
and other leukocytes; labrocyte degranulation. What mechanism of hypersensitivity
underlies the described alterations?
A Reagin reaction
B Inflammatory
C Autoimmune
D Immune complex
E Immune cytolysis

5
24
On autopsy of the 40-year-old woman suffering from rheumatic arthritis, the enlarged solid
spleen was revealed. On section its tissue is of the mahogany color with enlarged follicles,
which look like semi-transparent grayish-whitish grains. What pathological process is the
most likely?
A Sago spleen
B Glaze spleen
C Waxy spleen
D Hyaline spleen
E Porphyric spleen

25
Local lymphonodules enlarged near the infected wound. Increased amount of
macrophages, lymphocytes, lymphatic follicles in the cortical layer and large amount of
plasma cells were revealed on histological examination. What process in the lymphatic
nodules represent these histological changes?
A Antigen stimulation
B Acquired insufficiency of the lymphoid tissue
C Innate insufficiency of the lymphoid tissue
D Tumour transformation
E Hypersensibility reaction

26
On autopsy of the man with alcohol abuse for a long time it was revealed: dense,
small-knobby, small size liver. Microscopically: small pseudo-lobules, divided with thin
layers of connective tissue with lymphomacrophagial infiltrates; hepatocytes in the state of
globular fatty dystrophy. What is the most likely diagnosis?
A Alcohol cirrhosis
B Chronic active alcohol hepatitis
C Chronic persistent alcohol hepatitis
D Toxic liver dystrophy
E Fatty hepatosis

27
An 8-year-old child was admitted to the infectious department with fever (up to $38^oC$)
and punctuate bright-red skin rash. The child was diagnosed as having scarlet fever.
Objectively: mucous membrane of pharynx is apparently hyperaemic and edematic, the
tonsils are enlarged and have dull yellowish-grey foci with some black areas. What
inflammation is the reason for the pharynx alterations?
A Purulent necrotic
B Fibrinous
C Haemorrhagic
D Serous
E Catarrhal

28
A 30-year-old patient with bacteriologically proved dysentery developed the signs of
paraproctitis. What is the stage of local changes in this patient?
A Ulceration stage
B Fibrinous colitis
C Follicular colitis
D Catarrhal colitis
E Healing of the ulcers stage

6
29
A patient who has been abusing tobacco smoking for a long time has got cough
accompanied by excretion of viscous mucus; weakness after minor physical stress, pale
skin. The patient has also lost 12,0 kg of body weight. Endoscopic examination of biosy
material his illness was diagnosed as squamous cell carcinoma. Name a pathological
process that preceded formation of the tumour:
A Metaplasia
B Hypoplasia
C Hyperplasia
D Necrosis
E Sclerosis

30
Diagnostic scraping was performed to the woman with dysfunctional uterine bleeding.
Multiple convoluted glands, ganglially dilated cavities of some glands were revealed
histologically in the scrape. Name the type of general pathological process.
A Glandulo-gangliac hyperplasia
B Atrophy
C Metaplasia
D Displasia
E Hypertrophic excrescence

31
Tuberculine was injected intracutaneously to the child for tuberculin test. Marked
hyperemia, tissue infiltration developed on the place of injection in 24 hours. What
mechanism caused these modifications?
A Cells cytotoxity
B Reagin type cytotoxity
C Antibody cytotoxity
D Granuloma formation
E Immunocomplex cytotoxity

32
The intraoperational biopsy of mammal gland has revealed the signs of atypical tissue with
disorder of parenchyma stroma proportion with domination of the last, gland structures of
the different size and shape, lined with single-layer proliferative epithelium. What is the
most appropriate diagnosis?
A Fibroadenoma
B Papilloma
C Noninfiltrative cancer
D Infiltrative cancer
E Mastitis

33
Arterial hypertension, hyperglycemia, glucosuria were observed clinically for a long time in
the patient with upper type of obesity. Death was due to the cerebral haemorrhage.
Basophilic hypophysis adenoma, hyperplasia of adrenal gland cortex were revealed on
pathomorphological examination. What is the likely diagnosis?
A Cushing disease
B Diabetes mellitus
C Acromegaly

7
D Hypophysis nanism
E Adiposogenitalis dystrophy

34
On autopsy it was revealed: large (1-2 cm) brownish-red, easy crumbling formations
covering ulcerative defects on the external surface of the aortic valve. What is the most
likely diagnosis?
A Polypus-ulcerative endocarditis
B Recurrent warty endocarditis
C Acute warty endocarditis
D Fibroplastic endocarditis
E Diffusive endocarditis

35
Purulent endometritis with fatal outcome was progressing in the woman after abortion
performed not at the hospital. On autopsy multiple lung abscesses, subcapsule ulcers in
the kidneys, spleen hyperplasia were revealed. What form of sepsis developed in the
patient?
A Septopyemia
B Septicemia
C Chroniosepsis
D Lung sepsis
E Urosepsis

36
Autopsy of a 73-year-old man who had been suffering from the coronary heart disease
along with cardiac insufficiency for a long time revealed: nutmeg liver, brown induration of
lungs, cyanotic induration of kidneys and spleen. What kind of circulation disorder was the
cause of such effects?
A General chronic venous congestion
B Arterial hyperaemia
C General acute venous congestion
D Acute anaemia
E Chronic anaemia

37
A 22 year old patient from the West Ukraine complains of laboured nasal breathing.
Morphological examination of biopsy material of nasal mucous membrane revealed
lymphoid, epithelioid, plasma cells as well as Mikulicz's cells. What is the most probable
diagnosis?
A Rhinoscleroma
B Glanders
C Tuberculosis
D Leprosy
E Syphilis

38
Autopsy of a man who had been working as a miner for many years and died from
cardiopulmonary decompensation revealed that his lungs were airless, sclerosed, their
apexex had emphysematous changes, the lung surface was greyish-black, the incised lung
tissue was coal-black. What disease caused death?
A Anthracosis

8
B Silicosis
C Talcosis
D Asbestosis
E Aluminosis

39
Examination of coronary arteries revealed atherosclerotic calcific plaques that close vessel
lumen by 1/3. The muscle has multiple whitish layers of connective tissue. What process
was revealed in myocardium?
A Diffuse cardiosclerosis
B Tiger heart
C Postinfarction cardiosclerosis
D Myocarditis
E Myocardium infarction

40
A 63 year old male patient who had been suffering from chronic diffuse obstructive
disease, pulmonary emphysema, for 15 years died from cardiac insufficiency. Autopsy
revealed nutmeg liver cirrhosis, cyanotic induration of kidneys and spleen, ascites,
edemata of lower limbs. These changes of internal organs are typical for the following
disease:
A Chronic right-ventricular insufficiency
B Acute right-ventricular insufficiency
C Chronic left-ventricular insufficiency
D Acute left-ventricular insufficiency
E General cardiac insufficiency

41
Microscopical examination of an enlarged cervical lymph node revealed blurring of its
structure, absence of lymphoid follicles; all the microscopic fields showed cells with
roundish nuclei and thin limbus of basophil cytoplasm. It is known from the clinical data that
other groups of lymph nodes are also enlarged as well as spleen and liver. What disease
might be suspected?
A Lymphoid leukosis
B Lymphogranulomatosis
C Lymphosarcoma
D Myeloid leukosis
E Multiple myeloma

42
A worker of a cattle farm fell acutely ill and then died from the progressing intoxication.
Autopsy revealed enlarged, hyposthenic spleen of dark-cherry colour when dissected;
excessive pulp scraping. At the base and fornix of brain pia maters are edematous, soaked
with blood, dark-red ("scarlet hat"). Microscopic examination revealed serous
haemorrhagic inflammation of brain tissues and tunics along with destruction of small
vessel walls. What is the most likely diagnosis?
A Anthrax
B Tularemia
C Brucellosis
D Plaque
E Cholera

9
43
Histological examination of a skin tissue sampling revealed granulomas consisting of
macrophagal nodules with lymphocytes and plasmatic cells. There are also some big
macrophages with fatty vacuoles containing causative agents of a disease packed up in
form of spheres (Virchow's cells). Granulation tissue is well vascularized. What disease is
this granuloma typical for?
A Lepra
B Tuberculosis
C Syphilis
D Rhinoscleroma
E Glanders

44
A 40 year old man noticed a reddening and an edema of skin in the area of his neck that
later developed into a small abscess. The incised focus is dense, yellowish-green. The pus
contains white granules. Histological examination revealed drusen of a fungus, plasmatic
and xanthome cells, macrophages. What type of mycosis is the most probable?
A Actinomycosis
B Aspergillosis
C Candidosis
D Sporotrichosis
E Coccidioidomycosis

45
A physician examined a patient and found inguinal hernia. Through what anatomic
formation does it penetrate into the skin?
A $Hiatus$ $saphenus$
B $Anulus$ $femoralis$
C $Canalis$ $adductorius$
D $Lacuna$ $musculorum$
E $Anulus$ $inguinalis$ $superficialis$

46
Autopsy of a man who died from burn disease revealed brain edema, liver enlargement as
well as enlargement of kidneys with wide light-grey cortical layer and plethoric medullary
area. Microscopic examination revealed necrosis of tubules of main segments along with
destruction of basal membranes, intersticium edema with leukocytic infiltration and
haemorrhages. What is the most probable postmortem diagnosis?
A Necrotic nephrosis
B Tubulointerstitial nephritis
C Pyelonephritis
D Gouty kidney
E Myeloma kidney

47
A 30 year old man had been suffering from acute respiratory disease and died from
cardiopulmonary decompensation. Autopsy revealed fibrinous-haemorrhagic inflammation
in the mucous membrane of larynx and trachea, destructive panbronchitis, enlarged lungs
that look black due to the multiple abcesses, haemorrhages, necrosis. What is the most
probable postmortem diagnosis?
A Influenza
B Parainfluenza
C Respiratory syncytial infection

10
D Measles
E Adenoviral infection

48
A man with a wound of his limb that had been suppurating for a long time died from
intioxication. Autopsy revealed extreme emaciation, dehydration, brown atrophy of liver,
myocardium, spleen and cross-striated muscles as well as renal amyloidosis. What
diagnosis corresponds with the described picture?
A Chroniosepsis
B Septicopyemia
C Septicemia
D Chernogubov's syndrome
E Brucellosis

49
6 months after delivery a woman had uterine bleeding. Gynecological examination revealed
in the uterine cavity a dark-red tissue with multiple cavities that resembled of "sponge".
Microscopic examination of the tumour revealed some atypic light epithelial Langhans cells
and giant cells of cyncytiotrophoblast in blood lacunas. What tumour is it?
A Chorioepithelioma
B Squamous cell nonkeratinous carcinoma
C Adenocarcinoma
D Fibromyoma
E Vesicular mole

50
A patient with android-type obesity had been suffering from arterial hypertension,
hyperglycemia, glycosuria for a long time and died from the cerebral haemorrhage.
Pathologic examination revealed pituitary basophil adenoma, adrenal cortex hyperplasia.
What is the most likely diagnosis?
A Itsenko-Cushing's syndrome
B Diabetes mellitus
C Acromegalia
D Pituitary nanism
E Adiposogenital dystrophy

51
Mucous membrane of the right palatine tonsil has a painless ulcer with smooth lacquer
fundus and regular cartilagenous edges. Microscopically: inflammatory infiltration that
consists of lymphocytes, plasmocytes, a small number of neutrophils and epithelioid cells;
endovasculitis and perivasculitis. What disease is it?
A Syphilis
B Actinomycosis
C Tuberculosis
D Pharyngeal diphtheria
E Ulcerous necrotic Vincent's angina

52
Autopsy of a man with a malignant stomach tumour who had died from cancer intoxication
revealed in the posteroinferior lung fields some dense, grayish-red irregular foci protruding
above the section surface. Microscopic examination revealed exudate containing a large
amount of neutrophils in the lumen and walls of small bronchi and alveoles. Such

11
pulmonary alterations indicate the following disease:
A Acute purulent bronchopneumonia
B Acute bronchitis
C Croupous pneumonia
D Intermittent pneumonia
E Acute serous bronchopneumonia

53
Microscopical examination of a removed appendix revealed an edema, diffuse neutrophilic
infiltration of appendix wall along with necrosis and defect of mucous membrane with
affection of its muscle plate. What appendicitis form was developed?
A Ulcerophlegmonous
B Phlegmonous
C Gangrenous
D Superficial
E Apostematous

54
A 39 y.o. woman went through an operation in course of which surgeons removed her
uterine tube that was enlarged and a part of an ovary with a big cyst. Histological
examination of a tube wall revealed decidual cells, chorion villi. What was the most
probable diagnosis made after examination of the uterine tube?
A Tubal pregnancy
B Placental polyp
C Choriocarcinoma
D Papyraceous fetus
E Lithopedion

55
Autopsy of a 1,5-year-old child revealed haemorrhagic skin rash, moderate hyperaemia
and edema of nasopharyngeal mucous membrane, small haemorrhages in the mucous
membranes and internal organs; dramatic dystrophic alterations in liver and myocardium;
acute necrotic nephrosis; massive haemorrhages in the adrenal glands. What disease are
these alterations the most typical for?
A Meningococcal infection
B Scarlet fever
C Diphtheria
D Measles
E Epidemic typhus

56
48 hours after performing tuberculin test (Mantoux test) to a child a 10 mm papule
appeared on the spot of tuberculin introduction. What hypersensitivity mechanism
underlies these changes?
A Cellular cytotoxicity
B Anaphylaxis
C Antibody-dependent cytotoxicity
D Immune complex cytotoxicity
E Granulomatosis

57
Colonoscopy of a patient ill with dysentery revealed that mucous membrane of his large

12
intestine is hyperemic, edematic, its surface was covered with grey-and-green coats. Name
the morphological form of dysenteric collitis:
A Fibrinous
B Catarrhal
C Ulcerous
D Purulent
E Necrotic

58
A patient has been syffering from diarrhea for 5 day. On the fith day colonoscopy revealed
that membrane of rectum was inflamed, there were greyish-green films closely adhering to
the subjacent tissue. What is the most probable diagnosis?
A Dysentery
B Typhoid fever
C Nonspecific ulcerous colitis
D Salmonellosis
E Crohn's disease

59
Autopsy of a 48 y.o. man revealed a round formation 5 cm in diameter with clear-cut
outlines in the region of the 1st segment of his right lung. This formation was encircled with
a thin layer of connective tissue full of white brittle masses. Make a diagnosis of the
secondary tuberculosis form:
A Tuberculoma
B Caseous pneumonia
C Acute cavernous tuberculosis
D Acute focal tuberculosis
E Fibrous cavernous tuberculosis

60
A man had worked in a coal mine for over 20 years. After his death autopsy revealed that
his lungs were dense, grayish-black and had large areas of neogenic connective tissue
containing a lot of microphages with black pigment in the cytoplasm. What is the most likely
diagnosis?
A Anthracosis
B Anthracosilicosis
C Silicoanthracosis
D Talcosis
E Siderosis

61
Skin of a man who died from cardiac insufficiency has an eruption in form of spots and
specks. There are also bedsores in the area of sacrum and spinous vertebral processes.
Microscopical examination of CNS, skin, adrenal glands revealed in the vessels of
microcirculatory bed and in small arteries destructive-proliferative endothrombovasculitis
with Popov's granulomas; interstitial myocarditis. What diagnosis corresponds with the
described picture?
A Spotted fever
B Q fever
C Enteric fever
D Nodular periarteritis
E HIV

13
62
Autopsy of a 17 year old girl who died from pulmonary failure revealed a small area of
caseous necrosis in the inferior lobe of the right lung, and occurences of caseous necrosis
in the bronchopulmonary, bronchial and bifurcational lymph nodes. What is the most
probable postmortem diagnosis?
A Primary tuberculosis
B Hematogenous progression of primary tuberculosis
C Hematogenous tuberculosis with predominant lung affection
D Tuberculoma
E Caseous pneumonia under secondary tuberculosis

63
Autopsy of a man who died from the sepsis in his femoral bone revealed phlegmonous
inflammation that affected the marrow, haversian canals and periosteum. Under the
periosteum there are multiple abscesses, adjoining soft tissues of thigh also have signs of
phlegmonous inflammation. What pathological process was described?
A Acute hematogenous osteomyelitis
B Osteoporosis
C Chronic hematogenous osteomielitis
D Osteopetrosis
E-

64
An experimental animal was first sensibilized whereupon an antigen dose was introduced
subcutaneously. This injection resulted in the development of a fibrinous inflammation with
alteration of vessel walls, basal substance and fibrous structures of connective tissue in
form of mucoid and fibrinoid swelling and necrosis. What immunological reaction took
place?
A Immediate hypersensitivity
B Delayed-type hypersensitivity
C Reaction of transplantation immunity
D Normergic reaction
E Granulomatosis

65
Examination of a 55 year old woman revealed under the skin of submandibular area a
movable slowly growing pasty formation with distinct borders 1,0x0,7 cm large. Histological
examination revealed lipocytes that form segments of diffrent forms and sizes separated
from each other by thin layers of connective tissue with vessels. What is the most probable
diagnosis?
A Lipoma
B Fibroma
C Angioma
D Liposarcoma
E Fibrosarcoma

66
A 4 year old child complained of pain during deglutition, indisposition. Objectively: palatine
arches and tonsils are moderately edematic and hyperemic, there are greyish-white films
up to 1 mm thick closely adhering to the subjacent tissues. What pathological process are
these changes typical for?
A Inflammation

14
B Dystrophy
C Necrosis
D Metaplasia
E Organization

67
A 9 m.o. child has delayed dentition, it is also out of order. Upper jaw configuration is
horizontal ("high" palate); microscopically - irregular mineralization of tooth enamel,
wrinkled enamel prisms, some of them are vacuolized. Predentin zone is extended; there
are solitary denticles. What disease is it?
A Early rickets
B Late rickets
C Osteomalacia
D Gout
E Hypervitaminosis D

68
Microscopical renal examination of a 36 y.o. woman who died from renal insufficiency
revealed in the glomerules proliferation of capsule nephrothelium as well as of podocytes
and phagocytes accompanied by formation of "crescents", capillary loop necrosis, fibrinous
thrombs in their lumens; sclerosis and hyalinosis of glomerules, atrophy of tubules and
fibrosis of renal stroma. What is the most probable diagnosis?
A Subacute glomerulonephritis
B Acute glomerulonephritis
C Chronic glomerulonephritis
D Focal segmentary sclerosis
E Membranous nephropathy

69
A forensic medical expert examines the body of a 58 y.o. man who had been consuming
large amounts of alcochol for a long time and died at home. Microscopicaly: the right lung
is dense and enlarged, its incision revealed that the tissue is greyish and homogenous,
pleura is covered with greyish layers. Microscopically - alveolar cavities contain fibrin,
hemolyzed erythrocytes. Make a diagnosis:
A Croupous pneumonia
B Focal pneumonia
C Interstitial pneumonia
D Primary pulmonary tuberculosis
E Caseous pneumonia

70
Autopsy of a 50-year-old man revealed the following changes: his right lung was
moderately compact in all parts, the dissected tissue was found to be airless, fine-grained,
dryish. Visceral pleura had greyish-brown layers of fibrin. What is the most likely diagnosis?

A Croupous pneumonia
B Tuberculosis
C Bronchopneumonia
D Interstitial pneumonia
E Pneumofibrosis

71

15
Autopsy of a 56 y.o. man revealed in the right temporal part of brain a big focus of
softened grey matter that was semi-liquid and light grey. Arteries of cerebral tela contain
multiple whitish-yellow thickenings of intima that abruptly narrow the lumen. What is your
diagnosis?
A Ischemic stroke
B Brain abscess
C Hemorrhage
D Hemorrhagic infarction
E Brain edema

72
A 22 y.o. woman has enlarged lymph nodes. Histologically: a lymph node contains
lymphocytes, histiocytes, reticular cells, small and big Hodgkin's cells, multinucleated
Sternberg cells, isolated foci of caseous necrosis. What disease are these changes typical
for?
A Lymphogranulomatosis
B Lymphosarcoma
C Chronic leukosis
D Acute leukosis
E Lung cancer metastasis

73
Analysis of a punction biopsy material of liver revealed hepatocyte dystrophy with necroses
as well as sclerosis with disorder of beam and lobulous structure, with formation of
pseudolobules and regenerative nodes. What is the most probable diagnosis:
A Liver cirrhosis
B Chronic hepatosis
C Chronic hepatitis
D Progressive massive liver necrosis
E Acute hepatitis

74
Autopsy of a man, who had been suffering from the multiple bronchiectasis for 5 years and
died from chronic renal insufficiency, revealed that kidneys were dense and enlarged, with
thickened cortical layer of white colour with greasy lustre. What renal disease might be
suspected?
A Secondary amyloidosis
B Glomerulonephritis
C Chronic pyelonephritis
D Necrotic nephrosis
E-

75
Autopsy of a 49-year-old woman who died from chronic renal insufficiency, revealed:
kidneys were dense, reduced, multicoloured, with haemorrhagic areas. Microscopic
examination revealed some hematoxylin bodies in the nuclei of the renal tubule epithelium;
"wire-loop" thickening of the glomerular capillary basement membrane; here and there in
the capillaries some hyaline thrombi and foci of fibrinoid necrosis were present. What is the
most likely diagnosis?
A Systemic lupus erythematosus
B Rheumatism
C Arteriosclerotic pneumosclerosis
D Amyloidosis

16
E Atherosclerotic nephrosclerosis

76
Unpainfull formation without marked borders appeared in the soft tissues of the thigh in the
young man. On the tissue bioptate the formation lookes like a meat of a fish, consisting of
the immature fibroblast-like cells with multiple mitosis, which grow through the muscles.
What is the most likely diagnosis?
A Fibrosarcoma
B Myosarcoma
C Fibroma
D Cancer
E Myoma

77
A 45 y.o. patient consulted a doctor about plaque-shaped formation on his neck.
Histological examination of biopsy skin material revealed tumourous cells of round and oval
form with thin ring of basophilic cytoplasma that resemble of cells of basal epidermal layer.
What tumour is it?
A Basalioma
B Epidermal cancer
C Hydradenoma
D Trichoepithelioma
E Syringoadenoma

78
A 63 y.o. man fell ill with acute tracheitis and bronchitis accompanied by bronchial
pneumonia. On the 10th day the patient died from cardiopulmonary insufficiency. Autopsy
revealed fibrinous hemorrhagic laryngotracheobronchitis; lungs were enlarged, their
incision revealed the "coal-miner's" effect caused by interlacing of sections of bronchial
pneumonia, hemorrhages into the pulmonary parenchyma, acute abscesses and
atelectases. Internal organs have discirculatory and dystrophic changes. What is the most
probable diagnosis?
A Influenza, severe form
B Moderately severe influenza
C Parainfluenza
D Respiratory syncytial infection
E Adenoviral infection

79
Autopsy of a man who died from influenza revealed that his heart was slightly enlarged,
pastous, myocardium was dull and had specks. Microscopical examination of myocardium
revealed signs of parenchymatous adipose and hydropic dystrophy; stroma was edematic
with poor macrophagal and lymphocytic infiltration, vessels were plethoric; perivascular
analysis revealed petechial hemorrhages. What type of myocarditis was developed in this
case?
A Serous diffuse
B Interstitial proliferative
C Serous focal
D Purulent
E Granulomatous

80

17
A boy is 7 y.o. Objectively: against the background of hyperemic skin there is knobby
bright-pink rash on his forehead, neck, at the bottom of abdomen, in the popliteal spaces;
nasolabial triangle is pale. Examination of oropharyngeal surface revealed localized
bright-red hyperemia; tonsils are swollen, soft, lacunas contain pus, tongue is crimson.
Cervical lymph nodes are enlarged, dense and painful. What is the most probable
diagnosis?
A Scarlet fever
B Rubella
C Whooping cough
D Diphtheria
E Infectious mononucleosis

81
Gynecological examination of the uterine cervix in a 30-year-old woman revealed some
bright-red lustrous spots that easily bleed when touched. Biopsy showed that a part of the
uterine cervix was covered with cylindrical epithelium with papillary outgrowths; in the depth
of tissue the growth of glands was present. What pathology of the uterine cervix was
revealed?
A Pseudoerosion
B True erosion
C Endocervicitis
D Glandular hyperplasia
E Leukoplakia

82
A stillborn child was found to have thickened skin resembling of the tortoise shell,
underdeveloped auricles. Histological examination of skin revealed hyperkeratosis,
atrophy of the granular epidermis layer; inflammatory changes were not present. What is
the most likely diagnosis?
A Ichthyosis
B Leukoplakia
C Xerodermia
D Erythroplakia
E Dermatomyositis

83
A pathology-histology laboratory received a vermiform appendix up to 2,0 cm thick. Its
serous membrane was pale, thick and covered with yellowish-green films. The wall was
flaccid, of grayish-red colour. The appendix lumen was dilated and filled with
yellowish-green substance. Histological examination revealed that the appendix wall was
infiltrated with neutrophils. Specify the appendix disease:
A Acute phlegmonous appendicitis
B Acute gangrenous appendicitis
C Acute superficial appendicitis
D Acute simple appendicitis
E Chronic appendicitis

84
A 46 year old patient who had been suffering from tuberculosis for 6 years died from
massive pulmonary haemorrhage. Autopsy revealed different-sixed foci of sclerosis and
caseous necrosis in lungs, in the upper part of the right lung there was a cavity 5 cm in
diameter with dense grey walls, the cavity contained liquid blood and blood clots. What type
of tuberculosis is it?

18
A Fibrocavernous
B Acute cavernous
C Infiltrative
D Fibrous focal
E Acute focal

85
A patient died from cardiopulmonary decompensation. Histological examination revealed
diffused pulmonary lesion together with interstitial edema, infiltration of tissue by
limphocytes, macrophages, plasmocytes; pulmonary fibrosis, panacinar emphysema. What
disease corresponds with the described picture?
A Fibrosing alveolitis
B Chronic bronchitis
C Bronchopneumonia
D Pulmonary atelectasis
E Bronchial asthma

86
A 50 year old patient has been taking treatment thrice for the last 6 months because of
fractures caused by domestic accidents. Microscopical examination of bony tissue revealed
foci of lacunar resolution, giant-cell granulomas in the tumour-like formations, cysts. Bony
tissue was substituted by fibrous connective tissue. Examination revealed also adenoma of
parathyroid gland and hypercalcemia. What is the most probable diagnosis?
A Parathyroid osteodystrophy
B Myelomatosis
C Osteomyelitis
D Osteopetrosis
E Paget's disease

87
2 hours after a skeletal extension was performed to a 27 year old patient with multiple
traumas (closed injury of chest, closed fracture of right thigh) his condition abruptly
became worse and the patient died from acute cardiopulmonary decompensation.
Histological examination of pulmonary and cerebral vessels stained with Sudan III revealed
orange drops occluding the vessel lumen. What complication of polytrauma was
developed?
A Fat embolism
B Gaseous embolism
C Microbal embolism
D Thromboembolism
E Air embolism

88
A 50 year old patient underwent resection of tumour of large intestine wall. Microscopically
it presents itself as fascicles of divergent collagen fibers of different thickness and form and
some monomorphous fusiform cells that are irregularly distributed among the fibers.
Cellular atypia is not evident. What tumour is it?
A Hard fibroma
B Fibromyoma
C Soft fibroma
D Desmoma
E Fibrosarcoma

19
89
Autopsy of a 5 year old child revealed in the area of vermis of cerebellum a soft
greyish-pink node 2 cm in diameter with areas of haemorrhage. Histologically this tumour
consisted of atypical monomorphous small roundish cells with big polymorphous nuclei.
What tumour is it?
A Medulloblastoma
B Meningioma
C Glioblastoma
D Astrocytoma
E Oligodendroglioma

90
In course of severe respiratory viral infection there appeared clinical signs of progressing
cardiac insufficiency that caused death of a patient in the 2nd week of disease. Autopsy
revealed that the heart was sluggish, with significant cavity dilatation. Histological
examination of myocardium revealed plephora of microvessels and diffuse infiltration of
stroma by lymphocytes and histiocytes. What disease corresponds with the described
picture?
A Myocarditis
B Stenocardia
C Acute coronary insufficiency
D Myocardium infarction
E Cardiomyopathy

91
A 38 year old patient with full-blown jaundice, small cutaneous hemorrhages, general
weakness and loss of appetite underwent puncture biopsy of liver. Histological examination
revealed disseminated dystrophy, hepatocyte necrosis, Councilman's bodies. Lobule
periphery has signs of significant infiltration by lymphocytes, there are also individual
multinuclear hepatocytes. What is the most probable diagnosis?
A Acute viral hepatitis
B Acute alcoholic hepatitis
C Miliary hepatic cirrhosis
D Toxic degeneration of liver
E Chronic hepatitis

92
A 20 year old patient died from intoxication 8 days after artificial illegal abortion performed
in her 14-15th week of pregnancy. Autopsy of the corpse revealed yellowish colour of eye
sclera and of skin, necrotic suppurative endometritis, multiple pulmonary abscesses,
spleen hyperplasia with a big number of neutrophils in its sinuses. What complication after
abortion was developed?
A Septicopyemia
B Septicemia
C Hemorrhagic shock
D Chroniosepsis
E Viral hepatitis type A

93
A section of the left lung was found to have an area of dense red tissue. The area was
cone-shaped, stood out distinctly from the healthy tissue, with its base directed to the
pleura. The dissected tissue was granular, dark-red. What is the most likely diagnosis?

20
A Haemorrhagic infarction
B Lung abscess
C Lung gangrene
D Primary tuberculous affection
E Croupous pneumonia

94
A patient has a cluster of matted together dense lymph nodes on his neck. Histological
examination of a removed lymph node revealed proliferation of reticular cells, presense of
Reed-Sternberg cells. What disease is meant?

A Lymphogranulomatosis
B Lymphoblastic leukosis
C Myeloblastic leukosis
D Myelocytic leukosis
E Lymphocytic leukosis

95
A patient had been suffering from profuse diarrhea and vomiting for 2 days. He died from
acute dehydration. Autopsy revealed that the intestinal wall was edematic and hyperemic,
with multiple haemorrhages in the mucous membrane. Intestine lumen contains whitish fluid
resembling of rice water. What disease caused death?
A Cholera
B Dysentery
C Salmonellosis
D Typhoid fever
E Enterocolitis

96
Examination of a 66 year old patient revealed a lytic tumour in the locus of pathological rib
fracture. Histologically this tumour consists of atypical plasmoblasts. Further examination
revealed osteoporosis in the bones of vertebral column and pelvis. These changes are
typical for:
A Myelomatosis
B Tuberculous osteomyelitis
C Ewing's osteosarcoma
D Neuroblastoma
E Metastatic lung cancer

97
A patient died from acute cardiac insufficiency, among clinical presentations there was
gastrointestinal haemorrhage. Examination of mucous membrane of sromach revealed
some defects reaching myenteron; their edges and bottom were mostly even and loose,
some of them contained dark-red blood. What pathological process was revealed?
A Acute ulcers
B Chronic ulcers
C Erosions
D Thrombosis
E Inflammation

98
A 33 year old man died from uraemia. Autopsy revealed enlarged kidneys weighing 500,0

21
each and consisting of multiple cavities 0,5-2 cm in diameter. The cavities were full of
light-yellow transparent liquid. Renal pelvis and ureters had no pecularities. What renal
disease caused uraemia?
A Bilateral polycystic renal disease
B Chronic pyelonephritis
C Renal tumour
D Renal tuberculosis
E Rapidly progressing glomerulonephritis

99
A patient ill with tuberculosis died from progressing cardiopulmonary decompensation.
Autopsy in the area of the right lung apex revealed a cavity 5 cm in diameter
communicating with lumen of a segmental bronchus. On the inside cavity walls are covered
with caseous masses with epithelioid and Langhans cells beneath them. What
morphological form of tuberculosis is it?
A Acute cavernous tuberculosis
B Tuberculoma
C Caseous pneumonia
D Infiltrative tuberculosis
E Acute focal tuberculosis

100
A 30 year old woman has applied a lipstick with a fluorescent substance for a long time.
Then she got a limited erythema and slight peeling on her lip border, later there appeared
transversal striae and cracks. Special methods of microscopic examination of the affected
area helped to reveal sensibilized lymphocytes and macrophages in the connective tissue;
cytolysis. What type of immunological hypersensitivity was developed?
A IV type (cellular cytotoxicity)
B I type (reaginic)
C II type (antibody cytotoxicity)
D III type (immune complex cytotoxicity)
E Granulomatosis

101
Examination of a young woman revealed a node-like, soft and elastic homogenous tumour
of pinkish-white colour along the acoustic nerve. The tumour contains cell bundles with oval
nuclei. Cellular fibrous bundles form rhythmic structures made up by parallel rows of
regularly oriented cells arranged in form of a palisade with cell-free homogenous zone
(Verocay bodies) between them. What tumour is it?
A Neurinoma
B Malignant neurinoma
C Ganglioneurinoma
D Neuroblastoma
E Ganglioneuroblastoma

102
A 23 year old man has perforation of hard palate. In the area of this perforation there was
a compact well-defined formation. Microscopic examination of the resected formation
revealed a large focus of caseous necrosis surrounded by granulation tissue with
endovasculitis, cellular infiltration composed of lymphocytes, epithelioid cells (mainly
plasmocytes). What is the most probable diagnosis?
A Syphilis
B Tuberculosis

22
C Scleroma
D Sarcoma
E Leprosy

103
A 50 year old man who was referred to the hospital for treatment of cervical lymphadenitis
underwent test for induvidual sensitivity to penicillin. 30 seconds after he went hot all over,
AP dropped down to 0 mm Hg that led to cardiac arrest. Resuscitation was unsuccessful.
Autopsy results: acute venous plethora of internal organs; histological examination of skin
(from the site of injection) revealed degranulation of mast cells (tissue basophils).
Degranulation was also revealed in myocardium and lungs. What type of hypersensitivity
reaction is it?
A Anaphylactic
B Delayed-type hypersensitivity
C Complement-mediated cytotoxic
D Immunocomplex-mediated
E-

104
A 2 year old child had acute respiratory viral infection and died from cardiopulmonary
decompensation. Autopsy revealed that his right lung was hyperemic; in the 2nd, 6th and
10th segments and on the incision there were airless yellowish foci of irregular form, from
several mm up to 1 cm large. Microscopical examination revealed exudate consisting mainly
of neutrophils in the given areas of pulmonary tissue in the alveoles, bronchioles and
bronchial tubes. What is the most probable diagnosis?
A Focal pneumonia
B Interstitial pneumonia
C Croupous pneumonia
D Acute bronchitis
E Pulmonary abscess

105
The upper lobe of the right lung is enlarged, grey and airless, the inscision surface is
dripping with turbid liquid, the pleura has many fibrinogenous films; microscopical
examination of alveoles revealed exudate containing neutrophils, desquamated
alveolocytes and fibrin fibers. The bronchus wall is intact. What is the most probable
diagnosis?
A Croupous pneumonia
B Interstitial pneumonia
C Pulmonary abscess
D Focal pneumonia
E Influenzal pneumonia

106
A 28 year old patient had high arterial pressure, hematuria and facial edemata. In spite of
treatment renal insufficiency was progressing. 6 months later the patient died from uremia.
Microscopic examination of his kidneys and their glomerules revealed proliferation of
capsule nephrothelium and of podocytes with "demilune" formation, sclerosis and
hyalinosis of glomerules. What disease corresponds with the described picture?
A Subacute glomerulonephritis
B Acute pyelonephritis
C Nephrotic syndrome
D Chronic glomerulonephritis

23
E Acute glomerulonephritis

107
Autopsy of a man ill with severe hypothyroidism revealed that connective tissue, organ
stroma, adipose and cartilaginous tissues were swollen, semitransparent, mucus-like.
Microscopic examination of tissues revealed stellate cells having processes with mucus
between them. What type of dystrophy is it?
A Stromal-vascular carbohydrate
B Stromal-vascular adipose
C Stromal-vascular proteinaceous
D Parenchymatous proteinaceous
E Parenchymatous adipose

108
Examination of the anterior abdominal wall of a pregnant woman revealed a tumour-like
formation that arose on the spot of a tumour that was removed two years ago. The
neoplasm was well-defined, dense, 2х1 cm large. Histological examination revealed that the
tumour was composed of differentiated connective tissue with prevailing collagen fibres.
What tumour might be suspected?
A Desmoid
B Lipoma
C Fibrosarcoma
D Hibernoma
E Leiomyoma

Pathology of cell. Parenchymal dystrophy.


1. During an autopsy a parenchymal fatty dystrophy of the myocardium was diagnosed. What is the common or
descriptive name of the heart due to this dystrophy?
A. *'Tabby cat' heart ('Tiger's' heart)
B. Bovine heart
C. 'Hairy' heart
D. Solder plaque (bony heart)
E. Cor pulmonale
2. A patient with leukemia died from severe chronic anemia. An autopsy revealed an enlarged heart, with flabby
myocardium It had a dim pale-grey color, yellow spots and bars. Which pathological process was found in the heart
at post-mortem?
A. * Parenchymal fatty dystrophy
B. Vacuolar dystrophy.
C. Hydropic dystrophy.
D. Mesenchymal fatty dystrophy.
E. Mixed dystrophy.
3. A 53 year old patient died with symptoms of liver insufficiency. A post-mortem examination revealed the
enlarged, flabby, yellow-brown liver. Gross examination of the liver's section showed drops of fat. Microscopically:
hepatocytes on the peripheries of the hepatic lobules contained masses of small drops within the cytoplasm. Which
process most likely took place in the liver?
A. *Fatty dystrophy of the liver
B. Glucosylceramide lipidosis (Gaucher's disease)
C. Sphingolopidosis (Niemann-Pick disease)
D. Gangliosidosis (Тау-Sachs disease)
E. Systemic lipoidoses
4. A patient died from chronic cardiovascular insufficiency. At the post-mortem a 'tabby cat' heart was found. From
the side of the endocardium, a yellow-white striped pattern was noticeable. The myocardium was a dim with gray-
yellow color. Which process is most likely diagnosed?
A. * Fatty parenchymal dystrophy.

24
B. Carbohydrate dystrophy
C. Hydropic dystrophy.
D. Fatty mesenchymal dystrophy.
E. Amyloidosis.
5. A seven-year-old child presented with diphtheria of the pharynx. He subsequently died from acute cardiac
insufficiency. Post-mortem examination of the heart revealed that the cavities of the heart were extended
horizontally. Muscle of the heart were dim and flabby. Gross section showed motley appearance, with yellow areas.
Microscopically in the cytoplasm of some rnyocardial cells small vacu-oles were determined. The frozen sections
showed vacuoles within cells stained with sudan-III in orange color. Which type of dystrophy was found in
myocardial cells?
A. *Fatty dystrophy
B. Carbohydrate dystrophy
C. Vacuolar dystrophy
D. Hyaline dystrophy
E. Hydropic dystrophy
6. A man died due to cardiac insufficiency. At autopsy revealed the heart increased volume and flabby. A
myocardium was a clay-yellow color and dim. From the side of the endocardium a yellow-white striped pattern was
visible ('tabby cat'). Under the microscope the groups of myocardial cells lost their normal structure, their cytoplasm
contained shallow drops which were black when stained with sudan-IV. Which one of the following is the correct
diagnosis?
A. *Fatty dystrophy of myocardium
B. Cardiosclerosis
C. Rheumatic myocarditis
D. Obesity of the heart
E. Myomalacia
7. A 66-year-old male died from cardiac insufficiency. During the dissection an increase volume heart was found.
Observation of the heart revealed a flabby consistency with stretched chambers. The myocardium section had a dim,
clay-yellowish color. From the side of the endocardium a yellow-white striped pattern was present, which was
especially noted in the papillary muscles. Which pathological process is the most credible?
A. *Fatty dystrophy of the myocardium
B. Obesity of the heart
C. Dilatation cardiomyopathy
D. Myomalacia
E. Cardiosclerosis
8. A patient died from pulmonary-cardiac insufficiency. During the dissection a significantly enlarged anemic liver,
with yellow doughy consistencies was found. A liver specimen stained with hematoxylin and eosin exposed various
sizes of vacu-oles in the cytoplasm of the hepatocytes. Which one of the following dystrophies occurred?
A. *Parenchymal fatty
B. Parenchyma! carbohydrate
C. Hyaline
D. Mesenchymal fatty
E. Hydropic
9. A 38-year-old patient, suffering from chronic alcoholism and cirrhosis of the liver, developed profuse bleeding
due to varicose veins of the esophagus which resulted in death. During the autopsy a liver was noted to be
diminished in size with micronodular tuberosity. The organ was dense and rather yellow in color. A histological
evaluation of the cryostat specimens of the liver, stained with hematoxylin and eosin, revealed hepatocytes with
large, optically empty vacuoles. These vacuoles were black when stained with osmium acid. These optically empty
vacuoles hepatocytes indicate:
A. *Fatty dystrophy
B. Inclusions of hyaline.
C. Alcoholic hyaline (Mallory bodies)
D. Vacuolar dystrophy.
E. Carbohydrates dystrophy.
10. A 16-year-old girl presents with the symptoms of sharp pain during swallowing, lymph node enlargement of the
neck, and the body .temperature of 38°C. The mucous membrane of the tonsils revealed grayish membranes with
yellow tapes with were not easily separated from the defect. The patient's state progressively worsened which death
occurring on the 8th day of the disease due to cardiac insufficiency. Which of following histological changes in the
myocardial cells will be the most likely finding?
A. * Fatty dystrophy
B. Hydropic dystrophy
C. Hyaline dystrophy
D. Ballooning dystrophy
E. Mucous dystrophy

25
11. A 44-year-old woman died from chronic alcoholic intoxication. During the autopsy a significantly enlarged
liver of doughy consistency and rather yellowish color was found. Microscopically, after staining with hematoxylin
and eosin, cytoplasm of the hepatocytes contained optically empty vacuoles. Which type of dystrophy has taken
place?
A. *Parenchymal fatty dystrophy
B. Carbohydrate parenchymal dystrophy
C. Hyaline dystrophy
D. Mesenchymal fatty dystrophy
E. Hydropic dystrophy
12. A patient has died from toxic sepsis. During dissection a 'tabby cat' ('tiger's heart') was found. Microscopically,
lipids were detected in the cytoplasm of the myocardial cells. What is the primary morphological mechanism for
development for this dystrophy?
A. *Decomposition
B. Infiltration
C. Transformation
D. Pathological synthesis.
E. Neoplastic alterations.
13. Ultrastructural investigation of a liver biopsy, revealed that between the mitochondria there were numerous flat
cisterns and bubbles with secretory granules circumscribed with membrane. Name a cell structure with the
hyperplasic constituents?
A. * Golgi apparatus
B. Pinocytosis bubbles
C. Endoplasmic reticulum
D. Lysosoms
E. Microtubes
14. A 42-year-old female became ill with diphtheria and died from acute cardiac insufficiency. During dissection it
was noted that the heart cavities were extended and that the muscle of the heart was dim, motley and on a cut
surface had yellow areas. Which process was exposed in the myocardial cells?
A. *Fatty dystrophy
B. Carbohydrate dystrophy
C. Ballooning dystrophy
D. Hyaline dystrophy
E. Hydropic dystrophy
15. During the examination of a newborn, some apparent skin differences are noted. The skin is dry, with an uneven
surface and with the presence of grey plates which can be removed layer by layer. These changes are related to
which type of dystrophy?
A. *Homy dystrophy
B. Hydropic dystrophy
C. Hyaline dystrophy
D. Fibrinoid swelling
E. Mucoid swelling
16. A male patient had a prosthetic appliance on the lower jaw. The ventral surface of tongue revealed a dense, gray
plaque with a clear boundary. Histology revealed the thickened of the stratified squamous epithelium due granular
and to basal layers thickening, hyperkeratosis, acanthosis, lymphocyte's infiltration of connective tissue. Make a
diagnosis.
A. *Leukoplakia
B. Erythroplakia
C. Papilloma
D. Cancer in sity (infra-epithelial neoplasia)
E. Condyloma
17. During the preventive examination of a worker employed in the coal resins production the areas of thickening
and keratinization of the mucous membrane in the oral cavity were found. This occured mainly on the cheeks areas,
showing a whitish color with a rough surface. They were not painful. Which pathology is this related to?
A. * Leukoplakia
B. Papillomatosis
C. Glossitis
D. Stomatitis
E. Calcification
18. A 45-year old male is found to have a severe intoxication. A diagnosis of sepsis is made. Several days later he
dies. At autopsy, his myocardium grossly had a 'tiger heart' pattern. . Microscopically, lipids were detected in the
cytoplasm of cardiac hystiocytes. What morphogenetic mechanism prevails in the development of this dystrophy?
A. * Decomposition..
B. Infiltration.

26
C. Transformation
D. Abnonnal synthesis.
E. Colliquation.
19. A 3-month- old infant dies and autopsy is requested. Electron microscopic examination of liver tissue revealed a
great amount of flat cisterns and vesicles with secretory granules, surrounded by membrane, scattered among
numerous mitochondria. Which of the following cell ultra structures has been shown to be hyper-plastic?
A. *Golgi complex.
B. Pinocytic vesicles.
C. Endoplasmic reticulum.
D. Lysosomes.
E. Filaments.
20. A 36-year-old female develops liver failure followed with lethal outcome. Autopsy has shown an enlarged
liver of yellow-brown color and soft consistence. Drops of fat are noticed on the liver cut surface and on the scalpel.
Microscopically: hepatocytes at peripheral zone of a liver lobules contain small drops that fill cytoplasm and push
the nucleus to the periphery. What process in the liver do the following changes testify to?
A. *Fatty degeneration of liver.
B. Cerebrosidelipidosis (Gaucher's disease).
C. Sphingomyelinlipidosis (Niemann — Pick disease).
D. Gangliosidelipidosis (Тау —Sachs disease).
E. Generalized gangliosidosis (Norman—Landing disease).
21. Autopsy of a menopausal woman with a ling history of a chronic ischemic heart disease revealed soft and
enlarged heart. Its chambers were extended; the myocardium sectional view was lack-luster with grey- yellowish
coloring. An endocardium presented with yellow-white banding, most evident in papillary muscles. What is the
most likely pathological process in woman's heart?
A. *Fatty degeneration of myocardium.
B. Fatty heart.
C. Dilated cardiomyopathy.
D. Myomalation.
E. Cardiosclerosis.
22. A 77-year-old male with a dental prosthesis on his upper jaw is seen by his dentist because of a solid gray patch
on his tongue. A lesion has irregular contour, uneven surface, and clear borders. Microscopic investigation of its
biopsy revealed the thickening of stratified squamous epithelium, its hyperkeratosis, and acanthosis accompanied
with lymphocytes and macrophages infiltration of subjacent connective tissue. What is the most likely diagnosis?
A. *Leukoplakia.
B. Erythroplakia.
C. Papilloma.
D. Cancer in situ.
E. Condyloma.
Connective tissue's (mesenchymal) dystrophy
1. A 56 year old female has been ill with chronic fibrocavernous tuberculosis of the lungs for the past 20 years. She
entered the nephrology department with an uremia syndrome. A test for the presence of amyloid in kidneys was
positive. Which form of amyloid is indicated in this case?
A. * Secondary
B. Primary
C. Localized
D. Familial congenital
E. Senile
2. The dissection of a 49 year old male reveals a deformed mitral valve, which is thickened and does not completely
close. Microscopically the foci of the collagen fibers are eosinophilic and give a positive reaction on a fibrin test?
The most credible explanation is:
A. *Fibrinoid swelling
B. Fibrinoid inflammation
C. Mucoid swelling
D. Hyalinosis
E. Amyloidosis
3. A 56 year old patient with a six year history of peritonitis has died. During dissection the capsule of the liver and
the spleen was markedly thickened in places and was noted as being dense and semi-lucent. The most credible
explanation for this is:
A. *Hyalinosis
B. Necrosis
C. Mucoid swelling
D. Fibrinoid swelling
E. Amyloidosis

27
4. The dissection of a 48 year old patient who suffered with rheumatoid arthritis reveals an enlarged, dense spleen.
A spleen's section demonstrates its brown-reddish color with enlarged follicles which have the appearance of semi-
lucent, grayish-white corns. What is the name of these lesions in the spleen?
A. * Sago-like spleen.
B. Glazed spleen.
C. Sebaceous spleen.
D. Hyalinosis of spleen.
E. Porphyry spleen.
5. During the post-mortem performed on a 72 year old man there are noted some diminished areas of the spleen
with a pinkish color. Microscopic examination revealed that the follicles are diminished in volume and the walls of
the arterioles and trabeculas are thickened as well as containing homogeneous eosinophilic, PAS-positive masses.
Staining with picrofuksin dye reveals the masses to be a red color. These changes indicate the presence of:
A. *Hyalinosis
B. Amyloidosis
C. Mucoid swelling
D. Fibrinoid swelling
E. Sclerosis
6. A 52 year old male died from a heart attack. At the time of dissection a symmetric type of severe obesity
discovered. The rapture of the right ventriculum wall resulted in hemopericardium. Under epicardium an excessive
fat tissue formation discovered. A microscopy of the sample showed the excecive growth of fatty tissue
accompanied with atrophy of myocardial fibers. Which pathological process is most likely responsible for the
patient's death?
A. * Simple obesity of the heart.
B. Fatty dystrophy of myocardium.
C. Ischemic heart disease.
D. Hypertension
E. Acute myocardium infarct
7. During dissection of a 65 year old patient, who suffered from a fibrous-cavernous tuberculosis, an enlarged,
dense spleen was found. Spleen section grossly had brown-pinkish color, smooth, waxy-like surface. Which
pathological process listed below is the most credible?
A. * Sebaceous spleen.
B. Glazed spleen.
C. Porphyry spleen.
D. Sago spleen.
E. Cyanotic induration
8. A post mortem performed on a 50 year old male who died of a heart attack indicated a symmetric type of obesity
of the III degree with rupture of the walls of the right ventricle and hemopericardium. Under the epicardium surplus
deposits of fat were found. Microscopically, fatty tissue from the epicardium was dispersed in the myocardium with
an atrophy of the muscle fibers. Which process listed below is the most reliable?
A. *Obesity the heart.
8. Fatty dystrophy of myocardium.
C. Acute infarct of myocardium.
D. Ischemic heart disease.
E. Hypertension.
9. An autopsy of a 45-year-old female revealed the kidneys were dense, yellow in color and appear to have a greasy
brilliance. Which pathological process is most likely?
A. *Amyloidosis
B. Hyalinosis
C. Fatty dystrophy
D. Mucoid swelling
E. Hemochromatosis
10. Macroscopic examination of a stomach delivered from surgery, revealed a round lesion 1.5cm in diameter
which extended by the muscle layer at the antral zone of a small curvature. A semilucent dense area on the bottom
of the defect was also determined. It resembled hyaline cartilage. Which process developed in the bottom of the
stomach lesion?
A. *Localized hyalinosis
B. Amyloidosis
C. Mucoid swelling
D. Fibrinoid changes
E. Generalized hyalinosis
11. A skin biopsy of a patient with allergic vasculitis was submitted for examination. It is discovered that the vessel
walls were thickened and homogeneous. Picro-fuxin stained a tissues a yellow color. They were Shiff-positive.
Which pathological process developed in the walls of the vessels?

28
A. *Fibrinoid swelling
B. Amyloidosis
C. Mucoid swelling
D. Hyalinosis
E. Lipidosis
12. The post-mortem of a patient revealed feature of chronic kidney insufficiency. Grossly, kidneys were enlarged,
dense, wax-like, with foci of irregular depressed scars on their surface. Microscopically, the mesangeal areas were
expanded and the glomerular capillaries obstructed by Congo red stain-positive amorphous aceilular material. In
some sections the deposits took on nodular appearance. Which of the following diagnoses is most reliable?
A. *Amyloidosis of the kidneys (Amyloid nephropathy)
B. Acute glomerulonephritis
C. Chronic glomerulonephritis
D. Subacute glomerulonephritis
E. Lipoid nephrosis
13. At autopsy a 76-year-old male, with a history of peritonitis 10 years ago, is found to have thickened and dense
both liver and spleen capsules. They were translucent on a sectional view. What is the most likely pathology of the
described organs capsules?
A. *Hyalinosis.
B. Necrosis.
C. Mucoid swelling.
D. Fibrinoid swelling.
E. Amyloidosis.
14. A 55-year-old female, with a long history of rheumatoid arthritis, develops renal failure and dies. An autopsy
revealed an enlarged solid spleen. On the sectional view, its tissue had brown-reddish coloring with enlarged
follicles that look like translucent grayish-white grains. What is the most likely pathological process?
A. *Sago spleen.
B. Frosted spleen.
C. Lardaceous spleen.
D. Spleen hyalinosis.
E. Porphyry spleen.
15. A 66-year-old female, with a long history of post-traumatic osteomyelitis, is admitted to the hospital for
treatment of nephrotic syndrome. On the night pf admission she suddenly dies. Autopsy revealed dense, white
kidneys with scars in the cortical layer; they had a sebaceous glow on the cut surface. What is the most likely
kidneys pathology?
A. *Secondary amyloidosis.
B. Primary amyloidosis.
C. Idiopathic amyloidosis.
D. Chronic glomerulonephritis.
E. Chronic pyelonephritis.
16. A 55-year-old man, with a long history of a symmetrical type of severe obesity, developed acute heart
insufficiency followed with lethal outcome. An autopsy revealed right ventricle wall burst with hemopericardium
and excessive amount of fatty tissue under epicardium. Microscopically: adipose tissue from epicardium penetrates
myocardium with muscle fibers atrophy. Name the pathological process?
A. *Simple fatty heart.
B. Fatty degeneration of myocardium.
C. Ischemic disease.
D. Essential hypertension.
E. Acute myocardial infarction.

Mixed dystrophy
1. The necrotic Peyer's patches of the ileum from the patient with typhoid fever are stained in a yellow-brown color.
Which pigment impregnates the necrotic tissue?
A. * Bilirubin
B. Hemoglobin
C. Lipofuscin
D. Indol
E. Melanin
2. During post-mortem of a patient arrived from a tropical country, it is discovered that there is a hemomelanosis of
a liver, spleen and elements of the reticuloendo-thelial stroma. These changes are characteristic for which disease?
A. *Malaria
B. Dysentery
C. Diabetes mellitus
D. Exanthematic typhus

29
E. Grippe
3. A patient who suffered from cancer of the stomach died from cachexia. During the post-mortem the characteristic
alteration in the heart were found. How would this condition be termed in the heart?
A. *'Brown' atrophy
B. 'Hairy' heart
C. Solder plaque (bony heart)
D. Tiger's heart ('tabby cat')
E. Bovine heart
4. A man with insufficiency of the mitral valve complained of a cough and sputum with a brownish colouring.
Which pigment results in this color of the sputum?
A. *Hemosiderin
B. Melanin
C. Hemoglobin
D. Hemomelariin
E. Iron sulfate
5. The post-mortem of a man who presented in the hospital with a history of a snakebite reveals expressed
intravessels hemolysis. During dissection it is noted that the spleen, bone marrow and lymphatic nodes had a brown
colouring. Microscopic examination showed that the cytoplasm of macrophages got a brown pigment. Which
pigment accumulated in the tissues?
A. * Hemosiderin
B. Hematoidin
C. Hematin
D. Lipofuscin
E. Bilirubin.
6. The dissection of a patient who suffered from rheumatism and chronic rheumatic valvulitis revealed that mitral
valve leaflet was thickened with rough stony deposits. Name the pathology presented with stony appearance of the
valves?
A. * Dystrophy calcification
B. Metastatic calcification
C. Metabolic calcification.
D. Fibrinoid
E. Amyloidosis.
7. An endoscopy was performed on a patient with a chronic stomach ulcer complicated with hemorrhage. This
procedure revealed a brownish (coffee-like) liquid in the stomach. Which pigment results in the color of the stomach
contents?
A. * Hematin hydrochloride
B. Hemosiderin
C. Bilirubin
D. Ferritin
E. Porphyrin
8. A 46-year-old man has an acute stomach ulcer complicated by gastric bleeding and vomiting. Gastric masses had
a brown color and "coffee-like" appearance. Which pigment created such colouring?
A. * Hematin hydrochloride
B. Hemoglobin
C. Bilirubin
D. Hemomelanin
E. Iron sulfide
9. A 66-year-old patient complained of pain in the hands and feet joints. Physical examination revealed a
deformation and painful of the joints. Laboratory tests showed the increased level of uric acid salts in the blood and
urine. Which one of following is not being fully metabolized?
A. *Nucleoprotein.
B. Calcium.
C. Chrornoprotein.
D. Lipoprotein.
E. Potassium.
10. A man died from chronic sepsis. A post-mortem revealed an atrophy of the skeletal muscles and brown atrophy
of both myocardium and liver. Which one of the following pigments accumulated in tissues?
A. *Lipofuscin
B. Lipochrome
C. Hemosiderin
D. Hemomelanin
E. Melanin

30
11. A 58-year-old male has been ill for many years with leukemia. A post-mortem exposed a brown color in the
marrow, spleen, liver, and lymphatic nodes. The Perls' histochemical reaction was conducted. It was determined that
the reticular, the endothelial cells and histiocytes of these organs contained granules of a dark blue color. Which
pigment is responsible for the colouring?
A. * Hemosiderin
B. Bilirubin
C. Hematoidin
D. Hemomelanin
E. Hematoporphyrin
12. The post-mortem of a patient who suffered from malaria revealed jaundiced skin, sclera and mucous
membranes. Also, the spleen was enlarged and had dark-grey color. This colour of the spleen is due to the presence
of:
A. *Hemomelanin
B. Hemosiderin
C. Lipofuscin
D. Melanin
E. Hemoporphyrin
13. A 56 year old patient died from chronic cardiac insufficiency as a result of rheumatic heart-disease. A post-
mortem revealed that lungs were enlarged, dense with red-brownish coloring. What is the most likely diagnosis?
A. *Brown induration lungs
B. Acute bronchitis
C. Honey-comb lungs
D. Chronic bronchitis
E. Chronic emphysema
14. A patient with mitral valve insufficiency presents in his sputum cells, filled with brown pigment. The Perls'
reaction is positive. Name this pigment.
A. *Hemosiderin
B. Hematoidin
C. Melanin
D. Porphyrin
E. Bilirubin
15. A 66-year-old male, with a history of hernatogenic tuberculosis was examined. This revealed
hyperpigmentation of skin and mucous membranes, cachexia and insufficiency of the cardio-vascular system.
Which disease caused such changes?
A. *Addison's disease
B. Phaeochromocytoma
C. Simraond's disease
D. Gushing disease
E. Greves' disease
16. A 52-year-old male with a history of sub-acute septic endocarditis is examined by a physician. A doctor
revealed marked general pallor with icteric skin, sclera and visible mucous membranes. Blood test showed
accumulation of indirect reacting bilirubin (unconjugated bilirubin). The yellow staining of the skin, sclera and
mucous membranes indicates which one of the following?
A. *Prehepatic jaundice
B. Fatty dystrophy
C. Hemosiderosis
D. Hepatic jaundice
E. Posthepatic jaundice
17. A 62-year-old female with a history of stomach cancer with plural metastases died from a cachexia. Select the
characteristic changes of the heart expected to be revealed on dissection.
A. * Brown atrophy of myocardium.
B. Amyloid cardiomegaly.
C. Dilatation cardiomyopathy.
D. Hypertrophy cardiomyopathy.
E. "Tabby cat" ("Tiger's heart").
18. A patient developed a cyst in the cerebrum following a hemorrhagic stroke. Two years later the patient died
from pneumonia due to a complication of influenza. During examination of the brain cyst it is noted that the walls
have a rusty tint. Perls' reaction is positive. Name the process occurring in the wall of the cyst?
A. * Localized hemosiderosis
B. General hemosiderosis
C. Local hemomelanosis
D. Infiltration of bilirubin
E. Primary haemochromatosis

31
19. A patient with a long history of tuberculosis was examined at the hospital. Physical examination revealed a
grayish-brown skin color, lowered arterial pressure, hypodynamia and a decline of the level of 17-oxycorticosteroids
in the urine and blood plasma. A problem with the metabolism of which pigment is indicated by the clinical signs of
this patient?
A. *Melanin
B. Bilirubin
C. Lipofuscin
D. Lipochrome
E. Hemosiderin
20. A fragment of skin (1x2 centimeters) delivered for histological research. Grossly a small (0,5 cm in diameter)
slightly elevated brown lesion, sharply demarcated from the surrounding normal skin, was recognized.
Microscopically, a lesion presented with nevus cells nests, rich with brown pigment. This pigment had negative
Perls' reaction. Name the pigment.
A. *Melanin
B. Hematoidin
C. Hemosiderin
D. Bilirubin
E. Hemomelanin
21. A 5 5-year-old male with a history of bronchiectasis, pneuinosclerosis and cachexia died. During the post-
mortem examination the heart was found to be diminished in size, flabby, with thinned walls. A section revealed
brownish color of the heart's tissue. Which pigment was indicated in the myocardium?
A. * Lipofuscin
B. Hemosiderin
C. Hematoidin
D. Melanin
E. Lipochrome
22. A post-mortem of a 44-year-old patient with a history of mitral stenosis reveals dense lungs that are a brown
color. Which pathological process is most likely in the lungs?
A. *Hemosiderosis
B. Hemochromatosis
C. Icterus
D. Hemomelanosis
E. Lipofuscinosis
23. A post-mortem was performed on a 55-year-old male, who over last eight years suffered from chronic form of
malaria. At the dissection both grey matter of the cerebrum and a spleen had the ash-grey color. Which pigment is
responsible for this discoloration?
A. *Hemomelanin
B. Lipofuscin
C. Hematoporphyrin
D. Melanin
E. Hemosiderin
24. A 62-year-old male who has been ill with diabetes mellitus for 15 years died from a cerebral hemorrhage. Post-
mortem revealed kidneys diminished in size with a fine-grained surface. The epithelium of the canaliculi of distal
nephron's segment was high, with a light foamy cytoplasm. The Best's carmine staining demonstrated a bright red
coloring of the cytoplasm's accumulations. These changes in the epithelium resulted from the accumulation of:
A. *Glycogen
B. Lipids
C. Hyaline
D. Proteins
E. Amyloid
25. At autopsy 68-year-old male is found to have cancer of the esophagus, accompanied with cachexia. Grossly,
fatty tissue disappeared, both a liver and a heart were atrophic. Microscopy revealed brown-yellowish corn-like
deposited next to nuclei of myocardial cells. These accumulations had negative Perls' reaction. Name the material of
accumulations.
A. *Lipofuscin
B. Melanin
C. Hemosiderin
D. Ferritin
E. Hemomelanin
26. A 55-year-old patient with a bilateral adrenal glands lesions presented with dark brown colouring of the skin.
During histochemical examination of the skin the Perls' reaction was negative. Which pigment is responsible for this
discoloration of the skin?
A. *Melanin

32
B. Hemosiderin
C. Porphyrin
D. Lipofuscin
E. Biliverdin
27. A 58-year-old female with a long history of chronic dysentery died. At autopsy, the stroma and parenchyma of
the myocardium, kidneys, the mucous membrane of the stomach, and the connective tissue of lungs revealed violet
color amorphous masses, which had positive K.QSS' reaction. Which pathological process developed in the patient's
organs?
A. *Metastatic calcification
B. Dystrophy calcification
C. Metabolic calcification
D. Amyloidosis
E. Hyalinosis
28. A 45-year-old male, with a long histoiy of rheumatism and mitral valve insufficiency, develops a chronic cough
with rusty expectoration. What pigment colored sputum?
A. *Hemosiderin.
B. Melanin.
С Hemoglobin.
D. Malarial pigment.
E. Iron sulfide.
29. A 67-year-old male, with a long history of mitral valve's insufficiency, has been experiencing a cough with
red-brownish coloring of a sputum. Cells with brown pigmentation and positive Perls' test were detected in the
sputum. Which pigment responsible for the septum coloring?
A. *Hemosiderin.
B. Hematoidin.
C. Melanin.
D. Porphyrin.
E. Bilirubin.
30. A 38-year-old female with chronic stomach ulcer complicated with bleeding examined endoscopically. Stomach
masses had coffee ground coloring. Which pigment responsible for this coloring?
A. *Hematin chloride. В Hemosiderin.
C. Bilirubin.
D. Ferritin.
E. Porphyrin.
31. A 47-year-old male, with a history of secondary syphilis, has noticed foci of the skin depigmentation at his neck.
Name the pathological process of the skin.
A. *Leukoderma.
B. Metaplasia.
C. Leukoplakia.
D. Dysplasia.
E. Parakeratosis.
32. a 38-year old female is found to have bilateral adrenal adenopathy resulted in brownish coloring of her skin.
Perls' test of her skin's biopsy was negative. What pigment altered the color of the skin?
A. *Melanin.
B. Hemosiderin.
C. Porphyrin.
D. Lipofuscin.
E. Biliverdin.
33. A 70-year-old man has noticed recent swelling of joints of his hands and feet. Joints are painful and stiff.
Laboratory tests revealed an increased level of urates in blood and urine. What is the most likely substance caused
described pathology?
A. *Nucleoproteins.
B. Calcium.
C. Chrornoproteids.
D. Lipoproteins.
E. Potassium.
34. A 70 year-old male with a history of chronic shigellosis died. At post-mortem tissue samples were collected for
histopoly. Microscopic investigation of hematoxylyn and eosin slides revealed amorphous violet deposits in stroma
of the heart, kidneys, lungs and stomach mucosa. Koss' reaction was positive. What is the most likely pathological
developed?
A. *Metastatic calcification.
B. Dystrophic calcification.
C. Metabolic calcification.

33
D. Amyloidosis.
E. Hyalinosis.
Necrosis. Postmortem changes.
1. A male had a surgery due to "acute abdomen". During the operation it was noted that the peritoneum was dull and
in the lumen of the rnesenteric artery superior a thrombus was detected. About 80 centimeters of the ileac intestine
had a black colouring. Which process was diagnosed in the intestine?
A. *Gangrene
B. Decubitus ulcer
C. White infarct
D. White infarct with a hemorrhagic crown
E. Coagulative necrosis
2. A patient with diabetes rnellitus presents to his physician with an acute pain in the right foot. At inspection the
toe of foot had a black colour, the tissues of foot were edematous, with bed smell. Which form of necrosis was likely
to be diagnosed?
A. * Moist (wet) gangrene
B. Decubitus ulcer
C. Sequester
D. Dry gangrene
E. Infarct
3. A 63-year-old male died of an endemic typhus. During the post-mortem it was revealed that the muscles of the
abdominal wail and legs were dense with whitish-yellowish colouring. They resemble a candle. Name the
pathological process?
A. *Waxy necrosis (Zenker's necrosis)
B. Apoptosis
C. Fibrinoid necrosis
D. Colliquative necrosis
E. Caseous necrosis
4. A 72-year-old-male had an infarct of the dextral hemisphere of the brain. One year later a computer tomography
of the right hemisphere of the brain reveals a cavity with smooth walls and filled with liquid. Which pathological
process is he most likely to have?
A. * Post-infarct cyst
B. Hydrocephalus
C. Grey softening of a brain
D. Infarct of a brain
E. Hematoma
5. A post-mortem revealed a thrombus in the left artery mesencephalicae and a large locus of grey softening in the
tissues of the left hemisphere of a brain. Which pathological process is most likely to be present in the brain?
A. * Ischemic infarct
B. Coagulative necrosis
C. Abscess
D. Moist gangrene
E. Sequestrum
6. At a post-mortem of the 46-year-old male a large yellow - grey lesion was found in the left ventricle of the heart.
A fresh thrombus was found in the coronary artery. What disease is he most likely to have?
A. *Infarct of the myocardium
B. Cardiosclerosis
C. Myocarditis
D. Amyloidosis
E. Cardiomyopathy
7. At autopsy a 60-year-oid male is found to have ischemic heart disease and atherosclerosis of the coronary arteries
of heart. A section of the myocardium showed a white-yellowish focus, surrounded by the zone of hemorrhages in
the apex, anterior and lateral walls areas of a left ventricle. Which is the most likely diagnosis?
A. *Infarct of the myocardium
B. Post-infarction cardiosclerosis
C. Diffuse cardiosclerosis
D. Myocarditis
E. Fatty dystrophy of the myocardium
8. After a long staying in the bed a patient with circulatory deficiency got the skin and soft tissue darening above
the sacrums. These tissues became swallowed. Sloughing off the epidermis in this area resulted in ulceration. Which
complication is most likely?
A. *Decubitus ulcer
B. Dry gangrene
C. Phlegmon

34
D. Infarct
E. Abscess
9. Histological investigation of liver's biopsy revealed that some cells burn on small pieces with separate organellas
and nuclei fragments surrounded by a membrane. The inflammatory reaction was missing. Select pathological
process, the described changes are characteristic for:
A. *Apoptosis
B. Necrosis
C. Karyorrhexis
D. Plasmolysis
E. Plasmorrhexis
10. A male who had a long history of the intermittent claudication demonstrates the tissue of the foot fingers as
being dry with a black colour, resembling a mummy. On small interval from this place the dichromatic line (red
colour, is next to unchanged tissues, and white - yellow colour close to a tered tissues). Which type of a necrosis
occurred?
A. *Gangrene.
B. Infarct.
C. Sequester.
D. Decubitus ulcer.
E. Maceration.
11. A 62-year-old male got a surgery due to the inguinal hernia. Macroscopic examination reveals that the wall of
the intestine was a cyanotic, inflated, swallowed and coated with threads of a fibrin. Peristalsis was not heard.
Which pathological process occurred in the wall of the intestine?
A. * Moist gangrene.
B. Dry gangrene.
C. Coagulate necrosis
D. Colliquative necrosis
E. Decubitus ulcer
12. A postmortem of a man, who died from typhoid revealed muscles on the abdominal wall and legs were dense,
fragile, whitish-yellowish colour, resembling a candle. Which term best characterizes the muscles changes?
A. *Zenker's necrosis
B. Fibrinoid necrosis
C. Caseous necrosis
D. Colliquative necrosis
E. Apoptosis
13. A 48-year-old male, who had a history of hypertension for 12 years, present acute disturbance of the cerebral
circulation. He developed a headache and alteration of the motion in the right extremity. Following right-handed
hemiplegia resulted in fatal outcome. A postmortem revealed a systemic hyalinosis of the small arteries, thrombosis
in the left arteria cerebri media. In the left parietal-temporal area a lesion was found, which is called:
A. *Ischemic infarct
B. Hemorrhage
C. Abscess of a brain
D. Hemorrhagic infarct
E. Edema of the brain
14. A 45-year-old male suddenly died with the following findings revealed during the postmortem. In the back wall
of the left ventricle of the heart a myocardial infarction was found. Which of the following microscopic changes in
the myocardio-cytes can be seen in the locus of an infarct?
A. *Karyolysis
B. Fatty dystrophy
C. Carbohydrate dystrophy
D. Calcification
E. Protein dystrophy
15. A postmortem of on a previously ill 48-year-old patient found an obturation of the lumen of the middle cerebral
artery due to a thrombus. In the parietal-temporal area of the left hemisphere of the brain a locus of grey colour
tissue with soft texture is found. Which tern best characterizes the brain tissue changes?
A. *Infarct
B. Sequester
C. Gangrene
D. Caseous necrosis
E. Fibrinoid necrosis
16. A postmortem on the upper lobe of the right lung reveals the large triangle-like locus of the dark red dense
tissue. Histological examination indicates necrosis of the walls of the alveolus's and the lumens filled with
erythrocytes. Which is the most likely associated finding?
A. *Hemorrhagic infarct

35
B. Carnification
C. Lung's gangrene
D. Hemorrhage
E. Atelectasis
17. A postmortem on an elderly male with atherosclerosis reveals a thrombus in a branch of the internal carotid
artery as well as a grey locus of a moist softening of the brain's tissue. Which pathological process was found in the
brain?
A. *Ischemic infarct
B. Hemorrhagic infiltration
C. Hematoma
D. Encephalitis
E. Tumor of a brain
18. A surgery on a patient, with a history of syphilis revealed a locus of flabby tissue. Grossly, this locus was
yellowish, dry, structures and gummy. The most likely diagnoses is:
A. *Caseous necrosis
B. Infarct
C. Waxy necrosis
D. Fibrinoid a necrosis
E. Steatonecrosis
19. The investigation of the liver's biopsy revealed that some separately arranged cells burn on small-sized pieces
surrounded by a membrane. In some of them there were organelles, other had the fragments of dissolved nuclei. The
inflammatory reaction around these cells missed. Name these changes:
A. *Apoptosis
B. Atrophy
C. Necrosis
D. Hypoplasia
E. Dystrophy
20. An ill elderly patient with a atherosclerosis, develops pain in the left foot. Grossly was found the foot
enlargement, its tissues were black, flabby and macerated. The demarcation zone was not expressed. Which term
best characterized the foot tissues changes?
A. * Moist (wet) gangrene.
B. Mummification.
C. Coagulate necrosis.
D. Dry gangrene.
E. Sequestrum.
21. A 62-year-old female with atherosclerosis was admitted to the hospitalized. At surgery gross examination
revealed purulent peritonitis. During the operation a thrombus in the mesenteric arterias was found. Which was the
most likely cause of the peritonitis?
A. *Hemorrhagic infarct.
B. Angiospastic ischemia
C. Ischemic infarct
D. Stasis
E. Compressive ischemia
22. The examination of a child, who had a history of measles, revealed reddish-black, uneven, swollen, slightly
fluctuated lesions of cheaks and perineum area. Name the complication of measles?
A. * Moist gangrene (noma)
B. Dry gangrene
C. Gas gangrene
D. Decubitus ulcer
E. Trophic ulcer
23. A postmortem was performed on a female who died due to the cystadenocarci-noma metastases. The
postmortem revealed large segments of a necrosis of the skin and soft tissues within cubitus area. Name the form of
the necrosis.
A. *Decubitus ulcer
B. Infarct
C. Sequester
D. Caseous necrosis
E. Zenker's necrosis
24. A 58-year-old female with the history of atherosclerosis dies suddenly due to acute heart failure. Gross
inspection of the left ventricle of the heart revealed a whitish-yellowish 6x5 cm, dense lesion with uneven
boundaries and hemorrhagic zone next to it. Which is most likely diagnose?
A. *Infarct of the myocardium
B. Postinfarction fibrosis

36
C. Healed infarct
D. Myocarditis
E. Ischemic cardiomyopathy
25. An ultrastructural examination of a salivary gland revealed within cells pieces of the nuclei surrounded by a
membrane. Also condensate fragments of nuclear material and separate organelles were found. An inflammatory
reaction around these cells was missing. Which term most correctly defines these alteration?
A. *Apoptosis
B. Karyorrhexis
C. Coagulation necrosis
D. Karyopyknosis
E. Karyolysis
26. A patient with tuberculosis has a kidney biopsy performed. Histological investigation revealed the caseous
necrosis of the tissue accompanied by disorderly arranged fine grains of a chromatin. Which term most correctly
defines describe lesion?
A. * Karyorrhexis
B. Karyolysis
C. Karyopyknosis
D. Mitotic activity of nuclei
E. Apoptosis
27. A postmortem of a 48-year-old male who had a history of typhoid fever revealed that the rectus abdominis at
the wall was dense, a whitish colour, and resembled a candle. Which is the most likely diagnosis?
A. * Waxy necrosis
B. Fibrinoid necrosis
C. Colliquative necrosis
D. Caseous necrosis
E. Apoptosis
28. A 44 year old ill patient died due to the severe chronic heart failure. Pathologist diagnosed rheumatic
granulomatous myocarditis. Microscopic evaluation of the myocardium indicated the presence of granulomas which
consisted of macrophages with hyperchromatic nuclei and clear cytoplasm. Also necrosis was seen in the center of a
lesion. Which is the most likely type of necrosis in the center of the lesion?
A. * Fibrinoid necrosis.
B. Waxy necrosis.
C. Caseous necrosis.
D. Colliquative necrosis.
E. Fatty dystrophy.
29. A 57-year-old patient has a long history of the type II diabetes mellitus. Physical examination revealed the
alteration of the right foot tissues. They are dense, black with precise boundaries from normal tissues. Which term
most correctly describe the lesion?
A. *Dry gangrene.
B. Wet (moist) gangrene.
C. Gas gangrene.
D. Decubitus.
E. Trophic ulcer.
30. A postmortem examination of a dead body revealed a cloudy corneas, dry skin with yellowish - brown lesion.
Which term most correctly identifies describes a post-mortem alterations?
A. *Cadaver desiccation
B. Clotting of blood
C. Livor mortis
D. Rigor mortis
E. Algor mortis
31. The postmortem of a 48-year-old male reveals in the right temporal lobe of the brain a large grey lesion with a
softening, porridge-like texture. The basal arteries of the brain had numerous white - yellow thickenings of an intima
which significantly decreased lumen. Which is the most likely diagnosis?
A. *Ischemic infarct
B. Abscess of a brain
C. Hematoma
D. Hemorrhagic infarct
E. Edema of the brain
32. At post-mortem, a 60-years-old man, with a history of-typhoid fever, is found to have rectus muscles of the
anterior abdominal wall dense, whitish, and look like a stearic candle. What is the most likely diagnose?
A. *Zenker's necrosis.
B. Fibrinoid necrosis.
C. Colliquative necrosis.

37
D. Caseous necrosis.
E. Apoptosis.
33. A 65-year-old female, with a long history of diabetes mellitus, presented her black, edematous and painful
thumb of the right foot. Gross inspection revealed a focal epidermal detachment and malodorous discharge. What is
the most likely clinico-pathologic form of necrosis?
A. *Moist (wet) gangrene.
B. Decubitus ulcer.
C. Sequester.
D. Dry gangrene.
E. Infarction.
34. A 5-year-old boy with measles presents to his pediatrician with necrotic changes of his cheeks. Gross inspection
revealed that the cheeks soft tissues were edematous with reddish black fluctuated indistinctly outlined foci. What
is the most likely complication of a measles?
A. *Moist gangrene.
B. Dry gangrene. С Gas gangrene.
D. Decubitus ulcer.
E. Trophic ulcer.
35.A physical examination of 67-year old lady, with a history of femoral bone fracture, revealed a sequester
formation accompanied with chronic inflammation of a bone marrow and adjacent tissues. What is the most likely
disease caused such lesions?
A. *Osteomyelitis.
B. Reticulosarcoma.
C. Multiple myeloma.
D. Osteoclastoma.
E. Periostitis.
36, An elderly woman with a history of a stroke one year ago complains of a left limbs immobility. A computer
tomography examination revealed a cavity filled with liquor, at right hemisphere of her brain. What is the most
likely diagnose?
A. *Postinfarction cyst.
B. Hydrocephaly.
C. Grey encephalomalacia.
D. Cerebral infarction.
E. Hematoma.
Blood and lymph circulation disorders.
1. The postmortem of a 48-year-old male, with a long history of a chronic heart failure, revealed an enlarged liver.
Grossly, a liver had a motley pattern. Macro-scopically, the sectional view looked like a nutmeg on incision. Which
term most correctly defines these alteration?
A. * General venous plethora
B. General arterial plethora
C. Anemia
D. Hemorrhage
E. Bleeding
2. A post-mortem of a 43-year-old female revealed multiple, hemorrhagic infarcts in lungs. Some lungs' vessels had
reddish-brown dense masses within lumens, which were not attached to the vessel walls. Varicose phlebectasia of
the legs with thromboses of some veins were also determined. Which pathological process occurred in this case?
A. * Thromboembolism of pulmonary artery.
B. Fat embolism of pulmonary' artery.
C. Tissue embolism of pulmonary artery.
D. Foreign bodies embolism of pulmonary artery.
E. Gas embolism.
3. A male patient died after a abdominal surgery. During the postmortem numerous thrombuses were found in the
veins of the pelvis. Thromboembolic syndrome was diagnosed. Where it is necessary to search for
thromboembolies?
A. *Lungs artery
B. Portal vein
C. Left ventricle of heart
D. Brain
E. Veins of the lower extremity
4. The histological investigation of a liver detects venous plethora of center lobules, dystrophy and atrophy of
hepatocytes in the venous plethora's area, fatty a dystrophy of hepatocytes on the periphery of a lobe. The
replacement fibrosis in places of an atrophy of the hepatocytes was also revealed. Which pathological process does
mis refer to?
A. * 'Nutmeg' liver with precirrhotic phenomena.

38
B. Biliary hepatic cirrhosis. С Fatty hepatosis.
D. Hepatitis.
E. Toxic dystrophy of a liver.
5. A 35-year-old patient complains of repeated vomiting, diarrhea, decreased arterial pressure and tachycardia. He
supposes this condition related to the food poisoning. Laboratory tests detected Salmonella's infection.
Hematological examination revealed an increased munber of erythrocytes per unit volume. Which circulatory
disturbance took place to create this pathology?
A. *Clotting of a blood.
B. Hemolysis of erythrocytes and compensatory induction of ahemogenesis
C. General arterial plethora
D. Polycythemia
E. Hyperchromatic anemia
6. A male with a history of myocardial infarct died of cardiovascular failure. A post-mortem revealed a replacement
fibrosis, hypertrophy of the myocardium and dilatation of the cavities, especially the right ventricle. The liver was
enlarged. Its surface was smooth. Grossly, a motley pattern with dark red dots on the grey a background was
revealed. Histologically, central zones of the lobules were hy-peremic. On the periphery, around of periportal tracts
hepatocytes demonstrated fatty dystrophy. Name these liver changes.
A. *"Nutmeg" liver (chronic venous plethora)
B. Pseudo 'nut-meg' liver
C. Amyloidosis
D. Cirrhosis of a liver
E. Steatosis of a liver
7. A male patient involved in the traffic accident received a wound in the neck due to broken glass. The bleeding
was small, but a short time after the accident he died of acute dyspnea. A post-mortem of the heart revealed bubbles
excretion when pericardium filled with water. Indicate which pathological process took place:
A. *Air embolism
B. Gas embolism
C. Fat embolism
D. Thromboembolism
E. Foreign bodies embolism
8. A 52-year-old male with long history of rheumatic heart disease died of chronic heart failure. A post-mortem
revealed brown colour, enlarged dense lungs. Name the changes in lungs.
A. *Brown induration of lungs
B. Acute bronchitis
C. Horny-comb lungs
D. Chronic bronchitis
E. Chronic emphysema
9. A patient, who had fast elimination of 10 liters of an ascitic liquid from abdomen, suddenly lost his
consciousness. What was the cause of this phenomenon?
A. * Anemia of the brain
B. Thrombosis of the cerebral arteries
C. Brain hemorrhage
D. Arterial hyperemia
E. Thrombosis of the cerebral veins
10. A young male with a history of rheumatic heart disease since childhood, gradually develops mitral stenosis,
accompanied with episodes of heart failure. He presents to a hospital complaining of coughing with a reddish-brown
sputum. Name probable changes at patient's lungs.
A. *Brown induration of lungs
B. Emphysema of lungs
C. Atelectasis of lungs
D. Pneumosclerosis
E. Bronchiectasis
11. At autopsy an elderly female is found to have a blood clot in the femoral artery, which grossly had a striped
pattern. Microscopy revealed a congestions of fibrin fibers and broken down red and white cells. Which is the most
likely type of thrombus ?
A. * Mixed thrombus
B. Postmortem convolutions of a blood
C. Thromboemboli
D. Hyaline thrombus
E. Red thrombus
12. A patient has a purulent otitis and thrombosis of a sigmoid sinus of a dura mater. Name the complication if
thrombus breaks down and fragments circulate to distal vessels.
A. *Thromboembolism of branches of pulmonary artery

39
B. Thromboembolism of vessels of a brain with development of a grey softening of the brain
C. Thromboembolism of vessels of a retina of an eye and development of blindness
D. Thromboembolism of coronary arteries
E. Local anemia
13. A 43-year-old male, with a history of traffic accident and poly trauma, including multiple bone fractures,
presented to the hospital. On the second day after the trauma, he complained of a pain in the right half of the chest,
heavy breathing. At night, he died due to progressive heart and respiratory failure. Microscopic investigation
revealed Sudan-positive orange drops in pulmonary and cerebral vessels that completely occlude the lumens of
microcirculatory vessels. What is the most likely complication led to a patient's death?
A. *Fat embolism.
B. Gaseous embolism.
C. Medicamentous embolism.
D. Microbial embolism.
E. Thromboembolism.
14. A 62-year-old patient had a surgery due to acute appendicitis. She had been placed in a bed for 5 days. After she
decided to get out of bed she experienced a shortage of air, her face became cyanotic and she lost her consciousness.
After unsuccessful resuscitation the patient died. A post-mortem revealed a thromboem-bolism of the pulmonary
artery. Which of the following is most likely source of mromboembolism?
A. Thrombosis of veins of the lower extremity
B. Thrombosis of a portal vein
C. Thrombosis of mesenteric arteries
D. Thrombosis in a left ventricle of heart
E. Ball-shaped thrombus of an auricle
15. A male with a fracture of the shoulder bone carried on overlapped plaster bandage. Suddenly, an arm and the
visible part of the forearm became cyanotic, cold, and edematous. Which of the following is most likely happened?
A. *Local venous plethora
B. Local arterial plethora
C. Local anemia
D. Stasis
E. Thrombosis
16. An elderly female develops acute disorder of the cerebral blood circulation, followed with coma, resulted in
fatal outcome. A post-mortem revealed in the right hemisphere of the brain a large cavity, filled with blood. Which
pathological process took place in the brain?
A. *Hematoma
B. Hemorrhagic infiltration
C. Infarct of the brain
D. Diapedesis
E. Edema of the brain
17. A male suddenly died after an open fracture of the clavicle. A post-mortem revealed in the right ventricle of the
heart and pulmonary arteries the foamy blood. Which one was the cause of death?
A. *Air embolism
B. Bacterial embolism
C. Hemorrhage
D. Tissue embolism
E. Fat embolism
18. A male with a history of the decompression sickness developed symptoms of acute cerebral circulation disorder
and died soon. A post-mortem revealed in the left hemisphere of the brain the locus of a grey softening of the brain,
which was 6x5x3 cm. Which one is most likely to cause the death of the patient?
A. *Gas embolism.
B. Fat embolism.
C. Thrombosis.
D. Thromboembolism.
E. Atherosclerosis of the vessels.
19. A post-mortem of an elderly man with a long history of the ischemic heart disease and heart failure revealed a
nutmeg liver, brown induration of lungs, cyanotic induration of kidney and spleen. Indicate, what type of the blood
circulation disorder is most likely?
A. * Chronic general venous plethora.
B. Arterial hyperemia.
C. Acute general venous plethora.
D. Acute anemia.
E. Chronic anemia.
20. A 50-year-old male with a myocardial infarction died from the heart failure. A post-mortem revealed the edema
of the lungs and petechial hemorrhages at serous and mucus membranes. Microscopic examination indicated marked

40
dystrophic alterations of the nephron's epithelium at proximal canaliculi of kidneys. Also, the centrolobular
hemorrhage and necrotic zones were found in the liver. Name the type of the blood circulation disorder.
A. * Acute general venous plethora.
B. Arterial hyperemia.
C. Chronic general venous plethora.
D. Acute anemia.
E. Chronic anemia.
21. A neonate died of intoxication. A microscopic examination of an umbilical vein revealed a diffuse inflammatory
infiltration of the vessel's wall. Also, its lumen was filled with thrombus, rich with leucocytes and bacterial colonies.
Karyorrhexis was detected in many leucocytes. Which is most likely outcome of a thrombus in that case?
A. * Septic autolysis.
B. Aseptic autolysis.
C. Organization and vascularization of the thrombus.
D. Thromboembolism.
E. Calcification of a thrombus.
22. A 68-year-old female is admitted to the hospital for treatment of deep vein thromboses. The next day she
suddenly died. At autopsy a large plug of laminated blood clot is found to occlude the main pulmonary artery.
Which is likely to be found in her lungs?
A. * Thromboembolism
B. Thrombosis
C. Tissue embolism
D. Foreign bodies embolism
E. Fat embolism
23. A 77-years-old female with unstable angina presents to physician with slowly increasing heart failure
symptoms. On the night of admission to the hospital she dies. A post-mortem examination revealed the enlarged
liver with dense texture and the rounded edges. Sectional view had a nutmeg pattern with dark red dots on the
yellowish background. Which pathological process resulted in liver's alteration?
A. * Chronic venous plethora
B. Acute venous plethora
C. Arterial plethora
D. Arterial anemia
E. Chronic hemorrhage
24. A post-mortem of a 53-year-old male with a long history of hypertension revealed the cavity in the occipital
lobe of the brain. It measured 2x1 cm, had a brownish smooth walls and filled with transparent liquid. Which is the
most likely diagnosis?
A. * Cyst after hemorrhages
B. Softening of the brain
C. Abscess of a brain
D. Developmental defect of a brain
E. Cyst after softening of the brain
25. A 65-years-old patient, who presented with a 10 years history of ischemic heart disease, died due to heart failure.
An autopsy revealed the cyanotic induration of both spleen and kidneys, brown induration of lungs and "nutmeg"
liver. Which is the most likely type of the blood circulation disorder, resulted in such changes of internal organs?
A. * General chronic venous hyperemia
B. Genera! acute venous hyperemia
C. General arterial hyperemia after an anemia
D. Arterial ischemia as a result of reallocating a blood
E. Local chronic venous hyperemia
26. A 63-year-old patient with long history of ischemic heart disease and repeated myocardial infarction died due to
progressive cardiovascular failure. A postmortem revealed an enlarged, dense spleen with the dark red colour of the
sectional view. At the microscopic examination of spleen the pulp's sclerosis and atrophy of the follicles were found.
Which term most correctly defines theses changes?
A. *Cyanotic induration of a spleen
B. Sago spleen
C. Waxy spleen
D. Porphyry spleen
E. Septic spleen
27. A young male fractures his pelvis and femur in a motor vehicle accident. On the third day he dies after
respiratory distress and cerebral dysfunction. A histological examination revealed Sudan-positive orange globules
scattered within cerebral cortex, kidneys and lung microcirculatory vessels. Which is most likely diagnoses?
A. * Fat embolism
B. Gas embolism
C. Tissue embolism

41
D. Microbial embolism
E. Thromboembolism
28. A cosmonaut died due to the air-tightless of the space-shuttle cabin. Microscopic examination of the vessels of
the internal organs revealed multiple bubbles. Liver cells developed fatty dystrophy. In the brain and spinal cord
multiple small, grey, soft lesions were discovered. Indicate the most probable cause of such alterations.
A. *Gas embolism
B. Air embolism
C. Fat embolism
D. Thromboembolism
E. Tissue embolism
29. A patient with a long history of rheumatic disease died of cardiopulmonary failure. A post-mortem revealed a
stenosis of the mitral orifice as well as a brown induration of lungs. Which term most correctly defines this blood
circulation disorder?
A. *Chronic left ventricular failure.
B. Chronic right ventricular failure.
C. Acute left ventricular failure.
D. Acute right ventricular failure.
E. Portal hypertension.
30. A post-mortem of a 59-year-old patient with long history of idiopathic hypertension revealed in brain a cavity
with rusty colour walls. Which process preceded these changes?
A. *Hematoma
B. Diapedesis
C. Ischemic infarct
D. Plasmorrhagia
E. Abscess
31. A 65-year-old- patient presented with liver cirrhosis. The removal from his abdominal cavity of 10 liters of
ascitic liquid resulted a collapse. Grossly his peritoneum was hyperemic. Define the type of hyperemia in
peritoneum.
A. * Post-anemic hyperemia
B. Collateral hyperemia
C. Complementary hyperemia
D. Inflammatory hyperemia
E. Caused by arteriovenous fistula
32. A post-mortem of a 60-year-old female with a history of secondary hypertension revealed a cavity at the right
hemisphere of the brain 4x2,5 cm, filled with red clots of blood and softened brain tissue. What term best
characterized the brain changes?
A. *Hematoma.
B. Hemorrhagic impregnation.
C. Ischemic infarct.
D. Cyst.
E. Abscess.
33. An inexperienced scuba diver ascends from a depth of 55 meters to the surface in about 5 minutes. Shortly after
surfacing he complains of severe muscle contraction and intense abdominal pain. What is most likely diagnose?
A. *Gas embolism
B. Air embolism
C. Fat embolism
D. Thromboembolism
E. Tissue embolism
34. A 78-year-old male, with a history of stroke 2 years ago, died of pneumonia as a complication of severe
influenza. At post-mortem, gross investigation of his brain revealed a cerebral cyst with rusty color of its walls.
Perls' test was positive. Name the process reveled within a cystic wall.
A. *Local hemosiderosis.
B. Common hemosiderosis.
C. Local hemomelanosis.
D. Infiltration of bilirubin.
E. Primary hemochromatosis.
35. A 54-years old male with a history of ischemic heart disease presents at hospital with recurrent myocardial
infarction. Few days later, he died due to cardiac failure. Post-mortem revealed an enlarged solid spleen of dark
cherry color on the cut surface. Microscopically, pulp sclerosis and follicles atrophy were found out. What is the
most likely term to define spleen's alterations??
A. *Cyanotic induration of spleen.
B. Sago spleen.
C. Lardaceous spleen.

42
D. Porphyry spleen.
E. Septic splenitis.
36. A post-mortem of 77-year-old male, with a long history of cerebral atherosclerosis, revealed an atrophy of his
cerebral cortex. What was the most likely cause of cortex alterations?
A. *Insufficiency of blood supply.
B. Pressure.
C. Physical and chemical factors.
D. Neurotic.
E. Dysfunctional.
37. A 56-year-old male with a history of cerebral atherosclerosis suddenly dies. At, a there is a thrombus within a
middle cerebral artery's lumen. Gross inspection revealed a grey flabby focus at the parietotemporal part of the left
hemisphere of his brain .What are the most likely alterations diagnosed in a brain tissues?
A. *Infarction.
B. Sequestrum.
C. Gangrene.
D. Caseous necrosis.
E. Fibrinoid necrosis.
38. An autopsy of a patient who died suddenly at emergency room revealed in his brain a cavity of irregular shape (5
x 3.5 cm) filled with blood clots and macerated cerebral tissue. An area of cavitary destruction had a rim of brown
discoloration. This lesion was found within the subcortex nuclei at the right hemisphere of brain. What is the most
likely definition of described pathology?
A. *Hematorna.
B. Hemorrhagic impregnation.
C. Ischemic infarction.
D. Cyst.
E. Abscess.

Inflammation.
1 A histological investigation of enlarged neck lymphatic node revealed a congestion of epithelial cells,
lymphocytes and giant Pirogov-Langhan's cells. In the center of lesion a caseous necrosis was present. Which
pattern of inflammation do the lymphatic node display?
A. * Tuberculosis
B. Rhinoscleroma
C. Sarcoidosis
D. Glanders.
E. Syphilis.
2. A 3-year-old child died of a meningococca! fever. A macroscopic study revealed swallowed meninges which had
yellow-green colouring. Which pattern of inflammation was most likely in meninges?
A. *Fibrinous-purulent inflammation
B. Serous inflammation
C. Hemorrhagic inflammation
D. Catarrhal inflammation
E. Necrotic inflammation '
3. A 37-year-old male presents with a fever, dyspnea and pain in the right part of the chest. The pleurocentesis gave
700 mis of thick yellow-greenish liquid. What pathological process was diagnosed in a pleural cavity?
A. *Empyema of a pleura
B. Bronchopneumonia
C. Serous pleuritis
D. Hemorrhagic pleuritis
E. Carcinomatosis of a pleura
4. A 25 years old male presents with a 2-day history of itching and redness in the buccal area that appeared after
shaving. Physical examination by family physician revealed vesicles filled with transparent fluid on the background
the hyperemic buccal area. What is the most likely type of fluid in the vesicles?
A. *Serous exudate.
B. Transsudate.
C. Purulent effluent.
D. Mucous exudate.
E. Hemorrhagic exudate.
5. An autopsy of a 34-years old female, with a history of croupous pneumonia revealed opaque fluid in her pleural
cavity. There was also a grey membrane on the visceral pleura. What is the most likely type of the pleura's
inflammation?
A. *Fibrinous.
B. Catarrhal.

43
C. Suppurative.
D. Granulomatous.
E. Hemorrhagic.
6. A 4-year-old girl with 3 days history of diphtheria presents to the emergency with croup symptoms. Intensive
care was unsuccessful and child died at the hospital. A post-mortem revealed thickened, edematous, dull mucous
tunic of the larynx, trachea, and bronchi covered by grayish membrane easily separated from tissues underneath.
What is the most likely type of inflammation?
A. *Fibrinous.
B. Serous.
C. Suppurative.
D. Mixed.
E. Catarrhal.
7.At post-mortem a 5-year-old boy is found to have a severe tracheobronchitis complicated with asphyxia. Grossly,
a thick, gray, leathery membrane covered the mucous tunic of trachea and bronchi. The membrane loosely attached
to underlying tissues and easily removed with forceps. The lumen of the segmental bronchi was blocked with gray
masses of tissue debris, which microscopically consisted with necrotic tissues, neutrophils, fibrin and bacteria. What
is the most likely type of inflammation?
A. *Croupous.
B. Catarrhal.
C. Diphtheritic.
D. Purulent.
E. Serous
8. An autopsy of a 50-year-old male, who died of dysentery, revealed the hypere-mic mucosa of the colon, coated
with grey membranes, which can be removed from tissues underneath with some effort. Which type of an
inflammation described in the intestine?
A. *Diphtheroid inflammation
B. Croupous inflammation
C. Hemorrhagic inflammation
D. Serous inflammation
E. Catarrhal inflammation
9. A 30-year-old male has a neck lymph node removed for examination. Histologi-cal investigation revealed
granulomas, which consisted of epithelioid, lymphoid and rnultinuclear Langhans giant cells. Granulomas' centers
were presented with necrosis. What causative agent needs to be revealed in the necrosis zone for support of the
diagnosis of tuberculosis?
A. *Mycobacterium tuberculosis
B. Treponema pallidum
C. Staphylococcus
D. Frisch bacillus
E. Salmonella
10. An autopsy of a 53-year-old male, with a history of crupous pneumonia, revealed in his dextral pleural cavity
900 ml of cloudy, grey-yellowish colored liquid. Pleural membranes were found to be dim and plethoric. Name the
clinical -morphological form of the inflammation in the pleural cavity?
A. * Empyema.
B. Fibrinous inflammation.
C. Phlegmon.
D. Chronic abscess.
E. Acute abscess.
11. An autopsy of a 53-year-old male, who died from chronic renal failure, revealed the colon's mucosa, covered
with grey — yellow membranes, densely coherent to tissues underneath. The removal of the membranes resulted in
ulcers formation. Name a type of an inflammation?
A. *Diphtheroid
B. Serous
C. Catarrhal
D. Croupous
E. Purulent
12. A 65-year-old female with a long history of chronic glomerulonephritis died from chronic renal failure. A post-
mortem revealed on the surface of the epicardium and pericardium grey-whitish villous membranes. Which
pathological process took place in the pericardium?
A. *Fibrinous inflammation
B. Organization
C. Proliferative inflammation
D. Hemorrhagic inflammation
E. Arterial plethora

44
13. An elderly man presented to his physician with the complicated nasal breathing. The histological investigation of
a biopsy from his nose septum revealed granulo-matous inflammation in the mucosa with groups of large vacuolated
histiocytes containing bacteria (Mikulicz's cells). What is the most likely diagnosis?
A. *Rhinoscleroma
B. Syphilis
C. Tuberculosis
D. Malleus
E. Lepra
14. An elderly man, with a history of an operation, presented with a sepsis, developed on a background of a
decreased immune reactivity of an organism. A disease resulted in fatal outcome. Microscopical investigation of the
abdominal wall revealed the diffuse segmentonuclear leukocyte's infiltration of intermuscular spaces, edema of a
tissue and lysis of muscle fibers. Define the type of inflammation?
A. * Phlegmon.
B. Diphtheroid inflammation.
C. Abscess.
D. Necrosis.
E. Catarrhal inflammation.
15. A 64-years-old male died of rheumatism. An autopsy revealed epicardium covered with villiferous grey color
membranes, which were easily separated from tissues underneath. The separation of membranes presented an
edematous, hyperemic surface of epicardium. What type of an inflammation revealed in pericardium?
A. *Fibrinous pericarditis
B. Purulent pericarditis
C. Hemorrhagic pericarditis
D. Proliferative pericarditis
E. Catarrhal pericarditis
16. A 50-year-old male died of a pulmonary and cardiac insufficiency. An autopsy revealed in a left lung a 4 x 4 cm
cavity, filled with pus. The wall of a cavity had a rough pattern, presented by pulmonary tissue. What is the most
likely diagnose?
A. *Acute abscess
B. Chronic abscess
C. Cavernous tuberculosis
D. Hydatidosis
E. Fibrosing alveolitis
17. A post-mortem of a man revealed in his abdominal cavity approximately 2.0 liters of a purulent exudate. A
peritoneum was found to be dull, with a grayish tint. The intestinal serous membranes were covered with grayish
membranes, which were removed easily. Which of the following is the most accurate diagnosis?
A. * Fibro-purulent peritonitis
B. Hemorrhagic peritonitis
C. Serous peritonitis
D. Tuberculous peritonitis E.
18. A histological examination of the myocardium of a man, who died from heart failure, revealed a sclerosis of the
pericardial connective tissue and diffuse infiltration by lymphocytes, macrophages, plasmocytes and single
neutrophils. What is the most likely type of inflammation in a heart?
A. * Interstitial productive.
B. Granulomatous.
C. Alterative.
D. Exudative diffuse.
E. Exudative focal.
19. A 28-year-old male presented to the hospital with a fever and weakness, (asthenia). A biopsy was taken from
one of his enlarged neck lymph nodes for histological examination. Microscopic investigation revealed the foci of
necrosis, surrounded by epithelioid cells, giant multinuclear Langhans' cells and also lymphocytes. What is the most
likely diagnosis?
A. *Tuberculosis
B. Hodgkin's disease
C. Lympholeukemia
D. Sarcoidosis
E. Syphilis
20. A microscopic examination of a kidney biopsy revealed foci with granular, eosinophilic masses in their centers.
These lesions were surrounded by an infiltrate, consisted of lymphocytes, epithelioid cells and giant Langhans1 cells.
What is the most likely pathological process?
A. * Granulomatous inflammation
B. Coagulative necrosis
C. Caseous necrosis

45
D. Alterative inflammation
E. Proliferation and derivation of macrophages
21. A young male presented to his physician with considerably enlarged, hypere-mic, painful tonsils. A gross
examination of tonsils revealed dense, dirty-gray membranes on the surface. These membranes have spread to the
hard palate and were intimately attached to tissues underneath. An attempt to remove the membranes resulted in
bleeding. Which pathological process indicates these morphological changes?
A. *Diphtheroid exudative inflammation.
B. Croupous exudative inflammation.
C. Catarrhal exudative inflammation.
D. Purulent exudative inflammation.
E. Hemorrhagic exudative inflammation.
22. A 38-year-old man, with a history of chronic pyelonephritis, died from the chronic renal failure. Medical record
showed that an auscultation in the hospital determined a patient's "pericardial frictions rub". An'autopsy revealed a
dim, rough, and hairy - like epicardium. What type of pericarditis presented in that case?
A. * Croupous
B. Diphtheroid
C. Purulent
D. Ichorous
E. Serous
23. A child presented to a hospital with symptoms of asphyxia. A physical examination revealed in the larynx
whitish membranes, which obstructed lumen and were easily separated from tissues underneath.
Name the most probable type of an inflammation in the larynx?
A. *Croupous inflammation
B. Hemorrhagic inflammation
C. Diphtheroid inflammation
D. Catarrhal inflammation
E. Purulent inflammation
24. A macroscopic investigation of trachea revealed a dim, hyperemic mucosa, covered with grey - white
membranes. Which one is the most likely form of the inflammation?
A. *Fibrinous
B. Purulent
C. Serous
D. Proliferative
E. Mixed
25. An elderly man presented with a decomposition of some segments of spongy and cortical layers of an
anticnemion bones. The cavities are filled with creamy greenish-yellow masses. What is the type of inflammation?
A. *Purulent
B. Catarrhal
C. Serous
D. Proliferative
E. Mixed
26. A female patient presented to the hospital with a fever, asphyxia and pain in the right part of her chest. During
a pleurocentesis procedure 700 ml of a viscid yellow-green liquid was removed. What is the most likely diagnosis?
A. *Empyema of a pleura
B. Carcinomatosis of a pleura
C. Serous pleurisy
D. Fibrinous pleurisy
E. Hemorrhagic pleurisy
27. An autopsy of the 58-year-old man, revealed in his liver a focus of tissue destruction, 4 cm in diameter, filled
with a yellow-green liquid. What is the most likely diagnose?
A. * Abscess
B. Phlegmon
C. Anthrax
D. Empyema
E. Granuloma
28. A histological examination of biopsy from the nose mucous revealed epi-thelioid cells, plasmocytes, Mikulicz
cells, and eosinophilic (acidophilic, Russell's) bodies. What is the most likely diagnosis?
A. * Rhinoscleroma
B. Syphilis
C. Tuberculosis
D. Droplet infection
E. Allergic rhinitis

46
29. A physical examination of a 5-year-old boy reveals the fauces and the tonsils enlarged, plethoric, and coated
with irremovable whitish membranes. Which one of the following characterizes the changes in the fauces and
tonsils?
A. * Diphtheritic inflammation
B. Caseous necrosis
C. Fibrinous necrosis
D. Croupous inflammation
E. Purulent inflammation
30. A 49-year-old man, with 14 days history of acute lobar staphylococcal pneumonia in the left lower lobe of a
lung, died of pulmonary and cardiac insufficiency. A post-mortem revealed in the upper lobe of a right lung an oval
cavity, approximately 5 cm in diameter, filled with pus and connected with bronchus. Which complication of acute
pneumonia took place in this case?
A. * Abscess of the lung
B. Gangrene of the lung
C. Thromboembolia of a pulmonary trunk
D. Bronchiectasis disease
E. Acute bronchitis
31. An autopsy of an elderly male revealed a cavity with compacted walls filled with a thick, greenish liquid, which
had a stinking odor. What is the most likely disease?
A. * Abscess of the lung.
B. Gangrene of the lung.
C. Infarct of the lung.
D. Cavern of the lung.
E. Tuberculoma of the lung.
32. A 62-year-old man died of croupous pneumonia. A post-mortem revealed in the pleural cavity an opaque liquid
and a grayish membrane on visceral pleura. What is the most likely type of inflammation on the visceral pleura?
A. * Fibrinous inflammation
B. Catarrhal inflammation
C. Purulent inflammation
D. Granulomatous inflammation
E. Hemorrhagic inflammation
33. A 5-year-old child has a fever and pain at swallowing. A physical examination revealed enlarged, dark-red
palatine tonsils. They were coated with a grey - yellow membrane which was intimately attached to the surface of
the tonsils. Which kind of inflammation described in the tonsils?
A. * Diphtheritic inflammation
B. Croupous inflammation
C. Hemorrhagic inflammation
D. Purulent inflammation
E. Catarrhal inflammation
34. An autopsy of a dead body revealed 200 ml of a viscid yellow-green liquid in the abdominal cavity. What is the
most likely form of exudate inflammation?
A. * Purulent inflammation
B. Serous inflammation
C. Fibrinous inflammation.
D. Hemorrhagic inflammation.
E. Ichorous inflammation
35. A post-mortem of a young male revealed congested, thickened, opaque, yellowish-green meninges. What type of
exudative inflammation these changes are characteristic for?
A. * Purulent inflammation
B. Serous inflammation
C. Hemorrhagic inflammation
D. Fibrinous inflammation
E. Catarrhal inflammation
36. A woman, with a history of her hands' skin thermal burn, presented to physician painful blisters, filled with
opaque liquid. What is the most likely type of inflammation?
A. * Serous inflammation
B. Productive inflammation
C. Croupous inflammation
D. Granulomatous inflammation
E. Diphtheritic inflammation
37. A 44-year-old male had a skin biopsy procedure. Histological investigation of a tissue sample showed a caseous
necrosis surrounded by infiltrate, consisted from lymphocytes, some giant cells and the growth of connective tissue.
In addition, the endovasculitis was diagnosed. What is the most likely pathological process?

47
A. * Productive granulomatous inflammation.
B. Productive interstitial inflammation.
C. Abscess.
D. Catarrhal inflammation.
E. Serous inflammation
38. A 7-year-old child, with a history of angina, presented with enlarged paratra-cheal, bifurcation and neck lymph
nodes. A histological examination of his neck lymph node revealed the foci of necrosis, surrounded by lymphocytes,
epithelioid cells and Pirogov-Langhans' cells. Which one is the most likely pathology?
A. *Tuberculosis
B. Sarcoidosis
C. Rhinoscleroma
D. Malleus
E. Syphilis
39. The histological examination of a lung tissue revealed a lesion presented with a locus of necrosis, surrounded by
regular lines of epithelioid, lymphoid cells, plasma cells, macrophages and giant Pirogov-Langhans' cells.
Define a kind of inflammation?
A. *Tnberculosis inflammation
B. Banal productive inflammation
C. Inflammation at a lepra
D. Exudative inflammation
E. Alterative inflammation
40. A 6-year-old child presented to infectious disease department with a body temperature 38°C, punctulated, bright
red rash on a skin. The scarlet fever diagnosis is made. The fauces mucosa was brightly hyperemic, edematous;
tonsils were markedly enlarged, with the dim yellowish-grayish centers and sites of black color. What inflammation
underlies changes in fauces?
A. *Pumlent-necrotic
B. Fibrinous
C. Hemorrhagic
D. Serous
E. Catarrhal
41. A rectoromanoscopy revealed edematous, reddish color mucosa of rectum and sigmoid colon, covered with a
thick layer of mucus. Name the kind of inflammation?
A. *Catarrhal
B. Purulent
C. Mixed
D. Hemorrhagic
E. Fibrinous
42 A patient died with symptoms of the brain's edema and the dislocation of the stem. A post-mortem revealed
thickened, opaque, grayish-green color pia mater. Which pathological process took place in pia mater of meninges?
A. *Diffuse purulent leptomeningitis
B. Meningococcal leptomeningitis
C. Tuberculous leptomeningitis
D. Malignant arachnoendomelioma
E. Hydrocephalus
43. A 5-year-old child presented to a hospital with acute fever (up to 38° C), rhinitis, cough, lacrimation, and a
patchy rash on the skin. His pharyngeal mucosa was edematous and hyperemic, with whitish maculae on the cheeks.
What type of inflammation is the background of described changes?
A. *Catarrhal
B. Purulent
C. Fibrinous
D. Hemorrhagic
E. Serous
44. A 3-year-old child presented to a hospital with asphyxia. Examination of the larynx revealed whitish
membranes, which occluded a lumen and easily extracted. Diphtheria was suspected. Which form of inflammation
took place in the larynx?
A. *Croupous inflammation
B. Catarrhal inflammation
C. Diphtheroid inflammation
D. Serous inflammation
E. Purulent inflammation
45. A 47-year-old male presented with a sub-febrile temperature and enlarged lymph nodes. A biopsy investigation
of a lymph node revealed numerous granulo-mas with a caseous necrosis in the centers, surrounded by epithelioid
cells, giant multinuclear Langhans' cells and lymphocytes.

48
What is the most likely diagnosis?
A. * Tuberculosis
B. Lymphogranulomatosis
C. Lymphosarcoma
D. Lympholeukemia
E. Simple lymphadenitis
46. At a 45-year-old man the biopsy procedure of his nasal cavity's mucosa was performed. The diagnosis of
rhinoscleroma made. What cells are typical for the disease?
A. *Mikulicz's cells
B. Pirogovs' - Langhans' cells
C. Plasma cells
D. Lymphocytes
E. Schaumarm"s bodies
47. An autopsy of a 53-year-old male showed many white, millet sized nodules in his lungs. A microscopic
examination revealed granulomas with a necrosis in their centers and epithelioid, lymphoid, plasma cells,
macrophages and numerous large multinucleated cells (Langhans' cells) on the periphery of a lesion.
What is the most likely form of granuloma?
A. *Giant - cell granuloma
B. Macrophagal
C. Foreign body granuloma
D. Epithelioid cells E -
48. A patient presented to the hospital with combustion of his right hand. A physical examination revealed a
desquamation of epidermis with blisters formation. The blisters were filled with opaque liquid. What most
probable inflammation described in that case?
A. * Serous
B. Purulent
C. Catarrhal
D. Putrefactive
E. Fibrinous
49. A 43-year-old patient complains of hindered nasal breaming. Histological investigation of a biopsy of his
nasal cavity mucosa revealed lymphoid cells, epithelioid cells, plasma cells and Mikulicz's cells. What is the
most likely diagnosis?
A. *Rinoskleroma
B. Glanders (malleus)
C. Tuberculosis
D. Lepra
E. Syphilis
50. A 54-year-old patient complains of complicated nasal breathing. A nasal mucosa was thickened.
Histological investigation of a biopsy revealed Mikulicz's cells, epithelioid cells, plasma cells, lymphoid cells,
and hyaline sphere- like aggregates. What is the most likely diagnosis?
A. *SkIeroma
B. Adenoviral rhinitis
C. Allergic rhinitis
D. Rhinoviral infection
E. Meningococcal nasopharyngitis
51. A sample of cervix uteri was taken for histological examination. A microscopical investigation revealed a
tissue's inflammatory infiltration with vascular walls involvement of small vessels, arteries and veins. In
addition, there were also determined a presence of plasma cells, lymphocytes, epithelioid cells in the infiltrate
and foci of sclerosis and hyalinosis of tissue. What is the most likely disease?
A. *Syphilis
B. Tuberculosis
C. Leukoplakia
D. Erosio colli uteri
E. Condyloma
52. A physical examination of tonsils and soft palate mucosa revealed white-grey color membranes, which are
intimately attached to tissues underneath. An attempt to take out membranes results in formation of a deep
tissue defect. Diagnose pathology on a mucosa of tonsils and a soft palate.
A. *Diphtheritic inflammation В Serous inflammation
C. Croupous inflammation
D. Purulent inflammation

49
E. Mixed inflammation
53. Histological investigation of a biopsy revealed a granuloma, consisted of lymphocytes, plasma cells,
macrophages with foamy cytoplasm (Mikulicz's cells) and numerous hyaline's sphere- like aggregates. About
what disease it is possible to think?
A. *Rinoskleroma
B. Lepra
С Syphilis
D. Tuberculosis
E. Actinomycosis
54. A 40-year-old woman, with a history of the right palm's burn, presented to her physician an acute pain,
reddening and swelling of a palm. In a few minutes there was a bubble, filled with transparent yellowish
liquid. The display of what pathological process the described changes are?
A. *Exudative inflammation
B. Traumatic edema
C. Alterative inflammation
D. Proliferative inflammation
E. Vacuolar dystrophy
55. A 44-year-old man presented to the policlinic with a local pain in occipital area of a head and rise of a
body temperature there. Gross investigation of a lesion zone revealed the cone-shaped cyanotic infiltration
with a yellow-greenish apex, which rose above the surface of skin. What is the most likely diagnosis?
A. * Furuncle
B. Phlegmon
C. Abscess
D. Carbuncle
E. Empyema.
56. A skin sample was taken for histological research. Microscopical investigation revealed granulomas,
consisted of macrophages' knots with adding of lymphocytes and plasma cells. There were also determined
large macrophages with fatty vacuoles, which contained packed layers of causative agents of a disease (Virchow's
cells). For what disease the given description is characteristic?
A. *Lepra
B. Tuberculosis
C. Syphilis
D. Rinoskleroma
E Glanders (malleus)
57. Histological investigation of a skin biopsy revealed granulomas, which consisted of epithelioid cells,
surrounded mainly by T-lymphocytes. In addition, few giant rnultinucleated cells (Langhans* cells) were
determined among epithelioid cells. The caseous necrosis was diagnosed within the centers of granulomas. There
was lack of blood vessels. What is the most likely disease?
A. Tuberculosis
B. Syphilis
C. Lepra
D. Rinoskleroma
E. Glanders (malleus)
58. A histological investigation of a liver biopsy revealed granulomas, consisted of plasma cells and lymphoid cells.
Small vessels were characterized by endo - and perivasculitis. Some foci of caseous necrosis were also determined.
Such granulomas are typical for:
A. *Syphilis
B. Tuberculosis
C. Lepra
D. Rinoskleroma
E. Glanders (malleus)
59. A young man had a removal of mucous membrane new growths in his nasal cavity. A histological investigation
revealed diffuse lymphocytes, plasma cells and macrophages infiltration. Name the type of inflammation.
A. *Inflammation with formation of polyps
B. Inflammation with formation pointed condyloma
C. Interstitial inflammation
D. granulomatous inflammation E.Exudative inflammation
60. A 4-year-old child presented to his physician with a cold, high temperature, nausea and vomiting. He was
hospitalized and died in two days. An autopsy revealed thickened, dim, edematous, congested, greenish-yellow pia
mater of men-inges. What variety of exudative inflammation developed in pia maters?
A. *Purulent

50
B. Catarrhal
C. Hemorrhagic
D. Fibrinous
E. Serous
61. A young man was operated because of phimosis. Histological investigation of a removed prepuce revealed
polymorphic infiltrates, consisted of randomly located plasmocytes, epithelioid and lymphoid cells. The vasculitis
was diagnosed in vessels. What disease such changes are characteristic for?
A. * Syphilis
B. Tuberculosis
C. Actinomycosis
D. Periarteritis nodosa
E. Lepra
62. Microscopic investigation an elderly man's lungs revealed the foci of necrosis, surrounded by the bank of
epithelioid cells and lymphocytes. Between lymphocytes and epithelioid cells the rounded giant cells, with a
considerable quantity of nuclei, located on a periphery of a lesion were determined. Give the name for these
changes:
A. *Tuberculous granuloma
B. Sarcoid granuloma
C. Syphilitic granuloma
D. Lepromatous granuloma
E. Scleromatous granuloma
63. A 6-year-old child presented to department of infectious diseases with acute pain in a throat, difficulty at
swallowing, rise of body temperature up to 39°C and edema of neck. A gross oral investigation revealed enlarged,
hyperemic tonsils, covered with plenty of yellow membranes, which intimately attached to the mucous membrane.
An attempt to take off membrane results in deep, bleeding defect formation. What type of inflammation takes place?
A. *Diphtheritic
B. Purulent
C. Serous
D. Croupous
E. Hemorrhagic
64. A fragment of a stomach mucosa was taken for histology during endoscopy procedure. A microscopic
investigation revealed intact mucous membrane, covered by mucus, thickened, edematous and hyperemic with a
numerous tiny hemorrhages. Define the type of acute gastritis?
A. * Catarrhal (simple)
B. Erosive
C. Fibrinous
D. Purulent
E. Necrotic
65. An operatively removed appendix was sent to pathology department. Macro-scopical investigation revealed
markedly enlarged appendix. His serous membrane was dim, congested, covered by fibrin's membranes. The
appendix' walls were thickened. On a cut section pus in the lumen was detected. Microscopical investigation showed
a plethora of vessels, edema of all appendixes' layers and diffuses infiltration by leucocytes. What is the type of
inflammation?
A. *Phlegmonous
B. Apostematous
C. Simple
D. Superficial
E. Gangrenous
66. A 18-year-old girl had a neck lymphatic node removal at the biopsy procedure. Histological investigation
showed nodules, consisted of banks of epithelioid, lymphoid cells and large multinucleated cells (Langhans' cells),
located between them. In the centers of nodes a caseous necrosis was determined. What causative agent needs to be
revealed in the necrosis zone for making final diagnosis?
A. *Koch's mycobacterium
B. Mycobacterium leprae
C. Treponema pallidum
D. Rickettsia
E. Fungi
67. A young man complains of an itch and a skin reddening, which has appeared after shaving. Objectively: a cheek
skin is hyperemic; the blisters, filled with a transparent liquid, were detected. What character of a liquid in blisters?
A. * Serous exudate
B. Transudate
C. Purulent exudate
D. Mucosal exudate

51
E. Hemorrhagic exudate
68. A 4-year-old boy presented to a hospital with pain in a throat at swallowing and malaise. A physical examination
revealed grayish - whitish membranes on a pharynx and tonsils on a background of moderate edema and hyperemia.
These membranes were intimately attached to tissues underneath. What pathology the described changes testify to?
A. inflammation
B. Dystrophy
C. Necrosis
D. Metaplasia
E. Reganerative process
69. A 68-year-old man died from meningitis. An autopsy revealed thickened, congested, dim, edematous meninges
(pia maters), which were saturated by creamy greenish-yellow exudate. Define the type of inflammation?
A. *Puralent
B. Serous
C. Fibrinous croupous
D. Fibrinous diphtheritic
E. Hemorrhagic
70. Biopsy of an oral cavity's ulcer presented for a histological research. A microscopical investigation revealed
areas of caseous necrosis, surrounded by plasmo-cytes, epithelioid, lymphoid cells and few giant multinuclear
Pirogov-Langhans cells. There were also small vessels with signs of endo- and perivasculitis. Choose the most likely
diagnosis.
A. *Syphilis
B. Tuberculosis
C. Lepra
D. Rinoskleroma
E. Glanders (malleus)
71. An autopsy of 32-year-old male revealed slightly swollen and hyperemic lungs, covered with numerous millet-
like grayish nodules. Histological investigation determined lesions consisted of lymphoid, epithelioid and Langhans
giant cells. What is the most likely diagnosis?
A. *Tuberculosis
B. Croupous pneumonia
C. Bronchopneumonia
D. Interstitial pneumonia
E. Bronchiectatic disease
72. An operatively removed appendix was sent for histological examination. Mac-roscopical investigation revealed
thickened appendix. His serous membrane was dim, congested, with whitish, loose membranes; the lumen contained
turbid, whitish-yellow exudate. Histological study showed the diffuse neutrophil's infiltration of the appendix wall.
Choose the most likely type of appendicitis.
A. *Phlegmonous
B. Gangrenous С Simple
D. Superficial
E. Chronic
Immunopathological processes
1. A 15 year old boy died of the heart failure. He had a history of rheumatic heart disease in his medical record.
Histological investigation of the left auricle of the heart revealed the rheumatic granuloma (the Aschoff-Talalayev
body) with a central fibrinous focus associated, with a perimeter plasma cells, macrophages and giant cells. Which
of following best describe the type of immune response?
A. *Cell mediated (Type IV, delayed hyper sensitivity reaction)
B. Anaphylactic (Type I, immediate)
C. Cytolytic (Type II, immediate)
D. Immune complex (Type III, immediate)
E. Reaction of transplant rejection.
2. A 4-yers-old boy presents with multiple malformation of craniofacial bones. He died soon due to sepsis, resulted
from bronchopneumonia. Blood test revealed the immunoglobulins within normal range. At autopsy the thymical
aplasia was identified. Name the mane cause of death.
A. *Cellular immunodeficiency
B. Combined immunodeficiency
C. Secondary immunodeficiency
D. Acute leukemia
E. Chronic intoxication
3. A 48-year-old women with asthma presents with viral respiratory infection. The infection caused a status
asthmaticus with a fatal outcome. Histological investigation of lung tissue revealed a contracted bronchus and a
luminal plug containing mucus and cell debris. The submucosa was edematous and contained a mixed inflammatory

52
infiltrate, including a lot of eosinophils and degranulated must cells. What is the mechanism of hypersensitivity
presented in this case?
A. *Humoral immunity (Type I, armphylactic reaction)
B. Inflammatory reaction
C. Autoimmune reaction
D. Immune complex reaction (Type III of reaction)
E. Cell-mediated immunity (Type IV of reaction)
4. A young man presents with the enlargement of the regional lymph nodes next to his inflamed wound,
Histological investigation of the lymph node biopsy revealed the increased number of macrophages, lymphocytes
and lymphoid follicules within cortex zone. Plasma cells number also increased. These alterations are most likely
result of:
A. * Antigen stimulation
B. The acquired insufficiency of lymphoid tissue (secondary immunodeficiency state)
C. Hereditary immunodeficiency state
D. Neoplastic transformation
E. Hypersensitivity reaction
5. A newborn died after 3 days of life, as a result of a lethal outcome of intrauterine pneumonia. At autopsy a
newborn is sound have a great reduction of his thymic weight and it's sizes. Histological investigation of thymus
revealed the unclear boundary between cortex and medulla zone as a result of lymphocytes amount reduction. A few
Hassall corpuscles were found. They were focally keratinized and had concentric aggregated of epithelial cells. The
gland was also exhibited lobe atrophy and sclerosis. What pattern of reaction did the thymus demonstrate?
A. *Accidental thymus transformation (involution)
B. Thymus atrophy
C. Thymus hypotrophy
D. Thymus aplasia (agenesis)
E. Thymus neoplasia
6. A 3-monm-old child died because of sepsis. An autopsy revealed thymic hy-poplasia, both sizes and mass of
spleen reduction. Histologically, in spleen there was lack of the periarterial follicular T-zone and remarkable
absence of cells at a red spleeny pulp. Lymph nodes investigation showed also absence of cells within paracortical
(T-cells area). B-cells areas in immune system organs looked normal. What term best characterizes described above
changes?
A. Di George's syndrome
B. Combined immunodeficiency syndrome
C. Bruton's disease (X-linked agammaglobulinemia)
D. Accidental thymus transformation
E. Acquired immunodeficiency syndrome (AIDS)
7. A morphological investigation of a kidney graft revealed a diffuse stromal infiltration by lymphocytes,
plasmocytes, lymphoblasts, plasmablasts, and also a ne-crotic arteriitis. What pathology has developed in a graft?
A. *Immunological graft rejection
B. Glomerulonephritis
C. Ischemic kidney damage
D. Tubular necroses
E. Pyelonephritis
8. An 8-year-old girl, with a history of acute tonsillitis 3 weeks before the application, presented with nephrotic
syndrome (proteinuria, haematuria and cylindruria). These testify the glomerular basal membrane's lesion. What is
the mechanism of the basal membrane pathology?
A. * Immune complex mechanism
B. Granulomatous mechanism
C. Antibody-mediated mechanism
D. Reagin-mediated mechanism
E. Cytotoxical mechanism
9. At 46-year-old patient with autoimmune gastritis a biopsy was taken. Histologically, there was an infiltration by
lymphocytes and macrophages of a gastric mucous membrane. What type of hypersensitivity presented in that case?
A. * Type IY (cell-mediated hypersensitivity)
B. Type II (reagin type)
C. Type II (antibody-mediated cytotoxicity)
D. Type III (immune complex cytotoxicity)
E. Type V (granulomatous)
10. Histological investigation of a lymph node revealed a considerable quantity of the enlarged lymphoid follicles
with big germinal centers, and plenty of mitotic figures. What pathology these changes are characteristic for?
A. *Antigen stimulation with follicle hyperplasia
B. Atrophy of lymphoid tissue
C. Lymphosarcoma

53
D. Lymphogranulomatosis
E. Lympholeukemia
11. A child had a subcutaneous injection of tuberculin for a testing. Twenty-four hours later a physical examination
revealed the expressed hyperemia and induration of tissues in the place of injection. What was the mechanism of
these changes?
A. "Cellular cytotoxicity
B. Reagin cytotoxicity
C. Antibody cytotoxicity
D. Granulorhas formation
E. Immune complex
12. A 6-year-old child had the Mantoux test. In 3 days there was an inflammatory induration and a reddening of a
skin 15 mm in diameter. That was considered as the positive test. What type of hypersensitivity reaction presented in
that case?
A. *Hypersensitivity of slow type
B. Immune complex hypersensitivity
C. Complement-mediated cytotoxicity
D. Hypersensitivity immediate type E.
13. Physical examination of a pregnant woman with an Rh-negative blood revealed a high level of antierythrocyte
antibodies. The cutaneous flap of her Rh-positive husband applied for depression of antibodies. In 2 weeks a flap
was rejected. Microscopical investigation showed a disturbance of blood circulation, edema and cellular infiltration
mainly by lymphocytes, neutrophils and macrophages. What is the most probable pathology?
A. *Transplantation immunity
B. Hypersensitivity reaction immediate type
C. Hypersensitivity reaction slow type
D. Granulomatous inflammation
E. Interstitial inflammation
14. A 3-year-old child died of an acute destructive staphylococcal pneumonia. Macroscopically, thymus mass
reduced to 3,0 gr. Microscopical investigation of thymus showed reduction of lobules, considerable decline of
lymphocytes quantity, stromal collapse, and inversion of layers with cystic enlargement of the HassaH's bodies.
What is the most likely diagnosis?
A. *Accidental thymus involution
B. Thymomegaly
C. Thymus hypoplasia
D. Thymus dysplasia
E. Thymus agnenesia
15. The experiment animal with a history of previous sensibilization received the next subcutaneous dose of
antigen. In the place of injection a fibrinous inflammation with alteration of vessels walls, matrix and fiber
structures of connective tissue (mucoid and fibrinoid swelling) and fibrinoid necrosis developed. What immu-
nological reaction takes place?
A. * Hypersensitivity reaction immediate type
B. Hypersensitivity reaction slow type
C. Reaction of transplantation immunity
D. Normergic reaction
E. Granulomatosis
16. A man with a history of bronchial asthma died of asphyxia. Microscopical investigation of lungs revealed
excess of mucus with lots of eosinophils in the bronchial lumen, sclerosis of interalveolar septas and alveoli
dilatation. What mechanism of hypersensitivity reaction presented in that case?
A. *Reagin reaction
B. Cytotoxic reaction
C. Immune complex reaction
D. Cytolysis, conditioned by lymphocytes
E. Granulomatosis
17. The infiltration anesthesia by ultracain with adrenaline solution has been done to a young man. Then redness,
edema of the skin with itchy blisters suddenly developed. What type of hypersensitivity took place?
A. * Anaphylaxis
B. Cytotoxicity
C. Immune complex damage
D. Hypersensitivity slow type
E. Granulomatosis
18. A 33-year-old patient with a history of acute tonsillitis presented with the urinary syndrome (haematuria,
proteinuria and leukocyturia). Kidneys biopsy revealed an intracapillary proliferative glomerulonephritis. Electronic
microscopy determined large subepithelial deposits. What is the pathogenesis of this disease? A.*Immune complex
mechanism

54
B. Atopy
C. Cytotoxic, cytolytic action of antibodies
D. Cellularally caused cytolysis
E. Granulomatosis
19. A 10-month-old child has developed pneumonia. Clinical and laboratory tests revealed absence of mature B-
lymphocytes in peripheral blood and in B-zones of lymph nodes, acute reduction of immunogiobulins in serum. In
peripheral blood a total quantity of lymphocytes was normal. What kind of an immunodeficiency developed in that
case?
A. *Brutone sindrome
B. Neselof syndrome
C. Severe combined immunodeficiency
D. Di-George syndrome
E. Viskotta-Oldrich syndrome
20. A patient with a history of serious radioactive irradiation had bone marrow transplantation. In 2 months a rush
on the skin and diarrhea developed. Clinical and laboratory tests showed hepatic insufficiency, local necrosis of the
skin's epithelial cells, intestinal crypts and liver parenchymal cells. What was the reason for symptoms aggravation?
A. * "Graft against the owner" disease
B. Sepsis development
C. An acute graft rejection
D. A chronic graft rejection
E. An exacerbation of a chronic hepatitis
21. A 7-month-old child died of severe pneumonia complicated by sepsis. A postmortem revealed an absence of
thymus. In lymph nodes there were no lymphoid follicles and cortical substance, in a spleen follicles were markedly
reduced, germinal centers were absent. What is the most likely diagnosis?
A. *Thymus agenesia
B. Thymus aplasia
C. Thymus atrophy
D. Thymus hypoplasia
E. Acidental thymus involution
22. A 1 -year-old child died of a relapsing pneumonia. A post-mortem revealed hypoplasia of a thymus and
peripheral lymphoid tissue, atrophy of cerebellum cortex, angiotelectasis of bulbar conjunctiva.
Immunohistochemistry revealed decreased level of immunoglobulins. What is the most likely disease?
A. * Louis-Bar's syndrome
B. Di-George syndrome
C. Nezelof syndrome
D.West's syndrome
E. Brutone syndrome
23. A 20-year-old woman died of postnatal sepsis. A post-mortem revealed enlarged congested spleen with
abundant curettage from a cut section. Microscopical investigation showed hyperplasia and infiltration by
plasmocytes of the red pulp and spleen's follicles. The red pulp was also rich with macrophages. What pathology
resulted in spleen's alteration?
A. * Antigenic stimulation of an organism
B. Hereditary insufficiency of a peripheric lymphoid tissue
C. Reaction of hypersensitivity of immediate type
D. Reaction of hypersensitivity of the slowed type
E. Autoimmunization
24. Microscopical investigation of arterioles showed plasmatic saturation, mucoid and fibrinoid swelling, fibrmoid
necrosis of walls, diffuse infiltration by lymphocytes, plasmocytes and monocytes. In addition, the focal
proliferation of hystio-cytes, endotheliocytes and pericytes were determined. The final diagnosis was the
"periarteritis nodosa". What type of inflammation took place in arterioles?
A. * Acute immune inflammation
B. Acute not immune inflammation
C. Focal exudative inflammation
D. Diffusive exudative inflammation
E. Fibrinous inflammation
25. During operation of kidney transplantation in 15 minutes after vascular anastomoses application, transplant
became flabby, cyanotic and spotty; the pulsation was absent. Microscopical investigation of a kidney specimen
revealed periendothelial accumulation of neutrophils in glomeruli vessels. In addition there were also a thrombosis
and necrosis of some glomeruli. Specify the type of graft rejection.
A. *Hyperacute rejection
B. Accelerated rejection
C. Acute intersticial rejection
D. Acute vascular rejection

55
E. Chronic rejection
Adaptation and compensation.
1. A young male complains of diminishing leg muscles in size and volume as a result of the complicated femoral
fracture. The innervations of the muscles was not lost. Name the type of atrophy?
A. * Disuse atrophy
B. Degeneration atrophy
C. Ischemic atrophy
D. Atrophy due to compression
E. Atrophy due to chemical and physical influence.
2. At autopsy a male is found to have big kidneys with marked dilatation of the renal pelvis and calyces filled with
transparent liquid. A thinning of the renal parenchyma accompanied above lesions. Which is the most likely
finding?
A. *Hydronephrosis
B. Glomerulonephritis
C. Amyloidosis
D. Tuberculosis
E. Pyelonephritis
3. An autopsy of an elderly male revealed the atherosclerosis of the brain arteries accompanied with the thinning of
the brains cortex. Name the cause of the atrophy?
A. *Ischemia
B. Compression
C. Chemical and physical factors
D. Loss of innervations
E. Decreased workload
4. A 62-yer-old male died at the hospital after developing heart failure. At autopsy the weight of the heart was
SOOgm, with thickened right ventricular wall and dilated chamber. Name the alteration at the right ventriculum.
A. * Hypertrophy
B. Hyperplasic alteration due to inflammation
C. Metaplasia
D. Atrophy
E. Sclerosis
5. A young male presents with a large and lacerated wound covered with pus. The peripheral zone of the tissue
defect was filled with large amount of granulation tissue. Name the type of healing?
A. * Healing by first intention
B. Healing by second intention
C. Healing under a scab
D. Covering a tissue defect with epithelial cells
E. Organization followed by scarring
6. An autopsy revealed marked enlargement of a kidney. Gross examination of a kidney's section presented with
dilation of both renal pelvis and calyces resulted from renal stones. What from diagnoses is most faithful?
A. *Hydronephrosis
B. Simple cysts
C. Pyelonephritis
D. Benign nephrosclerosis
E. Polycystic kidney disease
7. A physical investigation of an elderly man with history of esophagus cancer with stenosis but without metastasis
revealed an atrophy of skeletal muscles and subcutaneous fat tissue. Skin was grayish-brown color, an epidermis
was thinned, and size of the heart decreased. Myocardium and liver also had a brown coloring. What is the most
likely diagnosis?
A. * Alimentary cachexia.
B. Myasthenia.
C. Cancer cachexy
D. Brown atrophy.
E. The Addisona Illness
8. At the man with long history of smoking the bronchial biopsy is taken. Microscopical investigation showed the
thickened mucous membrane with chronic inflammation and transformation of ciliated epithelium into stratified
one. What pathology such changes are characteristic for?
A. *Metaplasia
B. Нуperplasia of epithelium
C. Planocellular cancer
D. Leukoplakia
E. Hypertrophy of epithelium

56
9. A patient with a history of frequent hemorrhoid bleeding died of acute myocar-dial infarction. A post-mortem
revealed a red, succulent bone marrow of a hip diaphysis. What pathology developed in a bone marrow?
A. *Compensatory hyperplasia
B. Vicarious hypertrophy
C. Hypertrophy excrescences
D. Hormonal hyperplasia
E. Work hypertrophy
10. A 59-year-old patient with a history of prostate glandular hyperplasia operated in the urology. An operation
revealed a thickening of a urinary bladder wall up to 1 cm. It may be caused by:
A. *Work hypertrophy
B. Vicarious hypertrophy
C. Hormonal hypertrophy
D. Hormonal hyperplasia
E. Protective hyperplasia
11. A 7-year-old child presented with a poliomyelitis. His somatic muscles are weak, their volume is reduced, and
skin is dry and pale. Choose the most likely pathology, which takes place in the soft tissues?
A. *Atrophy.
B. Hypertrophy.
C. Hyperplasia.
D. Metaplasia.
E. Hypoplasia.
12. An elderly man with a long history of smoking presented with a cough accompanied by viscid mucous sputum,
weakness after the insignificant physical activity and pale color of the skin. He lost 12.0 kg of weight for the last
two months. Microscopical investigation of endoscopic biopsy showed squamous cell carcinoma. What pathology
preceded the tumor formation?
A. *Metaplasia.
B. Hypoplasia.
C. Hyperplasia.
D. Necrosis.
E. Sclerosis.
13. A 48-year-old patient with a history of a hypertensive disease died of heart failure. An autopsy revealed an
enlarged heart with dilated chambers. A thickness of the left ventricle wall was 2,5cm. Microscopically myocardial
cells were considerably enlarged, with fatty dystrophy and hyperchromic barrel-shaped nuclei. What is the most
likely pathology diagnosed in a heart?
A. *Excentric hypertrophy
B. Myocarditis
C. Concentric hypertrophy
D. Angiogenic cardiosclerosis
E. Cardiomyopathy
14. A 60-year-old man has a history of chronic bronchitis. Histological investigation of bronchus biopsy showed
thinned mucous membrane, cystic transformation of mucous glands, and replacement of prismatic epithelium on
stratified one. What of the listed pathological processes the most likely?
A. * Metaplasia.
B. Hyperplasia.
C. Heterotopia.
D. Heteroplasia.
E. Displasia.
15. Microscopical investigation of tissue from the edge of a chronic gastric ulcer showed necrosis, granulation
tissue, abundant development of connective tissue and metaplasia of epithelium. What type of pathology takes place
in that case?
A. *Pathological regeneration with disorder of phases.
B. Hypertrophy.
C. Physiology regeneration.
D. Reparative regeneration (substitution).
E. Reparative regeneration (restitution).
16. At a patient with a chronic cystitis biopsy of urinary bladder mucosa was taken. Microscopical investigation
revealed a transitional epithelium with foci of stratified epithelium without keratinization. What process underlies
the described changes in an epithelium?
A. *Metaplasia
B. Dystrophy
C. Hyperplasia
D. Dysplasia
E. Hyperkeratosis

57
17. After traumatic damage of a liver, subsequently there was complete hepatic structural and functional restoration.
Name the type of regeneration?
A. *Restitution
B. Incomplete regeneration
C. Pathological regeneration
D. Physiological regeneration
E. Substitution
18. At a patient with chronic bronchitis biopsy of bronchus mucous membrane was taken. Microscopical
investigation revealed the areas of stratified epithelium without keratinization. What pathology takes place in the
mucous membrane of bronchus?
A. *Metaplasia
B. Atrophy
C. Hypeiplasia
D. Dysplasia E.
19. A 42-year-old woman presented to the gynecologist with acyclic, profuse hem-orrhagic discharge. A histological
investigation of a curettage material revealed increased quantity of glands and their cystic dilation. What is the most
likely diagnosis?
A. * Endometrial hyperplasia
B. Endometrial atrophy
C. Endometrial hypertrophy
D. Metaplasia
E. Organization
20. A post-mortem of a 64-year-old woman with a long story of hypertensive disease revealed considerably
diminished, dense kidneys with a fine-grained surface. What pathology such changes are characteristic for?
A. * Blood insufficiency atrophy
B. Atrophy from pressure
C. Senile atrophy
D. Dysfunctional atrophy
E. Hypoplasia
21. An elderly man died of cardiac insufficiency. Twenty years ago he had the right lung pulmonectomy because of
a cyst. A post-mortem revealed an enlarged left lung. Diagnose pathology in the left lung.
A. *Vicarious hypertrophy
B. Neurohumoral hypertrophy
C. Dysfunctional atrophy
D. Dyscirculatory atrophy
E. Neurotic atrophy
22. A 42-year-old man had a lower extremity amputation. After a while painful knots appeared in a stump.
Microscopical investigation revealed amputation neuromas. What pathology such changes are characteristic for?
A. *Pathological regeneration
B. Complete reparative regeneration
C. Inflammation
D. Incomplete reparative regeneration
E. Metaplasia
23. A 38-year-old woman presented with complaints about the frequent uterine bleeding. The diagnostic curettage
was performed. A histological investigation of a curettage material revealed increased quantity of coiled glands,
some of them were with cystic dilation. What pathology these changes are characteristic for?
A. *Glandular-cystic hypeiplasia.
B. Atrophy
C. Metaplasia
D. Dysplasia
E. Hypertrophic vegetations
24. A man had the right lung pulmonectomy because of a tumor 7 years ago. Then the volume of the left lung
increased on 40%. What process developed in the left lung?
A. *Vicarious hypertrophy
B. Neurohumoral hypertrophy
C. False hypertrophy
D. Work hypertrophy
E. Hypertrophic vegetation
25. A histological investigation of an endometrium revealed coiled extended glands with 'saw-' and a 'spin-like'
pattern. A strornal proliferation with hypeiplasia of its cells was also determined. What is the most likely diagnosis?
A. * Glandular hyperplasia of endometrium
B. Acute endometritis
C. Leiomyoma

58
D. Hydatidiform mole
E. Placental polyp
26. A patient presented with small knots on a thin peduncle in his rectum. Microscopical investigation of the biopsy
material revealed a growth of connective tissue and glands. What is the most likely pathology?
A. *Hyperplastic polyp.
B. Hypertrophy.
C. Metaplasia.
D. Atrophy.
E. Sclerosis.
27. The biopsy of central bronchus mucosa of miner is taken. Microscopically, there is a transformation of a
cylindrical epithelium to the mature stratified one. What pathology such changes are characteristic for?
A.* Metaplasia
B. Hyperplasia
C. Leucoplakia
D. Dysphasia
E. Reparative regeneration
28. The biopsy of a vaginal portion of uterus cervix is taken. There is an augmentation of quantity of cells, rising of
mitotic activity and alteration of polarity of cells in basal part of an epithelial layer. What pathology such changes
are characteristic for?
A.* Dysphasia
B. Metaplasia
C. Atrophy
D. Acantosis
E. Hyperplasia
29. A 40-year-old man has a keloid scar after a treatment of the left arm burn. This process may be an example of:
A. *Pathologic regeneration
B. Hyperplasia
C. Metaplasia
D. Reparative regeneration
E. Vicarious hypertrophy
Epithelium neoplasms
1. A teenager male presents with slowly enlarging, painless nodule on his right hand's skin. Microscopic
examination of removed lesion revealed an increased number of epithelial layers, with a stroma underneath, with
developed a papilloma-tous pattern. Identify most likely type of atypia?
A. * Tissue
B. Cellular.
C. Metabolic.
D. Functional.
E. Hystochemical
2. A 35 year-old female with a family history of colon cancer inquires about screening. Colonoscopy revealed a
tumor and a tissue sample was taken for histology. Microscopic investigation has shown cells were arranged in
glandular-like pattern. They exhibit cell pleomorphism, atypia, invasive growth and pathologic mitosis's. What is the
most likely diagnose?
A. *Adenocarcinoma
B. Basal-cell carcinoma
C. Solid carcinoma
D. Mucous carcinoma
E. Undifferentiated carcinoma
3. A 50-year-old female is found to have a urinary bladder neoplasm. Biopsy investigation revealed that tumor
consists of thin, branching out papillae, covered with few layers of transitional cell epithelium. What is the most
likely diagnose?
A. *Papilloma
B. Basal cell
C. Transitional cell carcinoma
D. Squamous cell carcinoma
E. Fibroadenoma
4. A 60-year-old postmenopausal woman has been feel unhealthy and weak for about 3 month. A gross
investigation of her cervix uteri revealed a lesion and a biopsy from this area was obtained. Microscopically, a lesion
composed of atypical squamous cell, many of them displayed pathological mitoses. Keratin pearl formation was also
observed histologically. What is the most likely diagnose?
A. * Squamous cells carcinoma with keratin і sation
B. Transitional cell carcinoma
C. Squamous cells without keratinisation

59
D. Adenocarcinoma
E. Anaplastic carcinoma
5. A 48-year-old woman complains to her doctor of uterine bleeding. Endometrial biopsy investigation revealed a
presence of gland-forming cells with enlarged hyperchromic nuclei, cell atypia, abnormal mitoses. These cells were
also determined at myometrium. What term most correctly identifies this pathological process?
A. * Adenocarcinoma of uterus.
B. Adenomatous endometrial hyperplasia.
C. Acute endometritis.
D. Placental polyp.
E. Chorionepitelioma of uterus.
6. A 39-year-old woman presented in oncology with a stomach cancer. Physical investigation revealed metastatic
tumors in ovaries (Krukenberg tumors). What is most likely pattern of spread in that case?
A. * Lymphatic retrograde
B. Lymphatic orthograde
C. Gematogenous
D. Implantation
E. Along epithelium-lined surfaces
7. A 23-year-old female decided to have a surgeon's removal of a small nodule at her leg's skin. An operation
material delivered to pathology department. A histological examination of a new growth has shown that parenchyma
has been formed of integumentary epithelium with an increased amount of layers. Strorna together with epithelial
proliferation forms papillae. What is the most likely form of atypism?
A. *Tissue.
B. Cellular.
C. Histochemical.
D. Functional.
E. Metabolic.
8. A 45-year old male with a family history of a gastric cancer inquires about screening. A gastroscopic
examination of a patient revealed a pedunculated tumor mass 1.5 cm in diameter in the area of the lesser curvature
of stomach. What kind of growth does the tumor have?
A. *Exophytic.
B. Expansive.
C. Infiltrating.
D. Appositional.
E. Endophytic.
9. A 70 year-old male is found to have a nodule at his right bronchus mucous membrane. A microscopic
examination of the biopsy material from the tumor revealed the cell and tissue atypism, keratin pearls formation.
What is the most likely pathologic process at biopsy material?
A. *Malignant tumor.
B. Benign tumor.
C. Hyperplasia.
D. Metaplasia.
E. Hypoplasia.
10. A 57-year-old postmenopausal woman is found to have a chronic nonhealing lesion at her portio vaginalis
uteri.A biopsy was taken form this area. A histological examination of the biopsy material revealed a cellular
atypism within epithelial layer, but basic membrane was unchanged. What is the most likely diagnosis?
A. *Carcinoma in situ.
B. Erosion.
C. Adenocarcinoma.
D. Papillorna.
E. Endometriosis.
11. A 15-year-old boy is found to have a small, dense, nodule at his right forearm skin. Grossly it had a papillary
surface, which looked like a cauliflower. Microscopically, the tumor consists of many papillae. Parenchyma formed
of integumentary epithelium with an increased amount of layers. The polarity of epithelial cells, their stratification
and membrane wholeness are preserved. A connective tissue forms a stroma within a center of papillae. What is the
most likely diagnosis?
A. *Papilloma.
B. Fibroma.
C. Adenoma.
D. Fibroadenoma.
E. Cystadenoma.
12. A 66-year old male lost his appetite and has been loosing his weight for about 5 months. X-ray study revealed a
stomach neoplasm. Histological examination of a tumor's biopsy showed a great amount of signet-ring cells. Name
the histological variant of cancer.

60
A. * Adenocarcinoma.
B. Solid carcinoma.
C. Sarcoma.
D. Mucinous carcinoma. E..Carcinoid.
13. A 45-year-old woman has a small tumor of her left breast. Histological examination of a breast biopsy revealed
poorly differentiated atypical epithelial cells. They formed trabecules separated from one another by connective
tissue. The cells arid stroma proportion was approximately 1:1. What is the most likely histological variant of the
cancer.
A. * Solid carcinoma.
B. Adenocarcinoma.
C. Epidermoid carcinoma.
D. Sccirrhous fmrocarcinoma.
E. Small cell carcinoma.
14. Histological investigation of a node in the removed mammary gland revealed complexes of atypical
polymorphic epithelial cells, which had various sizes and forms. There were clear spaces at the centers of
complexes. The cells had large nuclei, with presence of atypical mitoses. Diagnose the pathology,
A. *Adenocarcinoma
B. Squamous cell nonkeratinous carcinoma
C. Solid carcinoma
D. Fibroadenoma of breast
E. Nondifferentiated polymorphocellular carcinoma
15. A microscopical investigation of a tumour revealed signet-ring cells. Name a histological variant of a cancer.
A. *Mucinous carcinoma
B. Adenocarcinoma
C. Sarcoma
D. Solid carcinoma
E. Carcinoid
16. A 34-year-old woman presented with a hoarseness of a voice, A laryngoscopy revealed a tumour of a larynx.
The neoplasm had a grey-white color and papillary surface. Microscopical investigation showed a new growth of a
connecting tissue, covered by a stratified epithelium with the marked keratinization. Cellular atypia was absent.
Most likely the tumor is:
A. *Papilloma
B. Fibroma
C. Polyp
D. Angioma
E. Angiofibroma
17. A microscopical investigation of a 50-year-old woman's endometrial curettage material with the clinical
diagnosis « ovarian - menstrual cycle's disorder » revealed growth of glandular structures. These glands consisted of
polymorphic cells with hypercliromic nuclei and mitoses figures. For what pathology the revealed histological
changes are characteristic?
A. *Adenocarcinoma of uterus
B. Placental polyps
C. Acute endometritis
D. Glandular hyperplasia of endometrium
E. Chorioepithelionia of uterus
18. A 52-year-old female patient with a history of the chronic bronchitis and pneu-mosclerosis presented to the
hospital for biopsy diagnostics. A microscopical investigation of a left bronchial's mucous from the suspicious site
revealed cellular and tissue atypia. There were also found some structures in the form of cancer pearls". What is the
most likely pathology presented in that case?
A. * Squamous cell keratinous carcinoma of bronchus
B. Chronic polypous bronchitis
C. Bronchiectasis
D. Sharp bronchitis
E. Squamous cell metaplasia of mucous of bronchus
19. Histological investigation of a bronchial biopsy revealed a tumour which is constructed from nests of stratified
epithelium's atypical cells with some characteristic "pearls". What is the most likely diagnosis?
A. *Squamous cell keratinous carcinoma
B. Squamous cell nonkeratinous carcinoma
C. Solid carcinoma
D. Mucous carcinoma
E. Scirrhous carcinoma

61
20. A microscopical investigation of a breast tumour revealed that the neoplasm was constructed of undifferentiated
atypical epithelial cells. These cells formed trabecules, separated by layers of a connective tissue. A parity of cells
and stroma was approximately 1:1. Name a histological variant of a cancer.
A. *Solid carcinoma
B. Adenocarcinoma
C. Squamous cell carcinoma
D. Fibrocarcinoma
E. Small cell carcinoma
21. Histological investigation of the removed breast node revealed different sizes and the form complexes of
atypical polymorphic epithelial cells among abundant stroma. The complexes had a clear space center. Cells were
characterized by large nuclei, the increased number of nucleoli, nucleoli organizers and presence of atypical mitoses.
What is the most likely diagnosis?
A. *Adenocarcinoma
B. Fibroadenoma of breast
C. Solid carcinoma
D. Squamous cell nonkeratinous carcinoma
E. Nondifferentiated polymorphic cells carcinoma
22. A post-mortem of a 59-year-old man, who died of a lung cancer, revealed plural metastases. What kind from the
listed below metastases it is possible to regard as implantation (contact) one according to a mechanism of
development?
A. *Multiple tumorous nodules of pleura
B. Metastasis in prebronchial, paratracheal lymphatic knots
C. Metastasis in a brain
D. Metastases in an adrenal gland
E. Invasion of tumor from bronchus in an esophagus
23. A 55-year-old man presented to a hospital with the atelectasis of a right lung's middle lobe, resulted from the
obturation of a midlobar bronchus by a node of soft tissues. A bronchoscopy revealed a new growth within an
obturation zone. A microscopical investigation of a biopsy sample showed the growths of an atypical glandular
epithelium with pathological mitoses, which spread in tissues underneath and a cartilage. What is the most likely
disease?
A. *Bronchogenic carcinoma of lungs
B. Dysplasia of epithelium of bronchus
C. Inflammatory polyps
D. Deforming bronchitis
E. Sarcoma of bronchus
24. A 65-year-old woman presented to the hospital with the menopausal bleeding. At histological investigation
revealed in the curettage material from the mucosa of her cervix uteri revealed a new growth of atypical epithelium
with formation so-called " cancer pearls ". What is the most likely diagnosis?
A. *Squamous cell keratinous carcinoma
B. Adenocarcinoma
С Squamous cell nonkeratinous carcinoma
D. Mucous cancer
E. Nondifferentiated carcinoma
25. A post-mortern of a 48-year-old woman with a history of an operated stomach tumour in the past revealed
markedly enlarged, dense, whitish color ovary. Histo-logical investigation of the ovarian tissue showedutterly
atypical epithelial cells, placed among layers and cords of a connective tissue. What if the most likely disease?
A. *Krukenberg's carcinoma of ovary
B. Serous cystadenocarcinoma
C. Pseudomucinous cystcarcinoma
D. Malignant thecoma
E. Malignant granular cell tumor
26. A histological investigation of a lung biopsy revealed atypical cells which form plural acinar structures and
produce mucus. What histological form of a cancer of lungs takes place at the patient?
A. *High differentiated adenocarcinoma
B. Low differentiated adenocarcinoma
C. Nondifferentiated carcinoma
D. Differentiated
E. Glandular squamous cell carcinoma
27. A gastroscopy of a 37-year-old man revealed a tumorous formation 1,5 cm in diameter in a small curvature of
his stomach. What character of a tumor growth?
A. *Exophytic
B. Expansive
C. Invasive

62
D. Infiltrating
E. Endophytic
28. Histological investigation of a biopsy from a tumor of the right bronchus' mucous membrane revealed a cellular
and tissue atypia, appearance of structures in the form of cancer pearls '. Define the pathological process.
A. *Malignant tumor
B. Benign tumor
C. Hyperplasia
D. Metaplasia
E. Hypoplasia
29. A histological investigation of the biopsy from cervix uteri of a 45-year-old woman revealed signs of cellular
atypia with intact basal membrane. What is the most likely diagnosis?
A. * Carcinoma in situ
B. Erosion
C. Adenocarcinoma
D. Papilloma
E. Endometriosis
30. A physical examination of a 42-year-old patient revealed enlarged supraclavi-cular lymph nodes. A histological
investigation of a lymph node biopsy showed the metastasis of a signet-ring cancer. Choose the most probable
localization of a primary tumour.
A. *Carcinoma of stomach
B. Cancer of esophagus
C. Cancer of thyroid gland
D. Carcinoma of lungs
E. Carcinoma of uterine cervix
31. A female patient in a climacteric period presented with relapsing uterine bleedings. The diagnostic curettage of a
uterus is executed. Microscopical investigation showed among blood glandular elements of different size and forms
created by atypical cells with hyperchromatic nuclei numerous mitoses (including pathological). What is the most
likely pathology?
A. *Adenocarcinoma
B. Glandular hyperplasia of endometrium
C. Choriocarcinoma
D. Adenomatous polypus
E. Signs of the interrupted pregnancy
32. A bronchoscopy of a 65-year-old patient revealed a polyp-like formation, 1,0 cm in diameter, in the proximal
part of the upper lobe bronchus of his right lung. A histological research revealed the tumour which consisted from
fine lymphocyte-like cells with hyperchromic nuclei. The cells grew like layers and cords. Specify, what of the
listed below kinds of tumors is the most authentic?
A. *Nondifferentiated small cell carcinoma
B. Nondifferentiated large cell carcinoma
C. Squamous cell carcinoma
D. Adenocarcinoma
E Glandular squamous cell carcinoma
33. A gastroscopic study of a 50-year-old patient revealed a crater-like lesion on small curvature in pre-pyloric zone
of stomach. From a regional site of formation a biopsy is taken, биопсия. A histological investigation showed a
tumor with glands-like structures of the various form and the sizes, growing into surrounding tissue. The cells had
marked signs pf atypia. Name a histological variant of the presented tumor.
A. * Adenocarcinoma
B. Squamous cell carcinoma
C. Scirrhous carcinoma of stomach
D. Mucous carcinoma of stomach
E. Solid carcinoma of stomach
34. A patient presented to her physician with gradually developed plaque on a skin of a cheek. The formation had
necrosis and an ulcer in the center. A histological investigation of a skin biopsy revealed a growth of atypical
epithelial cells with a lot of pathological mitoses. What is the most likely diagnosis?
A. *Carcinoma of skin
B. Sarcoma
C. Papilloma
D. Trophic ulcer
E. Fibroma
35. A 48- year-old man with a history of a chronic bronchitis has died of a cachexy. A post-mortem revealed in a
lumen of the right bronchus an endophytic growth of a light- grey softish tissue. Microscopic investigation showed a
development of an atypical stratified epithelium with a presence of an « epithelial [epidermic] pearl, pearly body ».
What is the most likely diagnose.

63
A. *Squarnous cell keratinous carcinoma
B. Squamous cell nonkeratinous carcinoma
C. Adenocarcinoma
D. Nondifferentiated carcinoma
E. Apudoma
36. An autopsy of a 50-year-old man, who died of cancer intoxication, revealed the thickening of a stomach's wall to
1, 2 cm. The mucosa was fixed, with ill-defined gastric folds. On a cut view a tissue was homogeneous, whitish with
chondroid density. For what macroscopical form of a tumour the described changes are characteristic?
A. * Infiltrate
B. Node
C. Ulcer
D. Ulcerous-infiltration
E. Cyst
37. A histological investigation of a breast tumor's biopsy revealed solid layers of fine epithelial cells with
polymorphic nuclei and a considerable quantity of pathological mitoses. In addition, the tumor had very little stroma
and a lymphocytic infiltration. Name the kind of presented tumor.
A. *Medullary carcinoma
B. Scirrhous carcinoma
C. Paget's disease
D. Adenoma
E. Adenofibroma
38. A mucus membrane biopsy is taken from a bronchus of a 52-year-old patient. A histological investigation
revealed cords of atypical epithelial cells, which grew into the tissues underneath. In the cord's center a concentric
pink color fonnations were determined ("cancer pearls", "epithelial [epidermic] pearl, pearly body"). Name a kind of
a tumor?
A. *Squamous cell keratinous carcinoma
B. Differentiated adenocarcinoma
C. Squamous cell nonkeratinous carcinoma
D. Melanoma
E. Transitional cell carcinoma
Tumours derived from a mixture of tissues
1. A 55-year-old female is found to have asymptomatic microscopic hematuria. A plain radiograph of the abdomen
revealed a neoplasm at her right kidney. A gross investigation of removed kidney revealed a node 8cm in diameter
at its upper pole. The cut surface of a tumor presented with multiple hemorrhages and necroses. Histologically, it
consists of light cells that form alveolar .and papillary structures. The invasive growth of the tumor is moderate.
Many cells have hyperchromic nuclei and atypical mitoses figues. What is the most likely diagnosis?
A. *Renal cell carcinoma.
B. Clear cell adenoma.
C. Adenocarcinoma.
D. Nephroblastoma.
E. Acidophilic adenoma with malignancy.
2. A 60-year-old man noticed worsening urinary tract symptoms and thus inquires about screening tests for kidney
pathology. Investigation revealed a node 8 cm in diameter at right kidney's apex. After surgery, grossly tumor
presented a patchy pattern with hemorrhages and zones of necrosis. Histological study showed large anaplastic
tumor cells with abundant foamy cytoplasm and with small central densely staining nucleus. Abnormal mitoses were
also found. What is the most likely diagnose?
A. * Clear -cell carcinoma
B. Clear -cell adenoma
C. Adenocarcinoma
D. Nephroblastoma
E. Acidophilic adenoma with malignization
3. A 45-yer-old female examined by her gynecologist due to uterine bleeding. A neoplasm was diagnosed and then
removed at surgery department. The uterine lesion presented with huge necrotic and hemorrhagic spongy masses.
Microscopic investigation revealed large clear atypical epithelial cells and variety of abnormal dark cells, arranged
around material blood spaces. Stroma was not defined. Atypical cells resembled cytotrophoblast and
syncytiotrophoblast cells. What is the most likely diagnose?
A. *Choriocarcinoma
B. Invasive hydatidiform mole
C. Adenocarcinoma
D. Cavernous haemangioma.
E. Medullary carcinoma
4. A 32-year-old woman with a history of abortion 8 months earlier presented to the emergency department
complaining of uterine bleeding. Examination done by gynecologist revealed a neoplasm of the uterus. Grossly,

64
tumor had a spongy structure with multiple hemorrhages. Microscopically, atypical clear epithelial Langhan's cells
and abnormal syncytiotrophoblast cells arranged around maternal blood spaces were detected. What is the most
likely diagnosis?
A. *Choriocarcinoma
B. Squamous cell carcinoma without keratinisation
C. Adenocarcinoma
D. Fibromyoma
E. Hydatidiform mole
5. A 45-year-old woman presented to oncology with a breast tumor. A biopsy revealed a tissue atypia with stromal
predominance over neoplastic parenchyma. Breast ducts and ductules were variable, they had one or two layered
epithelium without atypical mitotic activity. The intralobular stroma was dence. What is the most likely diagnosis?
A. *Fibroadenoma
B. Papilloma
C. Non-invasive carcinoma
D. Invasive carcinoma
E. Mastitis
6. The patient on a face skin had a tumorous formation plaque-like form with an ulcer. What is the most likely
diagnosis?
A. * Basal cell epithelioma
B. Caremoid
C. Thecoma
D. Pinealoma C. Thymoma
7. A physical examination of a 22-year-old woman, with few years' history of rny-asthenia, revealed a big tumor at
anterior mediastinum. Histologically a tumor consisted of the oblong (spindle-shaped) cells with oval dark nuclei,
which form bands and nests and has little Hassall's bodies. After a surgical removal of a tumor symptoms of a
myasthenia have started to disappear progressively. Diagnose a tumor which has etioiogical and pathogenic
connection with a myasthenia.
A. * Thymoma
B. Adenoma of thyroid gland
C. Adenoma of parathyroid gland
D. Paraganglioma
E. Medulloblastoma
8. A 48-year-old man has presented to his physician a plaque-like formation on a neck. Histological investigation of
a skin biopsy revealed tumorous ceils located as nests, having round and oval form with narrow rim of basophilic
cytoplasm. They reminded cells of a skin basal layer. Specify the tumor name.
A. * Basal cell epithelioma
B. Epidermal cancer
C. Hidroadenoma
D. Trichoepithelioma
E. Syringadenoma
9. A physical investigation of a 25-year-old-woman revealed in her breast a dense node 1,0 cm in diameter. A
biopsy research showed encapsulated growth of a connective tissue round a basal membrane of a mammary gland
ducts. Glandular elements had different diameter, did not form lobes. What is the most likely diagnosis?
A. *Fibroadenama
B. Fibroma
C. Metastasis of a cancer
D. Adenoma
E. Fibrous cancer
10. A physical examination of a 39-year-old woman revealed a soaking area by her breast nipple, a superficial ulcer
with inflammatory hyperemia and skin edema. A histological research of a biopsy from this area revealed in basal
layer of thickened epidermis atypical big cells with light and optically empty cytoplasm, with absence of
intercellular bridges. Such cells are found and in the ostium of the big ducts of a gland. What is the most likely
diagnosis?
A. *Paget's desease
B. Intraductal cancer
C. Basal cell cancer
D. Epidermoid carcinoma
E. Melanoma
Mesenchymal (connective tissue) tumors
1. A 30-year-old man has a node on his left leg the skin. Physical investigation revealed dense, mobile tumor,
circumscribed by a connective tissue capsule. Grossly it cut section was presented with dense witish fibres.
Microscopically the tumor composed of mature fibroblasts and a collagenous stroma. What is the most likely
diagnosis?

65
A. * Fibroma
B. Myoma
C. Histiocytoma
D. Dermatofibroma
E. Desmoid.
2. A 14x6x5 cm neoplasm excised from a retroperitoneurn of a 66-year-old woman at surgery department.
Microscopic investigation revealed atypical anaplastic cells, which contained round cytoplasmic vacuoles of lipid
that scallop the nucleus. The majority of cells were pleomorphic, some of them were round with chromosomal
abnormalities. What is the most likely diagnosis?
A. *Liposarcoma
B. Lipoma
C. Myosarcoma
D. Fibrosarcoma
E. Mesothelioma
3. A physical examination of 47-year-old woman of gynecology department revealed that her uterus contained
dicrete, firm, white nodules. Histological examination excised lesion demonstrated a tissue atypia of a sample. It
presented well-defferentiated mature cells of smooth muscules. What is the most likely diagnosis?
A. *Leiomyoma.
B. Carcinoma of the uterus.
C. Fibromyoma.
D. Chorionepithelioma.
E. Leiomyosarcoma
4. A 20-year-old man has had a slowly growing reddish nodule on his upper lip. He finally decides to have a
surgeon remove it. Microscopically the nodule is composed of benign varying sized tiny blood vessels. What is the
most likely diagnosis?
A. * Capillary hemangioma.
B. Venous hemangioma.
C. Cavernous hemangioma.
D. Hemangiopericytoma.
E. Glomus-angioma.
5. The patient, a previously dealthy man of 25 years, presented with a painless neoplasm in a soft tissues of his left
thigh. Grossly, a tumor had uneven boundaries and on cut section it looked like a fish flesh. Microscopic
investigation revealed immature connective tissue cells with pleomorphism, numerous mitotic figures and lym-
phocyte infiltration at the edge of the tumor. What is the most likely diagnosis?
A. *Fibrosarcoma
B. Myosarcoma
C. Fibroma
D. Carcinoma
E. Myoma
6. A 40-year-old woman presented with a very slowly enlarging subcutaneous mass at the right side of the chest
wall. Physical examination revealed a soft lobulated fluctuant swelling, not attached to the skin or underlying
muscle. Histologically a neoplasm was well-encapsulated and consisted of mature cells with clear cytoplasm that
varied considerably in size. What is the most likely diagnosis?
A. *Lipoma.
B. Fibroma.
C. Hygroma.
D. Papilloma.
E. Hemangioma.
7. A macroscopical investigation of operatively removed uterus revealed a tumour with a soft consistence,
hemorrhages and necroses. The tumor cut surface reminds ' the fish meat '. Histological research has found an
expressed cellular and tissue atypia; there were cells with pathological mitoses figures. What is the most likely
diagnosis?
A. Sarcoma.
В Adenocarcinoma.
C. Angioma.
D. Fibroma.
E. Lipoma.
8. At the young man in a skin depth the dense, mobile tumour, is defined. A microscopical research revealed
chaotically located fascicles of collagen fibers with a small amount of spindle-shaped cells. What tumour is
removed?
A. *Dense fibroma.
B. Leiomyoma.
C. Melanoma.

66
D. Lipoma.
E. Glomus-angioma.
9-А 4-year-old child presented with a flat red color knot on his neck skin, which turns pale at pressing by glass the
knot. What is the most probable diagnosis?
A. Hemangioma
B. Pigmented nevus
C. Melanoma
D. Leiomyoma
E. Lymphangioma
10. A 28-year-old man with a history of an elbow bruise 3 years ago presented with a tumorous growth in the area of
an epiphysis of a humeral bone. The formation did not have accurate borders. A histological investigation of biopsy
material revealed a considerable quantity of polymorphic cells of osteoblastic type with numerous pathological
mitoses. Make the presumable diagnosis.
A. *Osteosarcoma
B. Chondrosarcoma
C. Osteoid-osteoma
D. Fibrosarcoma
E. Sinovial sarcoma
11. A 48-year-old man presented to physician with mobile 1,0x0,7 cm formation under a skin of mandible. It had
precise borders, dough-like consistence and slow growth. A histologic research of formation revealed fat tissue cells
(lipocytes), which created lobules of different forms and the sizes, divided by thin layers of a connective tissue with
vessels. What is the most likely diagnosis?
A. *Lipoma
B. Fibroma
C. Angioma
D. Liposarcoma
E. Fibrosarcoma
12. A 33-year-old woman presented with a tumor-like formation on a white line of her abdomen, which during
pregnancy has started to increase in sizes. A histological research revealed that a tumor is constructed of the
differentiated connective tissue, in which collagen fibers prevail of cells. What tumor presented in this case?
A. * Desmoid
B. Dense fibroma
C. Fibrosarcoma
D. Soft fibroma
E. Dermatofibroma
13. A 15-year-old young man presented with a tumorous formation in the central site of his wrist bone. The node
grew slowly within last 3 years. A histologica! research of a removed neoplasm revealed mature chondrocytes
without mitoses, which randomly located in chondral lacunas. Cartilages of a capsule had different form and the
sizes due to variable quantity of chondral cells, between which there were basic substance with liquid layers of a
connective tissue. What is the most likely diagnosis?
A. *Chondroma
B. Chondroblastoma
C. Chondrosarcoma
D. Teratoma E.
14. A 50-year-old patient presented to the doctor with a ball-shaped, dense, motionless neoplasm, 2 cm in diameter,
under a skin in the right parietal site of his head. A histological research of a removed neoplasm revealed a chaotic
osteal beams pattern with a connective tissue between. What is the most likely diagnosis?
A. *Cancellous osteoma (Osteoma spongiosum)
B. Compact osteoma (Osteoma durum)
C. Osteoporosis
D. Osteomalacia
E. Osteosarcoma
15. At the young woman in the area of her distal extremity of a femur the tumor, which quickly grew, is removed.
Grossly, it had a motley pattern - from white-sulphur to brown-red color and a quaggy consistence. Microscopical
investigation revealed the basic tissue component of a tumor presented with osteal and the ossi-form structures
covered by atypical osteoblasts, with numerous thin-walled vessels and atypical mitoses figures. Make a diagnosis.
A. *Osteocarcoma
B. Chondroma
C. Osteoma
D. Саркома Юинга
E. Angiosarcoma
16. A 16-year-old child presented with a painful sottish node in his femur diaphy-sis. The formation grows quickly
with destruction of a spongy layer of a bone. Microscopical investigation revealed monornorphic round cells little

67
bit bigger than mature lymphocytes, with jejunely light cytoplasm which contains glycogen. In some zones these
cells form pseudo- rosettes with few mitoses. Between cells there are fibrinous membranes. What is the most likely
diagnosis?
A. *Ewing's sarcoma
B. Limphoma
C. Rhabdomyosarcoma
D. Neuroblastoma
E. Hemangioma
Neoplasms of the nervous system (including meninges) and melanin producing tissues
1. An eye of 53-year-old patient, excised at surgery due to neoplasm, presented in pathology department. Gross
investigation reveled a 1 *0,4 cm black lesion in the retina. Microscopic appearance of a tumor was characterized by
nests of immature cell with eccentric nuclei, prominent macronucleoli and cytoplasm brown pigment. What is the
most likely diagnosis?
A. *Melanoma
B. Neurinoma
C. Angiosarcoma
D. Neuroblastoma
E. Ganglioneuroblastoma
2. At autopsy a 8-year-old child is found to have a. poorly circumscribed tumor of cerebellum. Histologically
investigation reveled crowds of small immature cells with hyperchromatic, round-oval nuclei and scant cytoplasm.
A few rosettes were also found by pathologist at slide examination. What is the most likely diagnosis in that case?
A. *Medullob!astoma
B. Astrocytoma
C. metastasis of cancer
D. metastasis of sarcoma
E. Glioblastoma
3. An elderly man with a 2-year history of right leg amputation presents with 2 cm encapsulated tumor at the area of
a past surgical trauma of soft tissue. Histology investigation revealed disordered orientation of mature nerve fiber
bundles inter mixed with connective tissue. Higher magnification showed bundles of axons, Schwann cells,
fibroblasts and perineuria! cells within tumor mass. What is most likely diagnosis?
A. Neurinoma
B. Neurofibroma
C. Malignant neurinoma
D. Soft fibroma
E. Fibrosarcoma
4. A 66-year-old woman present with right eye bad vision, ophtalmoscopy revealed a neoplasm of retina which was
soon excised, at surgery together with an eye ball. Grossly, a neoplasm was soft, irregular in contour 1x1 cm in size
and had a brown coloring. Under microscope a lesion demonstrated nodular aggregates of infiltrating cells. There
cells contained large brown pigment nuclei with chromatin clumped at the periphery of nuclear membrane and
prominent nucleoli. Atypical mitoses figures were also revealed. What is most likely diagnosis?
A. *Melanoma
B. Schwannoma
C. Glomus tumor
D. Paraganglioma
E. Angiosarcoma
5. A 38-year-old woman has a seizure while shopping and is taken to the hospital. A scan of a brain demonstrated a
poorly circumscribed 5 cm tumor at right parietal lobe. A biopsy of this area contains of increased number of glial
cell nuclei and an interening feltwork of file cell processes that give the background a fibril lary appearance.
Abnormal mitoses and atypical central nervous system cells were absent within biopsy sample. What is the most
likely diagnosis?
A. Astrocytoma
B. Oligodendroglioma
C. Ganglioneuroma
D. Ependimoma
E. Chorioid papilloma
6. A 50-year-old woman presented with a pigmented painful skin lesion above the ankle. It had been present for
many years but in recent months it had enlarged quite rapidly, its outline got irregularity. Microscopic investigation
of a lesion biopsy revealed nests of atypical cells and single cells with eccentric nuclei, prominent macronucleoli
and cytoplasmic brown pigment. What is the most likely diagnosis?
A. * Melanoma
B. Basal cell carcinoma
C. Hemangioma.
D. Haematoma.

68
E. Carcinoid.
7. The man of 45 years has completely lost hearing on the right ear. Physical investigation has not revealed any
pathological changes in system of the right acoustic analyzer. The tomography of a brain has revealed a neoplasm, 7
cm in diameter, homogeneous, without precise contours in a site of cerebellopontine angle. Name a neoplasm.
A. *Neurilemmoma
B.Gangliocytoma
C. Astrocytoma
D. Chorioidal papilloma
E. Neuroblastoma
8. The tumor of a brain is diagnosed for the patient with neurologic disturbances. During operation the tumor,
which looks like the dense node bound to a firm cerebral membrane, is removed. Histological investigation revealed
the neoplasm constructed from endothelium-like cells closely adjoining to each other. What is the most likely
diagnosis?
A. *Meningioma
В. Glioblastoma
C. Anaplastic meningeoma
D. Astrocytoma
E. Neuroblastoma
9. The tumour of a brain is diagnosed for the patient with quickly increasing intrac-ranial hypertension. The
removed tumor of a parietal -temporal part of brain had a soft consistence and a motley pattern of a cut surface.
Histological investigation revealed neoplastic tissue constructed of polymorphic cells with sites of necroses and a
hemorrhage. What is the most likely diagnosis?
A. * Glioblastoma
B. Oligodendroglioma
C. Neuroblastoma
D. Astrocytoma
E. Meningioma
10. The woman on her face skin had a pigmental formation in the form of a nodule which quickly grew. The biopsy
is made. Microscopical investigation of biopsy sample revealed fields of spindle-shaped and polymorphic cells
which contain a brown pigment. In addition, there were diagnosed numerous mitoses. What is the most likely
diagnosis?
A. *Melanoma
B. Nevus
C. Cancer
D. Papilloma
E. Dermatofibroma
11. A physical investigation of a 60-year-old woman with one year history of a formation on a face revealed a
brown skin plaque with irregular form and black impregnations. A histological research of a skin biopsy showed in
epidermis and through all derma polymorphic big cells with pathological mitoses, large nucleoli and yellowy-brown
pigment in a cytoplasm of many cells. The specified cells grow in a kind of fine groups and also alone. Make a
diagnosis.
A. * Melanoma
B. Nevus
C. Papilloma
D. Xeroderma
E. Melanosis
Diseases of haemopoetic and allied systems
1. A 34- year-old male is found to have weakness confusion, other neurologic manifestation and polyuria X-ray
examination revealed multifocal destructive bone lesions throughout the skeletal system. Electrophoretic analysis
revealed increased level of Ig in the blood and Bence Jones protein in the urine. What is the most likely diagnosis?
A.*Myeloma
B. Acute monocytic leukemia
C. Chronic myeloleukemia
D. Lymphogranulomatosis
E. Histiocytosis
2. A 42-year-old female is admitted to the hospital for treatment a blood disorder. A disease complicated with
pneumonia led to lethal outcome. An autopsy revealed hyperplasia of a bone marrow with "pus" - like appearance,
splenomegaly (5 kg weight), hepatomegaly (6 kg weight), systemic lymph nodes enlargement. What is the most
likely diagnosis?
A* Chronic myeloleukemia
B. Chronic lymphatic leukemia
C. Myeloma
D. Polycytemia vera (erythremia, Osier's disease)

69
E. Lymphogranulomatosis
3. An elderly male is admitted to the hospital for treatment of humeral bone fracture. His fracture's area X-ray
showed a new growth and lytic zone within lesion. Histological examination of a biopsy revealed abnormal plasma
cells. What is the most likely diagnosis?
A. * Myeloma
B. Chronic osteomyelitis
C. Chondrosarcoma.
D. Bone fibrous dysplasia
E. Metastasis of adenocarcinoma
4. At autopsy an elderly female is found to have enlarged groups of a neck, axillary and mediastinal lymph nodes
matted together. They were firm and rubbery. The cut surface was gray-white, producing a "fish-flesh" appearance.
Microscopy revealed heterogeneous cellular infiltrate wich contained lymphocytes, classic and mononuclear Reed-
Berezovsky-Sternberg cells. What is the most likely diagnosis?
A. * Lymphogranulomatosis
B. Chronic lymphatic leukemia
C. Lymphosarcoma
D. Retikulosarcoma
E. Mycosis fungoides
5. A 67-year-old female presented with painless enlargement of lymph nodes. Histological examination of a biopsy
sample revealed that the whole lymph node pattern was unclear, with heterogeneous cellular infiltrate. It included
abnormal, immature cells admixed with lymphocytes, eosinophils, plasma cells and macro-phages. Binucleated
Berezovsky-Reed-Sternberg cells, where surrounded by multiple cell types. What is the most likely diagnosis?
A. * Lymphogranulomatosis
B. Acute myeloleukemia
C. Chronic myeloleukemia
D. Mycosis fungoides
E. Tuberculosis
6. A 25-year-old patient presented with peripheral adenopathy, involving a single cervical lymph node. A biopsy
sample investigation revealed an unclear node pattern with mixed cellular infiltration. Histologycal study under
higher magnification showed numerous variants of Berezovsky-Reed-Stenberg cells, lymphocytes, ne-crotic area
and mild diffuse fibrosis. What is the most likely diagnosis?.
A. * Lymphogranulomatosis
B. Nodular lymphoma
C. Burkitt's lymphoma
D. Lymphocytic lymphoma
E. Chronic lymphatic leukemia
7. A 65-year-old female is noted to be anemic. He serum protein electrophoresis demonstration a large monoclonal
Ig G kappa protein. In her bone marrow are increased numbers of atypical plasma cells. Her skull X-ray show
multiple lytic areas. What is the most likely diagnosis?
A.* Myeloma
B. Polycytemia vera (erythremia, Osier's disease)
C. Lymphocytic lymphoma
D. Chondrosarcoma
E. Bone fibrous dysplasia
8. A thoracotomy of a 58-year-old woman revealed in her anterior mediastinum the enlarged and soldered together
lymph nodes. Microscopical investigation revealed atypical cells with predominance of Hodgkin cells and giant
Reed—Berezovsky -Sternberg cells. A sclerosis was absent. What is the most likely diagnosis?
A. *Lymphogranulomatosis with low-spirited development of lymphoid tissue
B. Lymphogranulomatosis with predominance of nodularis sclerosis
C. Lymphogranulomatosis with predominance of lymphoid tissue
D. Lymphosarcoma
E. Mixed-cellular variant of lymphogranulomatosis
9. A tomography revealed enlarged lymphatic nodes. A histological investigation of lymph node's biopsy showed a
circular growths of a connective tissue, which surrounded a granuloma - like formation, made from lymphocytes,
plasmocytes and giant double- nuclear cells. What is the most likely diagnosis?
A. *Lymphogranulomatosis
B. Lymphosarcoma
C. Tuberculosis
D. Sarcoidosis
E. Lymphatic leukemia
10. The young man presented to his physician with enlarged neck lymph nodes. A microscopical investigation of a
lymph node biopsy sample revealed the lymphoid tissue proliferation with presence of a giant Reed—Berezovsky -
Sternberg cells, eosinocytes, zones of necrosis and sclerosis. What is the most likely diagnosis?

70
A. *Lymphogranulomatosis
B. Chronic lymphatic leukemia
C. Multiple myeloma
D. Lymphosarcoma
E. Histiocytosis
11. A biopsy of the enlarged lymph node was taken. A histological investigation revealed a diffuse growth of
lymphoid cells with adding of eosinocytes, atypical histiocytes, solitary giant Reed—Berezovsky - Sternberg cells
with two and more nuclei, cell's necrosis and sclerosis. What is the most likely diagnosis?
A. * Lymphogranulomatosis
B. Lymphatic leukemia
C. Burkett's lymphoma
D. Sarcoidosis
E. Myeloleukemia
12. An autopsy of a 67-year-old man revealed the systemic enlargement of lymph nodes with formation of
tumorous conglomerates. The spleen was also enlarged with a motley pattern of a cut surface. There were plural,
tiny, yellowish-white spots on a red background of a spleen's pulp. What is the most likely diagnosis?
A. * Lymphogranulomatosis
B. Sarcoidosis
C. Lymphosarcoma
D. Carcinoma of lung
E. Lymphatic leukemia
13. Clinical investigation of a patient revealed the enlarged lymph nodes, spleen and liver. A microscopical study of
the enlarged cervical (neck) lymph node showed the blurring of its structures and absence of lymphatic follicles. All
microscopical views were presented by cells with round nuclei and narrow ring of a ba-sophilic cytoplasm. What is
the most likely diagnosis?
A. *Lymphatic leucosis
B. Lymphogranulomatosis
C. Lymphosarcoma
D. Myeloleukemia
E. Multiple myeloma
14. An autopsy of a 35-year-old woman revealed the enlarged spleen, (weight SOOgm), liver (weight 4000 gm) and
lymph nodes. A bone marrow of a femur diaphysis was juicy (succulent), crimson-red color. Microscopical study of
a liver defined dense infiltrates within portal tracts, consisted of immature blood cells. These cells had a round
nuclei and narrow ring of a cytoplasm. What is the most likely diagnosis?
A. *Chronic lymphatic leukemia
B. Chronic myeloid leucosis
C. Generalizated form of lymphogranulomatosis
D. Acute myeloblastic leucosis
E. Acute lymphoblastic leucosis
15. A microscopic investigation of the enlarged neck lymph node biopsy revealed the blurring of its structures,
plenty of proliferating lymphocytes with adding of solitary giant Reed-—Berezovsky - Sternberg cells. What is the
most likely diagnosis?
A. *Lymphogranulomatosis, with predominance of lymphatic tissue
B. Lymphogranulomatosis, with exhaustion of lymphatic tissue
C. Mixed cell variant of lymphogranulomatosis
D. Lymphosarcoma
E. Nodular sclerotic variant of lymphogranulomatosis
16. At young men the increased cervical lymph node is removed. Microscopic investigation revealed the altered
node's structure, an absence of lymphoid follicles, sites of a sclerosis and necrosis. The cellular infiltrate is
polymorphic with a presence of lymphocytes, eosinocytes, and atypical one-nuclear cells and multinuclear giant
cells (Reed—Berezovsky - Sternberg cells). What is the most likely diagnosis?
A. * Lymphogranulomatosis
B. Acute lymphatic leukemia
C. Chronic lymphatic leukemia
D. Burkett's lymphoma
E. Mycosis fungoides
17. A liver biopsy was taken from a 66-year-old man, with a history of increased quantity of lymphocytes and pro-
lymphocytes in his blood. A histological investigation of a liver sample revealed plural accumulations of the
mentioned above cells, mainly between hepatic segments. For what disease above listed changes are characteristic?
A. *Chronic lymphatic leukemia
B. Acute lymphatic leukemia
C. Lymphogranulomatosis
D. Chronic persistence hepatitis

71
E. Hepatocellular carcinoma of liver
18. A 4-year-old girl died due to a post-hemorrhagic anemia, resulted from a gas-tro-intestinal profuse bleeding. An
autopsy revealed an anemia of her organs, the enlargement of the different groups of lymph nodes, thymomegaly,
mild hepatomegaly, splenomegaly and bright red bone marrow. Microscopical study showed hyper cellularity of a
bone marrow with monomorphic blast cells infiltrate, diffuse inflammatory tumor-like infiltrates in a liver, a spleen,
lymph nodes, meninges and substance of a brain. What is the most likely diagnosis?
A. * Acute lymphoblastic leucosis
B. Acute myeloblastic leucosis
C. Acute nondifferentiated leucosis
D. Acute monoblastic leucosis
E. Acute plazmoblastic leucosis
19. A 14-year-old boy presented to a hospital with enlarged subrnaxillary and cervical lymph nodes. A biopsy
procedure was performed. Microscopical investigation revealed the disorder of a lymph node typical structure, a
heterogeneous cellular population with a presence of giant multinuclear cells and plural one-nuclear big cells. There
were also eosinocytes, neutrophils and lymphocytes in the cell infiltrate. In addition, sites of necrosis and sclerosis
were found. What is the most likely diagnosis?
A. *Lymphogranulomatosis
B. Hyperplasia of lymphatic nodes
C. Granulomatous lymphadenitis
D. Purulent lymphadenitis
E. Non-Hodgkin's lymphoma
20. A radiological investigation of a man's head revealed in his maxillary and mandibular bones numerous round
defects with smooth walls. A histological study showed osteolysis and osteoporosis accompanied with insufficient
bone repair. The laboratory test of the urine detected the Bence Jones protein. What is the most likely diagnosis?
A. *Multiple myeloma
B. Chronic myeloleukemia
C. Chronic erythromyelosis
D. Acute myeloleukemia
E. Acute nondifferentiated leucosis
21. A physical examination of a young men's oral cavity revealed the atrophy of mucous membrane and red spots
on his tongue (atrophic; Hunter's; Moeller's glossitis). Sclera had a yellow coloring. A blood test showed the color
index above one. For what anemia these changes are characteristic?
A. * Nutritional anemia due to vitamine B,2 deficiency
B. Asiderotic anemia
C. Acute posthemorrhagic
D. Chronic posthemorrhagic
E. Hemolytic anemia
22. A physical examination of a 42-year-old man revealed enlarged lymph nodes. A histological investigation of a
lymph node showed lymphocytes, histiocytes, re-ticular cells, small and big Hodgkin's cells, multinuclear
Berezovsky -Reed-Sternberg cells (Sternberg-Reed cells) infiltration with solitary necrotic areas. What disease such
changes characteristic for?
A. * Lymphogranulomatosis
B. Lymphosarcoma
C. Chronic leucosis
D. Acute leucosis
E. Metastasis of carcinoma of lungs
23. A gross examination of a dead body revealed the skin's pallor and a yellowness of a sclera. Livores mortis were
not defined. The volume of blood in a heart and large vessels was reduced. A blood looked aqueous, hi a skin,
mucosa and serous membranes there were petechial hemorrhages. The internal organs, especially a spleen, a liver
and kidneys had a rusty color on a cut. A bone marrow of flat bones was a crimson-red and succulent In cortical
[tubular, cylindrical] bones it looked like a crimson jelly. Name the disease, connected with a deficiency of vitamin
В12.
A. * Pernicious anemia
B. Drepancytic (sickle-cell) anemia
C. Panmyelophthisis
D. Toxic anemia
E. Acute posthemorrhagic anemia
24. A post-mortem of a 56-year-old woman revealed the Hunter's (atrophic; Moeller's) glossitis, atrophy of mucous
membrane of a stomach and liver's hemosidero-sis. A bone marrow in all investigated bones was red. A
microscopical study showed hyper cellular infiltration in a lamina propria of a stomach with a presence of lymphatic
follicles. In a spinal cord there was a funicular myelosis and also haemopoiesis foci detected in a spleen. What is the
most likely diagnosis?
A. *Addison-Biermer anemia

72
B. Fanconi's (congenital aplastic) anemia
C. Hypoplastic anemia
D. Chronic gastritis (type A)
E. Chronic gastritis (type B)
25. A post-mortem of a 4-year-old girl revealed plural petechial hemorrhages on her skin, serous and mucous
membranes, large focal hemorrhage in a brain and necrotic tonsillitis. Microscopical study showed multiple cell
infiltrates with prevailing lymphocytes in a bone marrow, a liver, a spleen, a thymus, lymph nodes, tonsils and a
skin. What is the most likely diagnosis?
A. *Acute lymphatic leukemia
B. Chronic lymphatic leukemia
C. Hodgkin's lymphoma
D. Follicular non-Hodgkin's lymphoma
E. Mycosis fungoides
26. A post-mortem of a 15-year-old girl revealed enlarged neck, mediastinal and mesenteric lymph nodes, which
were integrated in conglomerates. On a cut section, the tissue pattern of the nodes was non-uniform, with foci of
necrosis. Microscopical investigation showed the uneven structure of lymph nodes, foci of sclerosis and necrosis.
The cell population was also heterogeneous and included uninuclear atypical cells, giant multinuclear atypical cells,
a significant amount of eosinocytes and neutrophils, and sparse lymphocytes. What is the most likely form of
Hodgkin lymphoma (lymphogranulomatosis)?
A. * Lymphogranulomatosis, mixed cell variant
B. Lymphogranulomatosis, lymphohistiocytic variant
C. Lymphogranulomatosis, nodular sclerosis
D. Lymphogranulomatosis, variant with low-spirited development of lymphoid
tissue
E. Hodgkin's sarcoma
27. A 63-year-old man, with 20 years history of working as the engineer for the service of electronic microscopes,
died of a sepsis. An autopsy revealed plural hemorrhages in serous and mucous membranes, a general
hemosiderosis, a fatty dystrophy of a myocardium, liver and kidneys, ulcerative -necrotic and purulent processes in
a gastro -intestinal system. The red bone marrow was replaced by a fatty. What is the most likely diagnosis?
A. *Hypoplastic anemia
B. Megaloblastic anemia
C. Posthemorrhagic anemia
D. Iron deficiency anemia
28. A 38-year-old man, with a history of an ulcer, resulted in a stomach's resection, in his blood test had a normal
quantity of erymrocytes, but reduced hemoglobin's concentration and decreased color index. An autopsy revealed
pale skin and visible mucous membranes; bone marrow of long tubular bones was brightly red. Erythro-cytes in a
smear had a normal form and the sizes. They look very pale because of bad staining by dyes. What pathological
process took place in this case?
A. * Hypochromic iron deficiency anemia
B. B12-folic acid deficiency anemia
C. Acute lymphoblast leukemia
D. Sicklemia sickle cell anemia
E. Aplastic anemia
29. A 44-year-old man presented to a gastroenterologist with pains in his epigastrium. A physical examination
revealed an icteritiousness of his skin and scleras, an alteration of a tongue's mucous membrane. A tongue grossly
looked shining, smooth, with red spots. In peripheral blood's smear there were found enlarged erythrocytes
(megaloblasts). A histological study of a gastrobiopsy from a body of a stomach showed a thinning of mucosa, a
reduction of glands quantity, superfluous growth of a connective tissue. Specify, what of diagnoses is the most
probable in this case:
A. * B12-folic acid deficiency anemia
B. Chronic posthemorrhagic anemia
C. Hemolytic anemia.
D. Chronic myeloid leukemia.
E. Aplastic anemia
30. A man, with a history of getting a high doze of ionizing radiation, presented to his physician with marked
stomatorrhagia (gingival hemorrhage), spontaneous skin and mucosas' haemorrhages. A blood test showed a
normochromal anemia and pancytopenia. The concentration of iron in blood's serum was normal. A histological
investigation of a bone marrow puncture sample revealed a replacement of a hemopoietic tissue by the fatty tissue.
What is the most likely diagnosis?
A. * Aplastic anemia
B. B12-fo!ic acid deficiency anemia
C. Hemolytic anemia
D. Myelodysplastic syndrome

73
E. Immune cytopenia
31. A patient presented with an infiltrative, plaque-like polymorphic skin rash, which had various contours, sizes
and a congested -cyanotic color. The lesions tended to peripheral growth and fusion. A microscopical investigation
of a skin biopsy revealed massive lymphocytes' proliferation, which occupied the entire derma and a hypodermic
fatty layer. What is the most likely diagnosis?
A. *Limphomaof skin
B. Systemic lupus erythematosus
C. Mycosis fungoi'des
D. Intradermal nevus

ATHEROSCLEROSIS
1. A 46-year-old male suddenly died after developing a heart failure. An autopsy revealed in abdominal aorta the
yellow color areas, which did not rise over its surface. Histological investigation of aorta showed the aggregations of
cells with a foamy cytoplasm among smooth mussels and macrophages. These cells had a motley-orange coloring,
when stained by a Sudan Ш. For what stage of an atherosclerosis such picture is characteristic?
A. *Lipoidosis
B. Liposclerosis
C. Atheromatosis
D. Ulceration
E. Atherocalcification
2. A post-mortem of an elderly male revealed microscopic alteration of his coronary artery. There was narrowing
of vessel's lumen due to a fibrous plaque with some admixture of lipids. Name the stage of atherosclerosis:
A. *Liposclerosis
B. Lipoidosis
C. Pre-lipoidosis
D. Atheromatosis
E. Atherocalcification
3. At autopsy of 63-year-old male revealed an atherosclerosis of the brain's arteries and a thrombosis of the internal
carotid artery's branch. Gross investigation showed a focus of moist softening in his brain's tissue. Define the
pathological process in the brain.
A. * Ischemic infarction
B. Hemorrhagic infiltration
C. Hematoma
D. Encephalitis
E. Tumour of the brain.
4. At autopsy of an elderly female an aorta tissue sample collected for histology. Microscopic investigation
revealed in aorta's intima the accumulation of xanthoma's cells. At what disease such morphological picture
is possible?
A. * Atherosclerosis
B. Hypertension
C. Syphilitic mesaortitis
D. Nonspecific aortoarteritis
E. Nodular periarteritis
5. A 54-year-old male died after developing a heart failure. An autopsy revealed a chronic venous hyperemia of the
internal organs, hypertrophy of the left ventricle of a heart and focal cardiosclerosis. Gross investigation of aorta
showed yellow-white plaques in the intima with fine-grained masses in their centers, which infiltrated the aortal
wall. Give the name of this pathological process.
A. * Atheromatosis
B. Lipoidosis
C. Liposclerosis
D. Arteriolosclerosis
E. Calcinosis.
6. A 66-year-old male suddenly died on his way to operation room. An autopsy revealed a hemorrhage into
retroperitoneal fat, a saccular dilatation of the abdominal aorta wall with rupture in arterial wall. The aorta's
defect had uneven edges and stony hardening of surrounding tissues. Name the most likely disease, resulted in
described complication?
A. * Atherosclerosis
B. Hypertension
C. Systemic vasculitis
D. Visceral Syphilis E.

74
7. At autopsy of 52-year-old male revealed yellowish areas at his aorta's intima, this did not bulge above its
surface. Histological investigation showed the accumulation of cells with foamy cytoplasm, stained by Sudan III
in yellow color. Name the most likely stage of atherosclerosis in aorta?
A. *Lipidosis
B. Liposclerosis
C. Atheromatosis
D. Pre-lipoidosis
E. Atherocalcification
8. A post-mortem of a 65-year-old male revealed yellowish streaks and spots in the abdominal aorta's intima, which
did not rise above endothelial surface. These areas were stained by Sudan III in orange color. What stage of
atherosclerosis such changes are characteristic for?
A. *Lipidosis
B. Liposclerosis
C. Atheromatosis
D. Ulceration
E. Atherocalcification
9. A 53-year-old female died from acute myocardial infarction. An autopsy revealed multiple whitish dense
plaques in her coronal arteries' intimae, which bulged in and narrowed the vessel's lumen. Name the most likely
stage of a coronary atherosclerosis?
A. * Liposclerosis
B. Lipoidosis
C. Atheromatosis
D. Atherocalcification
E. Ulceration
10. A 45-year-old male died in a traffic accident. A histological investigation of his aorta revealed a lipid infiltration
of its intima with fats accumulation in myocytes and macrophages of aortal medial layer. Name the stage of
atherosclerosis?
A. *Lipoidosis
B. Liposclerosis
C. Atheromatosis
D. Pre-lipoidosis
E. Atherocalcification
HYPERTENSION
1. An elderly male, with 15 years history of essential hypertension, died from renal failure. What would be the most
likely gross picture of his kidneys at autopsy?
A. *Small, dense, a surface is fine-grained.
B. Large pied and soft.
C. Large red
D. Large white and smooth
E. Large with the plural thin-walled cysts.
2. A 52-year-old male had a long history of hypertension. He presented to emergency care physician with
complains of an acute, persisted for few hours rise of a blood pressure. What is the most likely alteration of the
arterioles' walls due to hypertonic crisis?
A. *Fibrinous necrosis
B. Hyalinosis
C. Sclerosis
D. Amyloidosis
E. Calcinosis
3. An elderly woman, with a long history of bronchial asthma, ischetnic heart disease and hypertension, presented
with hypertrophy of her heart's left ventricle. Name the cause of the heart alteration?
A. * Arterial hypertension
B. Pulmonary insufficiency
C. Chronic ischemia of cardiac muscle
D. Bronchial asthma
E. Emphysema of lights
4. An elderly male, with 20 years history of hypertension, died from uremia. An autopsy revealed a heart
hypertrophy and diffuse cardiosclerosis. There were also small, dense kidneys with granularity of their surface.
Histological investigation of kidneys tissue showed the collapse of glomerular's arterioles and sclerosis. Some
glomeruli were replaced with pinkish homogenous masses, negative to the Kongo-red staining. Tubules were
atrophic. Name the kidney pathology?
A. * Primary-scarring kidneys
B. Secondary - scarring kidneys
C. Amyloidosis

75
D. Chronic glomerulonephritis
E. Chronic pyelonephritis
5. A 63-years-old patient, with a history of hypertensive disease, died from cardiac insufficiency. At autopsy, the
heart enlargement and dilated ventricular cavities were revealed. Microscopical investigation showed marked
hypertrophy of cardiomyocytes, with their fatty dystrophy and hyperchrotnic barrel-like nucleuses. What
pathological process is the most likely in a heart?
A * Eccentric hypertrophy
B. Myocarditis
C. Concentric hypertrophy
D. Angiogenic cardiosclerosis
E. Cardiomyopathy
6. A 64-years-old patient died from the brain's hemorrhage. An autopsy revealed markedly decreased kidneys
(6x3x2cm in sizes and 60, 0 grams weight). They were dense, anemic with an even, fine-grained surface. On a cut,
there was a constant thinning of the kidneys' cortex. Changes in kidneys are the implication of:
A. * Arteriolosclerotic nephrosclerosis
B. Atherosclerotic nephrosclerosis
C. Secondary- scarring kidney
D. Amyloidal- scarring kidney.
E. Gouty kidneys
7.-A 64-years-old male died from uremia. At autopsy revealed reduced in sizes kidneys, with 50.Og weight. Their
surfaces were regularly granulated; the cortexes were thin. A microscopic study showed considerably thickened
walls of glomeralar arterioles due to deposition of homogeneous unstructured pink masses. The lumens of these
vessels were markedly narrowed; nephrons were diminished with sclerotic changes, tubules were atrophic. For what
disease the described changes are characteristic?
A. * Hypertension
B. Chronic glomerulonephritis
C. Amyloidosis of kidneys
D. Pyelonephritis with scarring of kidneys
E. Acute necrotic nephrosis
8. A 65-year-old male long time was ill by hypertensive disease and died from chronic kidney insufficiency. The
autopsy showed, that both kidneys are considerably decrease in sizes, their surfaces are granulated; histologically -
most glomeruluses are containing hyaline, part of them in sclerotic condition, other in hyperplasia; in stroma - the
fields of sclerosis, arteriolo- and arteriosclerosis, elastofibrosis of large kidney arteries branches. What is the name
of the exposed changes?
A. * Arteriolosclerotic nephrosclerosis
B. Atherosclerotic nephrosclerosis
C. Secondary-restricted kidney
D. Chronic pyelonephritis
E. Amyloidal- scarring kidney.
9. An autopsy of 48-year-old patient, who died from the complications of hypertensive disease, revealed small,
dense kidneys with fine-grained surface. Parenchyma and cortex matter were atrophied. Give the name for such
kidneys.
A. * Primary-scarring kidneys
B. Amyloidal-scarring kidneys
C. Secondary-scarring kidneys
D. Pyelonephritic-scarrmg of kidneys E.
10. An autopsy of an elderly woman, with a long history of hypertensive disease, revealed small size and weight
(80 grams), firm kidneys. They had grey color and granular surface. On a cut, there was uniform thinning of a
cortex. How it is possible to name the changes in kidneys?
A. *Primary- scarring kidneys
B. Pyelonephritic scarring of kidneys
C. Secondary - scarring kidneys
D. Amyloidal- scarring kidneys
E. Diabetic glomerulosclerosis
11. A 63-year-old man, with a long history of hypertensive disease, died from a hemorrhage in the brain. An
autopsy revealed reduced in sixes, firm kidneys with fine-grained surface and thinned cortex matter. These changes
in kidneys are characteristic for:
A. *Primary-scarring kidneys
B. Secondary-scarring kidneys
C. Amyloidal-scarring kidneys
D. Pyelonephritic-scarring of kidneys
E. Infarct of the kidney

76
12. An autopsy of a 61-year-old female revealed the thickening of her heart's left ventricle up to 2.5 cm. Her
kidneys were small, contracted and firm. The surface of the kidneys had a granular appearance. On a cut, there was a
thinning of a cortical substance. Microscopical investigation showed arteriolosclerosis,
glomerulosclerosis and interstitial sclerosis. What is the most likely disease?
A. * Hypertension
B. Atherosclerosis
C. Ischemic heart disease
D. Rheumatism
E. Amyloidosis of kidneys
ISCHEMIC HEART DISEASE
1. An autopsy of a 49-year-old patient, who died from lungs edema, revealed in myocardium a yellow-grey, large
focus and a fresh blood clot in a coronal artery. What is the most likely diagnosis?
A. *Myocardium infarction
B. Cardiosclerosis
C. Myocarditis
D. Amyloidosis
E. Cardiomyopathy
2. An autopsy of a 58-year-old female revealed in myocardium a large, dense, grey focus, which histologically
consisted of the rough connective tissue fibers. It was surrounded by the hypertrophied muscular fibers. What
changes arose up in a heart?
A. *Postinfarction Cardiosclerosis
B. Ischemic stage of myocardial infarction
C. Necrotic stage of myocardial infarction
D. Diffuse Cardiosclerosis
E. Myocarditis
3. An autopsy of a 62-year-old male, with a history of ischemic heart disease, revealed an atherosclerosis of coronal
arteries and signs of hypertensive disease. On a cut of the heart, in the area of the apex and left ventricle's frontal and
lateral walls, there was a well defined yellowish focus, surrounded by hemorrhages. What is the most likely
pathological process in the cardiac muscle?
A. * Myocardium Infarction
B. Postinfarction Cardiosclerosis
C. Diffuse Cardiosclerosis
D. Myocarditis
E. Fatty dystrophy of myocardium
4. A 32-year-old man suddenly died during the emotionally strained work. An autopsy revealed uneven
myocardium blood supply. Histochemical investigation detected a decrease of the amount of glycogen. Electron-
microscopical study showed a destruction of mitochondria, the contractures of myofibrils. What is the most likely
disorder of a blood circulation?
A. *Acute ischemia
B. Chronic ischemia
C. Vacating arterial hyperemia
D. Acute vein hyperemia
E. Angioneurotic arterial hyperemia
5. An autopsy of a 56-year-old male, with a history of ischemic heart disease, revealed the edema of lungs. What
pathological changes could cause a pulmonary disorder?
A. * Acute insufficiency of left ventricle
B. Acute general anemia
C. Acute insufficiency of right ventricle
D. Ischemia of small circle
E. Blood stasis
6. A 48-year-old man, with a history of transmural heart infarction of the left ventricle's myocardium, died from
the veritable rupture of. heart (the heart tamponade). What process in an infarct zone could promote a heart's wall
rupture?
A. *Autolysis with melting of myocardium tissue (myomalacia)
B. Substitution of connective tissue in area of infarction (organization)
C. Rising of blood pressure in the small circulatory circle
D. Scar formation with thinning of wall of the left ventricle
7. A patient, with a history of myocardial infarction, develops symptoms of blood circulation insufficiency after
physical exercises. There is marked cyanosis and edema of subcutaneous tissue of his lower extremities. What
changes have developed on a place of a myocardial infarction at the recovered person?
A. *Cardiosclerosis
B. Intracellular regeneration
C. Myocarditis

77
D. Atrophy of myocardium E.
8. A 36-year-old man, with acute transmural infarction of the left ventricle's myocardium, died from the rupture of
the heart and tamponade. What process could be a reason of the heart rupture in the zone of infarction?
A. *Autolysis with melting of myocardium fabric (myomalacia)
B. Substitution of connective fabric in area of infarction (organization)
C. Rising of blood pressure in the small circulatory circle
D. Scar formation with thinning of wall of the left ventricle
9. A 58-year-old male, with a history of hypertensive disease, developed a long attack of substernal pain. The
infarction of myocardium was diagnosed. A patient died soon. An autopsy revealed flabby myocardium with uneven
blood filling. Histological and histochemical examinations showed disappearance of glycogen granules and
decreased activity of oxidizing enzymes. What stage of myocardial infarction presented in that case?
A. *Ischemic stage
B. Necrotic stage.
C. Organization
D. Recurrent myocardial infarction
E. Acute relapsing myocardial infarction
10. A 64-year-old patient, with long history of atherosclerosis and myocardial infarction, developed the attack of
substernal pain. A patient was hospitalized in 3 days and died soon from progressive cardiovascular insufficiency.
An autopsy revealed in the back wall of the left ventricle and interventricular septum of heart a white color focus,
about 3 cm in a diameter. It was fibred, falling back, with a clear boundary. Give the name for these changes:
A. *Focal cardipsclerosis
B. Myocardial ischemia
C. Myocardial infarction
D. Myocarditis
E. Dystrophy of myocardium
11. A 52-year-old male, after repeated intramural heart infarction of myocardium, gradually returned to health and
further supervision of a district internist. In 2 years he died in a motor-car accident. Define a pathological process in
myocardium, which wound be revealed at the autopsy?
A. *Focal cardiosclerosis
B. Diffuse cardiosclerosis.
C. Atrophy.
D. Necrosis.
E. Hyperplasia
12. A patient developed substemal pains in the 7 o'clock in the morning. He presented at the Emergency
Department in 8 o'clock in the morning, where electrocardiograph investigation revealed a myocardial infarction.
Ten minutes later he died. What most reliable morphological sign of myocardium infarction would be found at
histological investigation after the autopsy?
A * Disappearance of glycogen in myocardial cells
B. Vacuolar dystrophy of myocardial cells
C. Fatty infiltration of myocardial cells
D. Necrosis of myocardial cells
E. Weakening of myofibril of myocardial cells
13. A patient died from progressive cardiac insufficiency. An autopsy revealed a flaccid, dilated in diameter heart.
A cut surface investigation showed the irregular blood filling of a myocardium, resulted in the patchy pattern of its
tissue. A histological study determined a hyperemia of myocardium and stromal accumulations of small
mononuclear cells. The described morphological changes present:
A. *Nonpurulent interstitial myocarditis
B. Vein plethora
C. Fatty dystrophy of myocardium
D. Cardiosclerosis
E. Myocardial infarction
14. A 56-year-old patient presented to the hospital with symptoms of acute myocardial infarction. A diagnosis was
confirmed by the EKG and laboratory tests. For 5th days the condition acutely worsened. A progressive cardiac
insufficiency resulted in patient's death. A dissection confirmed the diagnosis of myocardial infarction, complicated
by the heart's wall rupture and tamponade of pericardium. What process developed in the area of myocardial
infarction?
A. *Aseptic autolysis
B. Organization
C. Encapsulation
D. Septic disintegration
E. Petrification
15. A 57-year-old man, with a long history of an alcohol abuse, died at the increasing phenomena of chronic heart
failure. An autopsy revealed the weight of a heart 580 grams; a languid, clay color myocardium, with intensive

78
diffuse interstitial fibrosis. Coronal arteries were intact. A microscopical study of myocardium showed a
combination of hydropic and fatty dystrophy of cells; atrophy and hypertrophy of cardiomyocites. There were also
some foci of a cells lysis, accompanied with sclerosis. What kind of cardiomyopathy described in that case?
A. *Alcoholic
B. Hypertrophic
C. Dilatative
D. Restrictive
E. Metabolic
16. A 66-year-old patient presented in the hospital with the acute recurrent myocardial infarction of front-lateral
wall of the left ventricle. On the 4th day of disease, an acute, marked difficulty in breathing, a cough with
considerable quantity of a foamy sputum discharge and facial cyanosis developed. A patient died from progressive
cardiac insufficiency. An autopsy revealed enlarged grey-pink color lungs. A foamy liquid flowed down from the
cut surface. What pathological process in lungs caused a death?
A. *Edema of the lungs
B. Lungs infarction
C. Hydro thorax
D. Pneumonia
E. Pneumosclerosis
RHEUMATIC DISEASES
1. An autopsy of 48-year-old female revealed a mitral stenosis with valve's incompetence. Histological investigation
showed the post- inflammatory cardiosclerosis and Aschoff-Talalayev nodes (granulomas). What is the most likely
diagnosis?
A. *Rheumatism
B. Systemic scleroderma
C. Dermatomyositis
D. Nodular periarteritis
E. Lupus erythematosus
2. An autopsy of a 34year-old patient with a long history of rheumatism, revealed the epicardial surface of the heart
with shaggy exudate formed by grey strands which easily separate from underlying tissues and described as 'bread-
and-butter' pericarditis ('hairy heart'). What is the most likely diagnosis?
A. *Fibrinous pericarditis
B. Purulent pericarditis
C. Hemorrhagic pericarditis
D. Proliferated pericarditis
E. Catarrhal pericarditis
3. A histology investigation of a mitral valve sample of a patient who died from complications of rheumatism
revealed a mucoid swelling, a damage of the endothelial cells and also thrombi at the valve closure line. Name the
type of the rheumatic endocarditis?
A. * Acute warty endocarditis
B. Diffuse endocarditis
C. Fibroplastic endocarditis
D. Relapsing warty endocarditis
E. Polyps-ulcerated endocarditis
4. A 9-year-old boy presented with painless, firm 1-2 mm nodules at the skin around ulnar and knee joints (at
extensor's area). Biopsy investigation revealed a central area of fibrinoid necrosis of the connective tissue
surrounded by lymphocytes and macrophages. What disease these nodules are characteristic for?
A. * Rheumatism.
B. Rheumatoid arthritis.
C. Systemic scleroderma.
D. Nodular periarteritis.
E. Lupus erythematosus.
5. A 28-year old female died from uremia. A post-mortem revealed an enlarged kidneys, which had a patchy
pattern with hemorrhages on there surface. Histological investigation showed eosinophilic deposits
(hematoxilin bodies), "wire loop" lesions in the basement membrane of the glomerular tuft, hyaline thrombi and foci
of fibrinoid necrosis and also 'onion skin' sclerosis at the spleen's vessels. What is the most likely diagnosis?
A. * Lupus erythematosus.
B. Rheumatism.
C. Systemic scteroderma.
D. Rheumatoid arthritis.
E. Nodular periarteritis.
6. A 56-year old male with a long history of a rheumatic heart insufficiency died with symptoms of hemiplegia
shortly before death. Histological examination of his mitral; valve revealed severe sclerosis, nodular collection of

79
inflammatory cells and vegetations composed mainly of platelets and fibrin. Name the most likely type of
endocarditis?
A. *Relapsing warty endocarditis
B. Acute warty endocarditis
C. Diffuse endocarditis
D. Fibroplastic endocarditis
E. Polyps-ulcerated endocarditis
7. Microscopic investigation of the heart auricle from a patient with a history of mitral stenosis revealed the Achoff-
Talalayev's bodies (granulomas). What was the most likely cause of the heart insufficiency supported by the
histology results?
A. *Rheumatic
B. Atherosclerotic
C. Syphilitic
D. Innate
E. Septic
8. A 32-year-old female with a long history of rheumatic valve's defect presented at the hospital with tachypnea and
dyspnea, leg's edema, ascites and hepatomegaly. She died from the chronic heart insufficiency. An autopsy revealed
a mitral stenosis. What was the most likely factor of the mitral stenosis morphogenesis?
A. *Leaves union between itself
B. Sclerosis and bulge of leaves
C. Sclerosis and shortening of leaves
D. Shortening of tendon filaments
E. Presence of small blood thromboses on the surface of valve
9. An autopsy of 61-year-old male with a history of rheumatoid arthritis revealed enlarged dense kidneys, which
had yellowish-whitish color and waxy appearance. Grossly, foci of scars were recognized at kidneys surface.
Microscopically, at the slides stained by Congo red, homogeneous pink masses at capillaries of glomeruli tufts,
arterioles walls and arteries, basal membranes of tubuli and in stroma were found. Name the described complication
of rheumatoid arthritis?
A. *Secondary amyloidosis of kidneys.
B. Postinfective glomemlonephritis.
C. Quickly progressive glomerulonephritis.
D. Acute necrotic nephrosis.
E. Fibroplastic glomerulonephritis.
10. A 48-year-old female after exposure to cold presents to her physician with painful deformed fingers joints,
which bones were restricted in their movements. Physical examination revealed small firm nodules near the joints.
Histological investigation of the nodules biopsy showed the centrally located core of fibrinoid necrosis with
surrounding rim of macrophages and hystiocytes. What is the most likely diagnosis?
A. *Rheumatoid arthritis.
B. Dermatomyositis.
C. Rheumatism.
D. Gout.
E. Deformed arthrosis.
11. A post-mortem of 40-year old female who died from uremia revealed enlarged kidneys, which had a patchy
pattern of their surface. Histological investigation of kidneys showed eosinophilic deposits (hematoxylin bodies),
"wire loop" lesions in the basement membrane of the glomerular tuft, hyaline thrombi and foci of fibrinoid necrosis.
Besides these, Libman-Sacks endorcarditis was also determined. What is the most likely pathology in kidneys?
A. *Lupus erythematosus nephritis.
B. Rheumatoid glomerulonephritis.
C. Choleric glomerulonephritis.
D. Sclerotic kidney.
E. Terminal glomerulonephritis
12. A 33-year old female died from chronic kidney failure. A post-mortem revealed multiple scars and infarcts in
kidneys and spleen. Histological investigation showed alteration of small and medium sized arteries presented with
sclerosis and mild endothelial proliferation. A severe lymphocytes and histiocytes infiltrates were also recognized at
the perivascular tissues. What is the most likely disease caused these alterations?
A. *Nodular periarteritis
B. Atherosclerosis
C. Hypertonic disease
D. Morphine's Disease
E. Visceral syphilis
13. A post-mortem of a 25-year old woman who died from chronic kidney failure revealed a reddish malar rash
('butterfly rash') and small (up to 0,2 cm) pale tan spreading vegetations over the mitral valve surface. Histological

80
investigation of kidneys showed foci of fibrinoid necrosis, eosinophilic deposits (hematoxylin bodies), "wire loop"
lesions in the basement membrane of the glomerular tuft and karyorrhexis. What is the most likely diagnosis?
A. *Lupus erythematosus.
B. Nodular periarteritis.
C. Rheumatism.
D. Rheumatic arthritis.
14. Histological investigation of the mitral valves of the heart revealed the focal desquamation of endothelial cells
replaced by thrombi. The connective tissue had mucoid swelling areas and also zones of sclerosis and
revascularization. Name the type of valve's endocarditis?
A. *Relapsing warty endocarditis
B. Diffuse endocarditis
C. Acute warty endocarditis
D. Fibroplastic endocarditis
E. Polyps-ulcerated endocarditis
15. A 62-year-old woman presents to her physician with considerable deformation of metacarpal phalangeal and
feet joints. Histological examination of the soft tissues adjacent to the joints revealed a mucoid swelling of the
connective tissue, areas of a fibrinoid necrosis surrounded by palisading epithelioid macrophages and sclerosis. Few
'rice bodies' were found within a synovia! cavity. What is the most likely diagnosis?
A. * Rheumatic arthritis
B. Rheumatism
C. Behterev's Disease
D. Hematogenic tuberculosis
E. Gout
16. A 44-year-old woman presents to her physician ulnar deviation of the hands and flexion-hypertension ('swan
neck" or "walrus flipper") deformities of the fingers. Her metacarpal phalangeal joints are easily exposed to a
dislocation and a subluxation. Microscopical examination revealed nodular proliferations of synovium,
cartilage destruction and "pannus" formation. What is the most likely diagnosis?
A. *Rheumatoid arthritis
B. Rheumatic arthritis
C. Osteoarthritis.
D. Lupus erythematosus. E-
17. An autopsy of a child, who died of a heart failure, revealed the dilated heart chambers. Microscopic
investigations showed the hyperemia of the myocardial stroma, edema, and diffuse interstitial - infiltrates
consists of hystiocytes, lymphocytes, some scattered neutrophils and eosinophils. What is the most likely
diagnosis?
A. *Diffuse interstitial exudative myocarditis
B. Focal interstitial exudative myocarditis
C. Nodular productive myocarditis
D. Interstitial productive myocarditis
E. Alternative myocarditis
18. A 35- year- old female presented her physician intermittent episodes of ischemia of her fingers, marked by
pallor, paresthesias and pain, accompanied by tightening and thickening of the skin and poliarthralgia. Histological
investigation of the skin biopsy from affected areas revealed mild epidermal atrophy, hyalinosis of collagen fibers
within derma, scattered perivascular lymphocytes' infiltrates. Underlying skeletal muscles expressed interstitial
edema, loss of cross-section striation, nidal necroses followed with petrification. What is the most likely diagnosis?
A. *Dermatomyositis
B. Systemic scleroderma
C. Lupus erythematosus
D. Nodular periarteritis
E. Rheumatism
19. A post-mortem of a 19-year old female revealed multiple furuncles on her skin, a warty endocarditis of the heart
valves (Libman-Sacks endocarditis), a focal hemorrhage under the endocardium, ulcerative stomatitis,
esophagus ulcers, pneumonia, nephritis, and a spleen hyperplasia with a perivascular sclerosis. Morphological
investigation of her brain showed areas of necrosis and signs of vasculitis within thalamus. What is the most likely
diagnosis?
A. *Lupus erythematosus
B. Rheumatism.
C. Septic endocarditis.
D. Ischemic heart disease.
E. Hypertensive disease.
20. The woman of 45 years within several years has difficulties at swallowing. She also notes the limitation of
fingers movements at printing on the computer keyboard, though joints are not painful. Physical investigation
revealed her "stony face" (no wrinkles) owing to tightening of the facial skin and restricted motion of the mouth.

81
The skin biopsy showed a widespread fibrosis of a derma without inflammatory infiltration. Diagnose disease on the
listed clinical and morphological data.
A. * Scleroderma
B. Lupus erythematosus
C. Dermatomyositis
D. Amyloidosis
E. Rheumatic arthritis
21. A 43-year-old woman presents to her physician with pain and immobility of the bones in metacarpal phalangeal
and feet joints. Similar complains in symmetric pattern were about ulnar and knee joints, though not so
severe. Physical examination revealed pastous skin over the joints, a partial ankylosis of metacarpal phalangeal and
feet joints, and also ulnar deviation of the hands and flexion-hypertension ('swan neck"or"walrus flipper")
deformities of the fingers. In addition, movable firm, rubbery and tender 1cm hypodermic nodules were found in
the phalangeal joints area. An aspirate of joint fluid showed increased turbidity and presence of white "grains" ("rice
bodies"). Immunofluorescence also revealed the rhematoid factor. What is the most likely diagnosis?
A. *Rheumatic poliartritis
B. Systemic disease of connective tissue
C. Gout
D. Osteoartrosis E-
22. A post-mortem of the patient who died from uremia revealed deformation of a spine column with severe
restriction of its mobility. Articular cartilages of spine joints were destructed with persistent chronic inflammation in
tissues of joints. The joints cavities filled with connective tissue, in some places leading to ossification and ankylosis
formation. In an aorta, heart, lungs a chronic inflammation and a focal sclerosis were discovered. In kidneys an
amyloidosis was recognized. What diagnosis in this case is most probable?
A. *Ankiloid spondiloartritis (the Behterev's disease)
B. The Pedget's disease (deforming ostosis)
C. Rheumatoid arthritis.
D. Parathyroid osteodystrophy.
E. Osteopetrosis (marble disease). ;
23. At the patient with suspicion on a systemic disease a biopsy from a site of the skin tightening and restricted
motility was taken. A histology investigation revealed all kinds of disorganization of connective tissue fibers with
mild cellular reaction and also transition in excessive sclerosis and a hyalinosis. What is the most likely diagnosis?
A. *Scieroderma
B. Nodular periarteritis
C. Lupus erythematosus
D. Psoriasis
E. Dermatomyositis
THE RESPIRATORY SYSTEM DISEASES
1. An autopsy of the miner, who had worked in coal mine more of 10 years, revealed in a lung whitish fibrous fibers
and nodules 0,2-0,3 cm in diameter. Histological investigation of nodules showed bundles of interacting concentric
pink collagen with apparent hyalinosis. There was a minimal inflammatory reaction and also noticeable amount of a
brownish dust. What is the most likely type of pneumoconiosis in this case?
A. *Silicosis
B. Talcosis
C. Asbestosis D.Siderosis E. Berylliosis
2. Histological investigations of the bronchus wall biopsy material, from the patient with a history of chronic
bronchitis, revealed granulation tissue and diffuse inflammatory infiltrate. What kind of a bronchitis was
diagnosed?
A. *Chronic polypous bronchitis
B. Chronic mucopurulent bronchitis
C. Chronic mucous bronchitis
D. Chronic purulent bronchitis
E. Chronic deforming bronchitis
3. A 72-year old female died of severe grippe. A post-mortem revealed "the big motley lungs". Histological
investigation showed acute hyperemia, hemorrhages, an edema of a pulmonary tissue, erythrocyte-rich exudate that,
filled the bronchi and alveoli. What is the most likely kind of pneumonia?
A. *Hemorrhagic bronchopneumonia
B. Catarrhal bronchopneumonia
C. Purulent bronchopneumonia
D. Desquamative bronchopneumonia
E. Fibrinous bronchopneumonia
4. A 52-year old female, with a history of the chronic glomerulonephritis and chronic renal failure, presented to the
hospital with coughing spells accompanied by thick phlegm and breathlessness. Bronchoscopy revealed congested,

82
edematous, bronchial mucus membrane wim small hemorrhages. A bronchial lumen was narrowed by thick mucus.
Name the process in bronchi?
A. *Secondary acute catarrhal bronchitis
B. Primary acute catarrhal bronchitis
C. Chronic catarrhal bronchitis
D. Destructive - ulcerous bronchitis
E. Catarrhal - purulent bronchitis
5. A 68-year old patient, with a history of the chronic inflammatory disease of lungs since his childhood, presented
to the hospital with coughing accompanied by scanty sputum (expectoration). The complications of pulmonary and a
heart failure resulted in his death. What changes in heart have been found at post-mortem?
A. *Right ventricular hypertrophy and dilatation
B. Left ventricular hypertrophy and dilatation
C. Ventricular hypertrophy
D. Ventricular dilatation
E. Heart without gross change
6. A female patient, with 10 years history of cough with a purulent sputum and dyspnea, died from respiratory and
heart failure. A post-mortem gross investigation of a dead body revealed her fingers reminded drum sticks. A
dissection of lugs showed a bronchial alteration with a saccular deformity and purulent inflammation. What is the
most likely diagnosis?
A. *Bronchiectatic disease
B. Tuberculosis
C. Chronic bronchitis
D. Abscess
E. Acute bronchitis
7. A patient died in 3 weeks of the pneumonia onset. Macroscopic investigation of lungs revealed the whole of a
left inferior lobe became enlarged, consolidated and airless. Grossly lungs got the appearance of a grayish brown,
dry surface with fibrin fibers on the pleura. Histological investigation showed the collapsed and bloodless alveolar
capillaries, the exudate within alveoli consisted mainly of neutrophil polymorphs. What is the most likely diagnosis?
A. * Croupous pneumonia
B. Focal bronchopneumonia
С .[nfluenzal pneumonia гриппозная
D. Fibrinous pleuritis
E. Intestinal pneumonia
8. The biopsy is taken from a suspicious site at the mucous the right bronchus of a 58-year old male, with a history
of the chronic bronchitis, pneumosclerosis and cardiopulrnonary insufficiency. Flistological investigation revealed
cellular and tissue atypia, presence of a "cancer pearls" structures. What pathological process associated with
histological changes listed below?
A. *Squamous cell keratinous carcinoma
B. Chronic polypous bronchitis
C. Bronchiectasis
D. Acute bronchitis
E. Squamous metaplasia of the bronchial epithelium
9. Histological investigation of a biopsy of a bronchus revealed a tumor which is constructed from clusters of
atypical cells of a laminated (stratified) squamous epithelium, some areas with typical "pearls". What is the most
likely diagnosis?
A. * Squamous cell keratinous carcinoma
B. Squamous cell nonkeratinous carcinoma
C. Solid carcinoma
D. Mucinous carcinoma
E. Scirrhous carcinoma
10. A 42-year old male, with a history of the amyloidosis, died of chronic renal failure. A post- mortem revealed in
the lower lung lobe multiple dilated bronchi filled with abundant foul sputum. A gross investigation of a cut surface
showed a honey-comb pattern of a lungs tissue. A microscopic study demonstrated the chronic inflammation within
bronchial wall and also a replacement of elastic and muscular fibers by a connecting tissue. These changes in a lung
are regarded as:
A. *Bronchiectasis
B. Bronchopneumonia
C. Chronic bronchitis
D. Chronic pneumonia
E. Abscesses of lungs
11. A 53-year old male, with a history of the chronic diffusive bronchitis, presented to the hospital symptoms of
cardiopulmonary insufficiency, then resulted in the lethal outcome of the disease. An autopsy revealed
increased volume and hyperinflation of lungs, which covered a mediastinum with their edges and keep their form

83
when removed from the body and put at autopsy table. Gross investigation showed a pale grey colour of
lungs and a crunch sound while making an incision of a pulmonal tissue. A pressing a finger on a lungs tissue
creates a fossa. A mucopurulent exudate was determined within bronchi's lumen. What is the most likely diagnosis?
A. *Chronic diffuse obstructive emphysema
B. Chronic focal emphysema
C. Intestinal emphysema
D. Primary idiopathic. emphysema
E. Vicarious compensatory emphysema
18. A 48-year old male, with a history of common cold after acute cooling, presented to his physician with
symptoms of lung a heart insufficiency. He died soon at the hospital. A post-mortem revealed the right lung
enlarged, firm, hypoventilated, with a liver-like consistency and fibrin fibers on the pleura. On a cut section
pulmonary tissue had a gray coloring and looked granular. A turbid liquid flowed down the surface. Histological
investigation showed an acute inflammation with the exudate within alveoli consisted mainly of neutrophil
polymorphs. What is the most likely diagnosis?
A. *Croupous pneumonia
B. Focal pneumonia
C. Intestinal interalveolar pneumonia
D. Staphylococcal pneumonia
E. Idiopathic fibrosing alveolitis
19. A 52-year old male presented to hospital with symptoms of acute pneumonia, complicated on the 6th day by
pulmonary edema. The latter one resulted in the patient's death. A post-mortem revealed the upper part of the right
lung enlarged, firm, with fibrin fibers on the pleura. On a cut section pulmonary tissue had a gray coloring and
looked granular. A turbid liquid flowed down the surface. Histological investigation showed the exudate within
alveoli consisted of fibrin, neutrophil polymorphs, macrophages, and disintegrated red cells. What is the most likely
diagnosis?
A. *Croupous pneumonia
B. Staphylococcal bronchopneumonia
C. Virus pneumonia
D. Hypostatic pneumonia
E. Adult respiratory distress syndrome
20. An autopsy of the middle age man with a long history of bronchiectasis revealed markedly enlarged adrenals
volume at the expense of a cortical layer. Adrenals were pale, dense, and sebaceous. Microscopic investigation
showed the unstructured, amorphous, eosinophilic, Congo red positive deposits by reticular stroma and within
vessels walls. These changes indicate a development of:
A. *Amyloidosis
B. Mucoid swelling
C. Fibrinoid swelling
D. Lipidosis
E. Hyalinosis
21. A post-mortem revealed the left lung enlarged, firm, with fibrin fibers on the pleura. On a cut section pulmonary
tissue had a gray coloring. A turbid liquid flowed down the surface. What is the most likely diagnosis?
A. *Croupous pneumonia
B. Focal pneumonia
C. Intestinal pneumonia
D. Cancer of lungs
E. Bronchiectatic disease
22. A 42-year old woman, with a history of common cold after acute cooling, presented to his physician with
symptoms of cough, chest pain, fever, and difficulty in breathing. She died on the 5th day staying at the hospital. A
post-mortem revealed the inferior lobe the right lung enlarged, firm, with membranous fibrin fibers on the pleura.
On a cut section pulmonary tissue had a gray coloring and looked granular. What is the most likely diagnosis?
A. *Croupous pneumonia
B. Influenzal pneumonia
C. Bronchopneumonia
D. Caseous pneumonia
E. Measles pneumonia
23. Histological investigation of pulmonary segment, removed from a coal worker's lung, revealed multiple
roundish, nodules composed of concentric whorls of dense, hyaline fibers of collagen. What is the most likely
disease?
A. *Silicosis
B. Tuberculosis
C. Bronchitis
D. Fibrosing alveolitis
E. Cancer of lungs

84
24. A 46-year old male, with a history of the chronic bronchiectasis, died from uremia. An autopsy revealed
enlarged, firm kidneys, with sebaceous (waxy) cut surface. To what disease there correspond such changes?
A. *Amyloidosis of kidney
B. Glomerulonephritis
C. Acute tubular necrosis
D. Pyelonephritis
E. Arteriolosclerotic nephrosclerosis
25. A 54-year old male, with a history of destructive purulent bronchitis died of multiple organ failure. An autopsy
revealed cardiomegaly, a sebaceous (waxy) kidney and a sago spleen. What process has complicated bronchitis?
A.* Secondary amyloidosis
B.Senile amyloidosis
C. Tumor-like amyloidosis
D. Primary amyloidosis
E. Generalized hyalinosis
D. Silicoanthracosis
E. Asbestosis
35. An autopsy of a patient, who had been working within 20 years the coal worker, revealed grayish-black dense,
sclerotic lungs. The vast areas of collagen tissue new growth and multiple macrophages, carrying black pigment
within cytoplasm, were recognized under microscope. What is the most likely disease?
A. *Anthracosis
B. Anthracosilicosis
C. Silicoanthracosis
D. Talcosis
E. Siderosis
36. An autopsy of the 60-year-old man with a long history of over consumption of alcohol revealed the right lung
enlarged, firm, with fibrin fibers on the pleura. On a cut section pulmonary tissue had a gray coloring. Histological
investigation showed the exudate within alveoli consisted of fibrin and disintegrated erythrocytes. What is the most
likely diagnosis?
A. *Croupous pneumonia
B. Focal pneumonia
C. Intestinal pneumonia
D. Primary tuberculosis of lung
E. Caseous pneumonia
37. An autopsy of the 47-year-old man revealed the right lung firm with dry hypoinflated granular pulmonary
tissue. Visceral pleura presented with membranous gray-brown fibrin fibers on the pleura. What is the most
likely diagnosis? What is the most likely diagnosis?
A. *Croupous pneumonia
B. Tuberculosis
C. Bronchopneumonia
D. Intestinal pneumonia
E. Pneumofibrosis
38. A 42-year old man presented to the hospital with symptoms of malaise, complicated with a collapse one hour
later. He died soon, not coming to consciousness. A post-mortem revealed the lungs enlarged, congested, cut
with a sound of crunch. Whole of a right inferior lobe had a liver-like texture with some fibrin fibers superposed on
the pleura. On a cut section pulmonary tissue exhibited a gray-brown coloring. What is the most likely diagnosis?
A. *Croupous pneumonia
B. Bronchopneumonia
C. Acute venous hyperemia in lungs
D. 1'oxic shock syndrome
E. Acute myocardial infarction
39. An autopsy of the 69-year-old man with a long history of chronic bronchitis revealed increased pulmonary
volume and hyperinflation, pale grey color, soft texture lungs that kept their form when removed from the body and
put at autopsy table. Histological investigation showed markedly dilated proximal acini, thinned and somewhere
ruptured interalveolar septas. For what disease of lungs such morphological picture is characteristic?
A. *Emphysema of lungs
B. Chronic intestinal pneumonia
C. Atelectasis of lungs
D. Bronchiectatic disease
E. Pleural empyema
40. An autopsy of a patient, who had been working within 20 years the coal worker, revealed enlarged, slightly firm
and hypoinflated lungs. At the gross investigation of the lungs sectional view multiple, round, grayish-black, diffuse
nodules were discovered. Histological study showed nodules composed of concentric whorls of dense collagen
fibers, some of them- with hyaline degeneration. What is the most likely disease?

85
A. *Nodular form of silicosis
B. Miliary tuberculosis of lung
C. Bronchiectatic disease
D. Croupous pneumonia
E. Diffusive - sclerotic form of silicosis
DISEASES OF ALIMENTARY SYSTEM
1. An operation of a male with a medical history of a hematemesis revealed a stomach ulcer penetrated into the
muscular layer. Ulcer edges were dense. A bleeding vessel was detected at the ulcer's bottom. What is the type of an
ulcer?
A. * Chronic ulcer with hemorrhage
B. Chronic ulcer with penetration
C. Ecute ulcer with bleeding
D. Chronic ulcer with perforation
E. Chronic ulcer with malignisation
2. A removed appendix delivered to pathology department. Macroscopic investigation revealed its
thickening, enlargement, dull and hyperemic serous membrane. A yellowish-green liquid lumen was visible on a cut
section within its lumen. What is the most likely type of the appendicitis?
A. * Suppurative appendicitis.
B. Catarrhal appendicitis.
C. Superficial appendicitis.
D. Gangrenous appendicitis.
E. Apostematous appendicitis
3. A microscopic investigation of a colon biopsy revealed a superficial defect of a mucosa, decrease of goblet cells
number with lack of mucus in them. In addition, considerable lympho- plasmocyte infiltration with presence
of segmented leucocytes and eosinocytes was defined. Choose the most probable and exact diagnosis.
A. *Nonspecific ulcerative colitis in exacerbation
B. Crohn's disease
C. Chronic ischemic colitis
D. Dysentery in the third stage
E. Amebiaz of intestine with forming of ulcers
4. Microscopic study of appendix determined intensive leukocyte infiltration hyperemia, stases throughout all
its layers. What is the most likely type of appendicitis?
A. * Suppurative appendicitis
B. Gangrenous appendicitis
C. Superficial appendicitis
D. Simple appendicitis
E. Chronic appendicitis
5. Macroscopic investigation of a removed appendix revealed its thickening, enlargement, dull and hyperemic
serous membrane with whitish membranaceous incrustation. A whitish-yellow, opaque, viscous liquid detected on a
cut section within its lumen. What is the most likely type of the appendicitis?
A. * Suppurative appendicitis.
B. Gangrenous appendicitis
C. Simple appendicitis
D. Superficial appendicitis
E. Chronic appendicitis.
6. The removed appendix thickened and covered by fibrinopuralent incrustation. A purulent exudate infiltrated
all appendix layers; the destruction of mucosa membrane was evident. What is the most likely diagnosis?
A. Flegmonous-ulcerative appendicitis
B. Simple appendicitis
C. Suppurative appendicitis
D. Gangrenous appendicitis
E. Superficial appendicitis
7. An autopsy of a 48-year-old woman with a history of the stomach neoplasm revealed the markedly enlarged,
firm, white color ovary. Microscopic investigation showed markedly atypical epithelial cells placed among the
layers and fibers of a connecting tissue. What is the most likely diagnosis?
A. *Krukenberg tumor of ovary.
B. Serous cystadenocarcinoma
C. Pseudomucinous cystadenocarcinoma.
D. Malignant thecoma.
E. Malignant granulosa cell tumor.
8. A post-mortem of a male with a history of anemia accompanied by vomiting by dark gastric contents reveled in a
stomach about 1 liter of liquid blood and bloody clots. A gross investigation showed an on small curvature of a
stomach an oval solitary ulcer with the elevated firm edgesand a smooth floor. What is the most likely diagnosis?

86
A. *Chronic gastric ulcer
B. Chronic atrophic gastritis
C. Acute gastritis
D. Acute gastric ulcer
E. Chronic hyper-trophic gastritis
9. A 48-year old female presents to her physician enlarged supraclavicular lymph nodes. Microscopic investigation
of a biopsy from a lymph node revealed the metastasis of the signet-ring cancer. Choose the most probable
localization of a primary tumor.
A. *Cancer of stomach.
B. Cancer of oesophagus.
C. Cancer of thyroid.
D. Cancer of lung.
E. Cancer of cervix uteri.
10. A histological investigation of a gastric biopsy revealed a thinning of a stomach mucosa with reduction of a
glands quantity. This was accompanied with a considerable growth of a connecting tissue, dilation of a
glands lumen, lymphocytes and plasmocytes infiltration of the mucosa. What is the most likely diagnosis?
A. * Chronic atrophic gastritis
B. Chronic superficial gastritis
C. Chronic atrophic gastritis with an intestinal metaplasia
D. Phlegmon of stomach E.
11. Microscopic study of appendix determined intensive leukocyte infiltration throughout all its layers. What is the
most likely type of inflammation?
A. * Suppurative appendicitis
B. Apostematous appendicitis
C. Gangrenous appendicitis.
D. Superficial appendicitis.
E. Flegmonous-ulcerative appendicitis
12. A histological investigation of the removed stomach ulcer revealed in its floor the fibrinopurulent exudate, a
zone of the fibrinoid necrosis, a granulation tissue with a fibrous tissue underneath. What is the most likely
diagnosis?
A. * Chronic ulcer
B. Acute ulcer
C. Acute erosion
D. Phlegmon of stomach E.
13. A macroscopic investigation of the appendix, 9 cm in length and 0.9 cm in thickness, revealed dull and
hyperemic serous membrane. Microscopic study determined edema, stases in capillaries and venules, fine
hemorrhages throughout the mucosa. Besides these, the necrotic foci with leukocyte infiltration around them were
also verified. What is the most likely diagnose?
A. * Superficial appendicitis.
B. Simple appendicitis.
C. Suppurative appendicitis
D. Flegmonous-ulcerative appendicitis
E. Gangrenous appendicitis.
14. A gastroscopy of a 44-year old patient, with a history of a pain in epigastrium after meal, revealed a hyperemia
of the stomach mucosa with the stomach folds reduction. Histological study showed a thinning of the mucosa, a
reduction of the glands quantity accompanied with the growth of a connecting tissue, lymphocytes and plasmocytes
infiltration. Specify, what of the listed diagnoses is the most probable?
A. *Chronic atrophic gastritis.
B. Acute catarrhal gastritis.
C. Acute suppurative gastritis.
D. Chronic superficial gastritis.
E. Giant hypertrophic gastritis.
15. A physical examination of a fauces of a 12-year-old child with tonsillitis revealed the reddening and slight
edema of mucosa, with reactive enlargement of tonsils, dotted by pinpoints of exudates emanating from the tonsillar
crypts.What clinical-morphological variant of tonsillitis is the most probable in this case?
A. *Lacunar tonsillitis
B. Catarrhal tonsillitis
C. Purulant tonsillitis
D. Fibrinous tonsillitis
E. Necrotic tonsillitis
16. A 44-year old patient, with a history of the chronic duodenum peptic ulcer, died of peritonitis. An autopsy
revealed multiple steatonecroses of a retroperitoneal tissue and a pancreas. A gross investigation of a duodenum

87
demonstrated an ulcerative defect, which was 5 mm in diameter and 10 mm deep. The edges presented necrotic
masses. Diagnose the complication of a duodenal peptic ulcer?
A. *Penetration
B. Hemorrhage
C. Stenosis
D. Perforation E.
17. A histological investigation of a gastric biopsy of a 50-year old woman revealed a thinning of a stomach
mucosa with reduction of a glands quantity, foci of an intestinal metaplasia; a plethora, an edema and a stromal
sclerosis. These were accompanied with a diffuse leucocytes, lymphocytes and plasmocytes infiltration of
the mucosa. What is the most likely diagnosis? A. *Chronic atrophic gastritis in an active phase
B. Chronic atrophic gastritis in a nonactive phase
C. Chronic superficial gastritis
D. Acute catarrhal gastritis
E. Acute fibrinouse gastritis
18. A gross investigation of a stomach revealed a deep defect of a gastric wall with a lesion of a muscular layer. The
proximal edge was undermined and distal one was flat. A histological study of the removed stomach showed in its
floor the zone of the fibrinoid necrosis, a granulation tissue and massive fibrosis, which replaced the muscular layer.
What is the most likely diagnosis?
A. *Chronic ulcer in exacerbation
B. Chronic ulcer in remission
C. Acute ulcer
D. Erosion
E. Cancer-Ulcer
19. Microscopic study determined edema, diffusive leukocyte infiltration of appendix wall and also a mucosa!
lesion with alteration of its muscular layer. What is the most likely diagnose?
A. * Flegmonous-ulcerative appendicitis
B. Suppurative appendicitis
C. Gangrenous appendicitis
D. Superficial appendicitis
E. Apostematous appendicitis
20. A gastroscopy of a 55-year old patient revealed a diffusive swelling, hyperemia and solitary small hemorrhage
of the stomach mucosa. These were accompanied with considerable quantity of a muddy, viscous grey exudate on a
gastric surface. What gastritis has developed at the patient?
A. *Catarrhal gastritis
B. Hemorrhagic gastritis
C. Flegmonous gastritis
D. Fibrinous gastritis
E. Corrosive gastritis
DISEASES OF THE LIVER
1. A microscopical investigation of a liver biopsy revealed a dystrophy, necroses of hepatocytes, a sclerosis,
accompanied with destruction of a hepatic beam and a lobular structure, a formation of false lobes and regenerative
nodes. What is the most likely diagnosis?
A. * Cirrhosis of liver
B. Chronic gepatoz
C. Chronic hepatitis
D. Massive diffuse necrosis of liver
E. Acute hepatitis
2. An autopsy of a 53-year old patient, with a history of a chronic alcoholism and repeated attacks of an alcoholic
hepatitis, revealed a firm, yellow color liver with pointed edge. A liver surface was coarsely scarred with multiple
fine knots on a cut surface. What is the most likely diagnosis?
A. * Cirrhosis of liver.
B. Cancer of liver.
C. Fatty dystrophy of liver.
D. Chronic hepatitis.
E. Acute hepatitis.
3. A 62-year old patient, with a history of a cholelithiasis accompanied with cholangitis and cholangiolitis,
presented to the hospital liver cirrhosis. What is the most likely type of cirrhosis presented?
A. *Biliary
B. Infection
C. Toxins
D. Nutritional deficiency
E. Circulatory

88
4. A patient presented to the hospital an ascites, twice enlarged spleen, a varicose esophagogastric veins and veins
around and within the rectum. A histological investigation of liver biopsy revealed micronodular cirrhosis. What
process has complicated cirrhosis?
A. * Portal hypertension
B. Cardiac insufficiency
C. Hepatotenal syndrome
D. Hepatic-cellular insufficiency E.
5. A 62-year old patient, with a history of chronic virus hepatitis, died of an acute posthemorrhagic anaemia
resulted from an esophageal varicose veins bleeding. An autopsy revealed the reduced, dense liver with coarsely
scarred surface. A microscopical investigation shpwed regenerative false nodules separated by fibrous tissue,
contained the remnants of portal tracts. What morphogenetic type of cirrhosis took place in this case?
A. *Portal cirrhosis.
B. Postnekrotic cirrhosis.
C. Mixed cirrhosis.
D. Viral cirrhosis.
E. Biliary cirrhosis.
6. A 42-year-old male, with a history of rheumatic stenosis of the left atrioventricular orifice resulted in
chronic heart insufficiency, presented at the hospital with dyspnea at small physical exercises, tachycardia, a
cyanosis of his lips, bubbling rales in the inferior segments of lungs, leg's edema. What histological
changes will be characteristic for a liver?
A.*Necrosis of hepatocytes in the center of lobule, fatty dystrophy on periphery
B. Necrosis of hepatocytes in the center of lobule, hyalin dystrophy on periphery
C. Necrosis of hepatocytes in the center of lobule, hydropic dystrophy on periphery
D. Fatty dystrophy of hepatocytes in the center of lobule, necrosis on periphery
E. Hydropic dystrophy of hepatocytes in the center of lobule, necrosis on periphery
7. A 52-year-old patient presented with fatigue, loss of appetite, muscle and joint aches, an increase in body
temperature to 38°C. On the 7th day a patient exhibited a yellow color in the skin, urine and around the whites of the
eyes and also an acute pain in right hypochondrium. A histological investigation of his liver biopsy revealed a
destruction of the liver's beam architecture,-vacuolar and ballooning dystrophies of hepatocytes, some cells were
necrotic. The Councilman's bodies were also determined. On the periphery of lobes were registered the enlarged
quantity of multinuclear hepatocytes. What form of virus hepatitis is most likely?
A. * Icteric hepatitis
B. Malignant hepatitis
C. Chronic hepatitis
D. Cholestatic hepatitis
E. Anicteric hepatitis
8. An autopsy of a male, with a history of a drug abuse, revealed vacuolar dystrophy of hepatocytes, a
Councilman's bodies, periportal inflammation consisted mainly of chronic inflammatory cells, lymphocytes,
plasma cells and histiocytes. What is the most likely etiology of the disease?
A. *Viral
B. Bacterial
C. Toxic
D. Parasite
E. Fungus
9. The liver biopsy is taken from the patient with symptoms of a parenchymatous icterus and a portal
hypertension. Histological investigation revealed a fatty dystrophy of hepatocytes, a destruction of a hepatic beam
and a lobular structure, a formation of false lobes and regenerative nodes. These were accompanied with a porto-
portal fibrous septae formation and periportal lympho- macrophageal infiltrates. What is the most likely
diagnosis?
A. * Cirrhosis of liver
B. Alcoholic hepatitis
C. Chronic hepatosis
D. Viral hepatitis
E. Toxic dystrophy
10. An autopsy of a male, with a history of a chronic alcohol abuse, revealed a small, firm, micronodular liver.
Histological investigation showed a fatty dystrophy of hepatocytes, a formation of small regenerative (false) nodes
separated by fibrous septae with lympho- macrophageal infiltration. What is the most likely diagnosis?
A, *Alcoholic cirrhosis.
B. Chronic active alcoholic hepatitis.
C. Chronic persistent alcoholic hepatitis.
D. Toxic dystrophy of liver.
E. Fatty hepatosis.

89
11. A patient presented with vomiting, loss of appetite and jaundice. A histological investigation of his liver biopsy
revealed vacuolar and ballooning dystrophies of hepatocytes and necrosis of some cells. The Councilman's bodies
were also determined. Besides, a destruction of the liver's beam architecture within lobes and inflammation
consisted mainly of chronic inflammatory cells, lymphocytes, plasma cells and histiocytes were registered. What
form of virus hepatitis is most likely?
A. Viral hepatitis
B. Purulent hepatitis
C. Cirrhosis of the liver
D. Toxic dystrophy of the liver
E. Malaria
12. A 35-year-old man exhibits clinical symptoms of the virus hepatitis В. А puncture liver biopsy revealed: a
destruction of the liver's beam architecture with polymorphism of hepatocytes and plural mitosis figures.
Hepatocytes were enlarged, filled by vacuoles which contained a transparent liquid. What kind of a dystrophy is
characteristic for the described case?
A. * Hydropic dystrophy
B. Fatty dystrophy
C. Hyalin dystrophy
D. Carbohydrate dystrophy E.
13. An autopsy of a female, with a history of a rheumatic disease with a mitral stenosis, resulted in decompensation
stage, revealed a general venous plethora, small, firm and micronodular liver. Gross investigation of a cut surface
showed a lobular structure and yellowish-brown color of hepatic tissue. Define the most probable pathology
diagnosed in a liver.
A. *Portal cirrhosis
B. Fatty dystrophy
C. Billiary cirrhosis
D. Postnecrotic cirrhosis
E. Atrophy
DISEASES OF THE URINARY SYSTEM
7. An autopsy of a 59- year-old man revealed the coral-like stone filled all renal pelvis of a right kidney. The kidney
was enlarged, hyperemic with the capsule, taken out hardly. The pelvis and calyxes cavities were dilated, filled by a
turbid, greenish- yellowish viscous liquid. The mucosal layer was dim with some hemorrhages. A cut surface
investigation showed motley pattern of the kidney's tissue, with yellow spots in up to 1cm in diameter. What
complication to nephrolithiasis has developed?
A. *Chronic pyelonephritis with exacerbation
B. Tumour of kidney
C. Postinfectious glomerulonephritis
D. Primary amyloidosis
E. Rapidly progressive glomeralonephritis.
8. Histological study of a renal biopsy revealed a necrosis and a marked dystrophy of the convoluted tubules
epithelium, tubulorrhexis, stromal edema and hyperemic medulla. Name the pathological process?
A. *Necrotic nephrosis
B. Glomerulonephritis
C. Pyelonephritis
D. Cystic disease
E. Kidney stone disease
9. A 59-year-old man with a long history of bronchiectasis disease, a pneumorrhagia, presented with
facial and back edema, and proteinuria of 33mg/l. A pulmonary bleeding soon resulted in his death. A post-
mortem revealed the enlarged firm kidneys with a sebaceous (waxy) cut surface. Histological
investigation showed the accumulation at glomeruli and under tubular epithelium homogeneous eosinophilic,
Congo-Red positive masses, which gave a metachromasia reaction with the methyl violet. Name process
which has developed in kidneys?
A. *Amyloidosis
B. Hyalinosis
C. Fatty dystrophy
D. Mucoid swelling
E. Fibrinoid swelling
10. An autopsy revealed substantial enlargement of a right kidney. A gross investigation of a renal cut section
showed a stone in it with marked dilation of the pelvis and calyces by urine and thinning of the renal parenchyma.
What of diagnoses is the most likely?
A. * Hydronephrosis.
B. Pyelectasia
C. Pyelonephritis.
D. Cyst of kidney E.

90
11. An autopsy of a 44-year-old female revealed a substantial enlargement of a right kidney reminding grape
clusters. The cut surface showed cavities varying in size from 0, 5 to 3 cm in diameter, which were filled by serous
liquid and colloid masses. Kidney parenchyma between cavities was thinned to 0, 1 cm. What is the most likely
diagnosis?
A. *Renal cystic disease
B. Chronic pyelonephritis
C. Acute pyelonephritis
D. Nephrolithiasis
E. Dysphasia of kidneys
12. A post-mortem of a patient with a long history of cystitis and a dyskinesia of ureters revealed morphological
signs of uremia. Kidneys were contracted, with irregularly scarred surface. In the renal pelvis fine urate stones and
sand were registered. Histological investigation showed 'the thyroid kidney' and areas of an interstitial inflammation.
What is the most likely diagnosis?
A. *Chronic pyelonephritis
B. Acute pyelonephritis
C. Atherosclerotic-contracted kidney
D. Primary- contracted kidney
E. Amyloidal- contracted kidney
13. An autopsy of the 58-year-old patient revealed an enlarged kidneys, which had a well defined yellow-grey with
red specs cortical layer. Kidney's medulla colored in a dark red. Histological investigation revealed proliferation of
capsular epithelial cells and podocytes with crescents formation, sclerosis and hyalinosis -of the glomerular tufts,
stromal fibrosis. What is the most likely kidneys disease?
A. *Extracapillary productive glomeralonephritis
B. Intracapillary productive glomerulonephritis
C. Extracapillary exudative glomerulonephritis
D. Intracapillary exudative glomerulonephritis
E. Interstitial nephrite
14. A 58-year-old patient with a history of purulent osteomyelitis died of chronic renal insufficiency. A post-
mortem revealed the enlarged firm kidneys of white-yellow color with a sebaceous (waxy) cut surface. What is the
most likely diagnosis?
A. * Amyloidosis of kidneys
B. Chronic glomerulonephritis
C. Subacute glomerulonephritis
D. Septic nephrite
E. Acute necrotic nephrosis
15. A post-mortem revealed the enlarged kidneys with a lobulated surface due to plural cavities, with the smooth
wall, filled with a transparent liquid. What is the most likely disease?
A. *Renal cystic disease
B. Necrotic nephrosis
C. Pyelonephritis
D. Glomerulonephritis
E. Infarct
16. A physical examination of a young woman revealed marked edemas and high proteinuria. A histological
investigation of kidneys biopsy determined a disappearance of podocytes, a declining of heparansulfate in a
basal membrane of glomerular capillaries. What is the most likely disease?
A. * Idiopathic nephrotic syndrome
B. Postinfectious glomerulonephritis
C. Rapidly progressive glomerulonephritis
D. Chronic glomerulonephritis
E. Acute necrotic nephrosis
17. A post-mortem of a 52- year-old man, with a long history of chronic glomerulonephritis, resulted in his death,
revealed markedly reduced, firm, finegrained kidneys. Microscopical investigation showed fibrinous inflammation
of serous and mucosas, dystrophic changes of parenchymatous organs and a brain edema. What complication has led
to the serous and mucosas lesions?
A. *Uremia
B. Anaemia
C. Sepsis
D. DIC-syndrome
E. Thrombocytopenia
18. A young man died of a burn disease. A post-mortem revealed the brain edema, liver and kidneys enlargement.
Gross investigation showed the thickened pale-grey cortical layer and hyperemic medulla. Histological study
demonstrated focal tubular necrosis at different points along the nephron with a disruption of tubular basement

91
membrane interstitial edema with leukocytic infiltration and a hemorrhage. What of the listed diagnoses the
most authentic?
A. * Necrotic nephrosis.
B. Tubulointerstitial nephritis.
C. Pyelonephritis.
D. Gouty kidney.
E. Myeloma kidney.
19. Histological investigation of a kidney biopsy showed sclerosis, lymphocyte-plasmocyte infiltration of the renal
pelvis walls and calyces; tubules dystrophy and atrophy. The intact tubules were dilated, stretched by colloid-like
masses. The epithelium was flattened. In whole, the microscopic picture of tissue sample had 'a thyroid kidney'
pattern. What diagnosis is most probable?
A. * Chronic pyelonephritis
B. Sharp pyelonephritis
C. Glomerulonephritis
D. Nephrosclerosis
E. Tubulointerstitial nephritis
20. The patient treated for a cold by the big doses of paracetamol, presented symptoms of an oliguria and a/otemia.
In 5 days he died of an acute renal insufficiency. Histological investigation of kidneys revealed the diffusive edema
of an interstitial tissue of a kidneys cortex, its infiltration by lymphocytes, the eosinophils, some neutrophils.
Almost intact glomerules were found along with a destruction of a tubular epithelium. Name the most probable type
of a kidneys lesion?
A. * Tubulointerstitial nephritis
B. Acute glomerulonephritis
C. Nephrotic syndrome
D. Pyelonephritis
E. Necronephrosis
21. A macroscopic investigation of the removed kidney reveled renal swallowing, congestion and a capsule easily
taken out. Renal pelvis and calyxes were dilated, filled by turbid urine. Their mucosa was dim, with the areas of
hemorrhages. The cut surface of a kidney had a motley pattern; yellow-grey zones were surrounded by plethora and
hemorrhages areas. To what disease there corresponds such macroscopic picture of kidneys?
A. *Acute pyelonephritis
B. Acute glomerulonephritis
C. Amyloidosis of kidneys
D. Nephrolitiasis
E. Renal cystic disease
22. Histological investigation revealed proliferation of Bowman's capsule epithelial cells, podocytes and
macrophages, resulted in crescents formation, which compressed glomeruli. Necrotic capillary loops had
fibrinous thrombi within their lumens. A sclerosis and hyalinosis of some glomeruli were detected. In addition, the
expressed nephrocytes dystrophy, an edema and stromal infiltration of kidneys was observed. What is the most
likely kidneys pathology?
A. * Rapidly progressive glomerulonephritis
B. Postinfectious glomerulonephritis
C. Chronic glomerulonephritis
D. Chronic pyelonephritis
E. Amyloidosis of kidneys
23. A 53-year-old patient with a history of a chronic glomerulonephritis died of chronic renal insufficiency. A post-
mortem revealed smaller than normal, firm kidneys. Their capsule was taken out hardly, baring a granular renal
surface. On a cut section a cortical and cerebral layers were thin with dry, anemic, grey color kidneys tissue. How is
called such kidneys?
A. * Secondary-scarring kidneys
B. Primary- scarring kidneys
C. Atherosclerotic - scarring kidneys
D. Amyloidal- scarring kidneys
E. Pyelonephrotis - scarring kidneys
24. A 5-year-old girl with a history of an acute respiratory virus infection presented with widespread swelling, a
massive proteinuria, a hypoalbummemia, a lipidemia. A microscopic investigation of the renal biopsy revealed an
absence of small podocytes processes within vascular glomerulus. What is the most probable diagnosis?
A. *Lipoid nephrosis
B. Postinfectious glomerulonephritis
C. Rapidly progressive glomerulonephritis
D. Focal segmental glomerulosclerosis
E. Membranous glomerulonephritis

92
25. An autopsy of a patient, who died from poisoning of ethylene glycol, revealed enlarged edematous kidneys.
Their capsule was taken out very easily. Gross investigation showed the wide pale-grey cortical layer and dark red
medulla. What disease has developed at the patient?
A. * Necrotic nephrosis
B. Acute pyelonephritis
C. Acute glomerulonephritis
D. Acute tubulointerstitial nephritis
E. Lipoid nephrosis
26. A 12-year-old boy with a history of acute tonsillitis two weeks ago presented with edematous face in the
mornings, increasing of arterial pressure, urine in a kind of "meat slops"('coca-cola' colour). Immunohistochemistry
of kidneys biopsy revealed immune complexes on basal membranes of capillaries and within glomerular
mesangium. What disease has developed at the patient?
A. * Acute glomerulonephritis
B. Acute interstitial nephrite
C. Lipoid nephrosis
D. Acute pyelonephritis
E. Necrotic nephrosis
27. The young man presented in hospital with a headache and giddiness. Within last year he often had high arterial
pressure. Medical drugs almost did not help. Kidneys biopsy investigation revealed the extra capillary epithelial
proliferation with a crescent formation. For what disease described pathology is characteristic?
A. * Rapidly progressive glomerulonephritis
B. Acute glomerulonephritis
C. Lupus glomerulonephritis
D. Wegener's granulomatosis
E. Chronic glomerulonephritis
28. A 48-year-old woman died of renal insufficiency. A post- mortem revealed enlarged flaccid kidneys with wide,
swallowed, dim cortical layer. A yellow-grey with red specks cortex was delimited from dark red
cerebral substance. Microscopic investigation showed a proliferation of Bowman's capsule epithelial cells,
podocytes and macrophages, resulted in crescents formation. For what disease described changes are
characteristic?
A. * Subacute glomerulonephritis.
B. Acute glomerulonephritis.
C. Chronic glomerulonephritis.
D. Acute pyelonephritis.
E. Amyloidosis of kidneys.
29. A macroscopical investigation of a removed kidney revealed in proximal area of an ureter the concrement
which obturated its lumen. The kidney was markedly enlarged; a parenchyma was atrophic with considerably dilated
pelvis and calyces. Microscopical study of kidney's sample showed the diffusive sclerosis, an atrophy of glomeruli
and tubules. Survived tubules were cystically dilated. What complication of a nephrolithiasis developed at
the patient?
A. * Hydronephrosis.
B. Pyonephrosis.
C. Pyelonephritis.
D. Glomerulonephritis.
E. Chronic paranephritis.
30. An autopsy of the 68-year-old man revealed a sallow-grey colour of his skin with petechial hemorrhages. His
face and tongue had a powdery 'uremic frost'. Fibrinous hemorrhagic laryngitis, a tracheitis, a shaggy pericardium
and a catarrhal gastroenterocolitis were also diagnosed. For what syndrome the given complex of morphological
changes is characteristic?
A. *Chronic renal insufficiency
B. Acute hepatic insufficiency
C. Acute renal insufficiency
D. Chronic heart insufficiency E.
31. A man died of renal insufficiency. A post- mortem revealed enlarged flaccid kidneys with wide, yellow-grey
with red specks cortex. Microscopic investigation showed a proliferation of Bowman's capsule epithelial cells,
resulted in crescents formation. Capillary loops exhibited necrotic alterations and fibrin thrombi within their lumens.
What is the most likely diagnosis?
A. * Subacute glomerulonephritis
B. Acute glomerulonephritis
C. Lipoid nephrosis
D. Chronic glomerulonephritis
E. Amyloidosis of kidneys

93
32. A post-mortem of a patient with a history of chronic renal insufficiency revealed the enlarged, firm, sebaceous
(waxy) kidneys with multiple retractions on their surface. Histological investigation showed many glomeruli were
replaced by the Congo-Red positive masses. The same substance was found on the basal membranes of capillaries,
in mesangium and also within arterial walls and kidneys stroina. What of the listed diagnoses the most likely?
A. * Amyloidosis of kidneys
B. Acute glomerulonephritis
C. Chronic glomerulonephritis
D. Subacute glomerulonephritis
E. Lipoid nephrosis
33. A 60-year-old woman died of a chronic renal failure. An autopsy revealed the irregularly contracted and
markedly lobulated kidneys with a fibrous tissue alternated with an intact parenchyma. Renal pelvises were
dilated, their walls -thickened. Histological investigation showed sclerosis, lymphocyte - plasmocyte infiltration of
the renal pelvis walls, and interstitium. What diagnosis is most probable?
A. *Chronic pyelonephritis
B. Acute pyelonephritis
C. Acute glomerulonephritis
D. Tubulointerstitial nephritis
E. Chronic glomerulonephritis
34. The kidney biopsy is taken from the man who is in clinic with symptoms of a mercury poisoning. Histological
investigation revealed focal tubular necrosis, an edema, hyperemia, hemorrhages and leukocytic infiltration in a
stroma. What condition has developed at the patient?
A. * Acute necrotic nephrosis
B. Acute glomerulonephritis
C. Chronic renal insufficiency
D. Acute pyelonephritis
E. Chronic pyelonephritis
36. A 42-year-old man died of renal insufficiency. Microscopic investigation revealed a proliferation of Bowman's
capsule epithelial cells, podocytes and macrophages, resulted in crescents formation, necrosis of capillary loops and
fibrin thrombi within their lumens. In addition, the histological study showed sclerosis and hyalinosis of glomeruli,
tubular atrophy and stromal fibrosis of kidneys. What of the listed diagnoses the most likely?
A. * Subacute glomerulonephritis
B. Acute glomerulonephritis
C. Chronic glomerulonephritis
D. Focal segmental sclerosis
E. Membranous nephropathy
37. A patient died from uremia. A post- mortem revealed enlarged flaccid kidneys with wide, swallowed with red
specks cortex. Medulla had a dark red coloring. Microscopic investigation showed epithelial crescents
which compressed capillaries. Nephrocytes exhibited a dystrophy; a stromal edema and infiltration were also
recognized. What is the most likely diagnosis?
A. * Glomerulonephritis
B. Pyelonephritis
C. Nephrolithiasis
D. Nephrotic syndrome
E. Amyloidosis of kidneys
38. The elderly man, with 15 years history of a membranous -proliferative glomerulonephritis, constantly
received hemodialysis therapy. The last half a year he did not receive a treatment. He presented to the hospital with
extremely severe condition, without consciousness, with a smell of urea from his body and edemas. A marked
pleuritis, pericarditis and peritonitis were also diagnosed. In a whole, all symptoms were regarded as a uremia. What
kind of an inflammation is the most probable?
A. *Fibrinous inflammation
B. Purulent inflammation
C. Catarrhal inflammation
D. Hemorrhagic inflammation
E. Serous inflammation
39. A postmortem of a 58-year-old male revealed in kidneys asymmetrical, coarse, corticomedullary scars.
Histological investigation of a kidney showed atrophy of cyst-like dilated tubules, filled with colloid eosinophilic
masses. Interstitium was characterized by pronounced periglomerular sclerosis. What diagnosis is most probable?
A. *Chronic pyelonephritis with scarring
B. Chronic glomerulonephritis with scarring of kidneys
C. Amyloidosis of kidneys
D. Tuberculosis of kidneys
E. Kidneys at hypertension

94
40. At the patient operated concerning a widespread tumour of an abdominal cavity, the considerable bleeding and
declining of arterial pressure took place. After operation the acute renal insufficiency, resulted in the patient's death,
developed. An autopsy revealed the wide light pink cortical layer, which was clearly delimited from dark red
pyramids. Histological investigation showed the absence of nuclei in epithelium of convoluted tubules,
tubulorrhexis, venous hyperemia. Nuclei of the cells of vascular glomeruli and straight tubules were unchanged.
What pathology of kidneys has developed at the patient?
A. * Necronephrosis
B. Infarct
C. Glomerulonephritis
D. Pyelonephritis
E. Nephrosis
41. A 49-years-old- patient with a 10 days history of dysentery presented with raised body temperature, a pain in
lumbar area, a fever and a considerable quantity of leucocytes in urine. Histological investigation of a puncture renal
biopsy revealed hyperemia, interstitial leukocytic infiltration and numerous inflammatory cells (mainly
neutrophiles) with a desquamated epithelium, filling renal tubules. What complication has developed at the patient?
A. *Acute pyelonephritis
B. Chronic pyelonephritis
C. Pyelitis
D. Glomerulonephritis
E. Necrotic nephrosis
ENDOCRINE SYSTEM DISORDERS
1. A 23-years-old male presented to his physician with excess of the somatotropic hormone, the enlarged nose, lips,
ears, a mandible, hands and feet. What is the most likely diagnosis?
A. *Acromegaly
B. Pituitary nanism
C. Itsenko-Cushing disease
D. Addison's disease
E. Adiposogenital dystrophy
2. A histological investigation of the removed enlarged thyroid gland revealed irregular form and sizes follicles.
The follicular epithelium proliferated both into follicles lumen and outside. Layers of the connective tissue separated
various sites of the thyroid. What is the most likely diagnosis?
A. *Macro-microfollicular goiter
B. Diffuse goiter
C. Autoimmune thyroiditis
D. Follicular carcinoma
E. Riedel's thyreoiditis
3. A 42-year-old man presented with a thyroid gland enlargement in two times. A physical investigation revealed
firm, irregular lobulated gland. Histological investigation showed a diffusive infiltration of thyroid gland by
lymphocytes, plasmocytes with follicles formation and the enhanced growth of a connective tissue. What is the most
likely diagnosis?
A. * Hashimoto's goiter.
B. Endemic goiter.
C. Sporadic goiter.
D. Diffuse toxic goiter.
E. Riedel's thyreoiditis
4. A 52-year-old man with a history of hematogenic tuberculosis presented to the hospital with hyperpigmentation
skins (melanosis) and mucosas, a cachexia, cardiovascular insufficiency. What disease caused such changes?
A. *Addison's disease
B. Pheochromocytoma
C. Simmonds' disease
D. Itsenko-Cushing disease
E. Graves' disease
5. A 48-years-old woman presented with insomnia (inability to get enough sleep, a bad dream), a fatigue,
irritability, exophthalmos (protuberance of eyes) and tachycardia. The thyroid gland was enlarged. She died soon
due to cardiovascular insufficiency. Histological investigation of the thyroid tissue revealed a proliferation
of an epithelium with formation of papillas, a colloid colliquation, lymphocyte and plasmocyte infiltration and
formation of lymphatic follicles with the germinal centres. What is the most likely diagnosis?
A. * Diffuse toxic goiter. (Basedow's disease)
B. Endemic goiter
C. Sporadic goiter
D. Hashimoto's thyreoiditis
E. Benign tumor of thyroid gland

95
6. A 48-year-old male, with a long history of living in the mountain district of Central Asia, presented with a
thyroid gland enlargement, resulted in complicated swallowing. A physical investigation revealed an increasing of a
body mass, slowness, drowsiness, and puffy face. A microscopic study of a thyroid gland showed irregular follicles
with a hypochromic colloid. What of diagnoses is the most probable?
A.,* Endemic goiter.
B. Basedow's goiter.
C. Hashimoto's thyreoiditis.
D. Sporadic goiter.
E. Riedel's thyreoiditis.
6. A 56-year-old woman with a long history of parathyroid glands disease, died from increased renal insufficiency.
A post-mortem revealed a bones deformation of extremities, a vertebral column, and ribs. Bones were fragile, soft
and easily deformed or cat. They had some motley tumorous formations. Kidneys were contracted. Histological
investigation revealed lacunar bone resorption. In addition, a microscopic study demonstrated giant cell granulomas
within the centres of tumorous formations, accumulations of erythrocytes and a hemosiderin. Name the described
disease.
A. *Parathyroid osteodystrophy.
B. Metastases of cancer in a bone
C. Chronic renal insufficiency
D. Paget's disease
E. Myeloma
7. A 42-year-old woman presented with the obesity, chiefly of the trunk and face, steroid diabetes, an arterial
hypertension and secondary dysfunction of ovaries. An autopsy. revealed a hypertrichosis, a hirsutism, strias on a
skin of hips and a abdomen. In the anterior lobe of the pituitary gland a tumour was found. Histological
investigation showed the basophilic adenoma of the pituitary and hyperplasia of a fascicular layer in adrenals. What
of the listed diagnoses is the most likely?
A. * Itsenko-Cushing disease
B. Itsenko-Cushing syndrome
C. Simtnonds' disease
D. Adiposogenital dystrophy
E. Pituitary nanism
8. A histological investigation of a thyroid gland, presented to pathology, revealed a mild atrophy of a parenchyma,
sclerosis, diffusive infiltration of a stroma by lymphocytes and plasmocytes with formation of lymphoid follicles.
What is the most likely diagnose?
A. *Autoimmune thyroiditis
B. Parenchymatous goiter
C. Toxic goiter D; Thyroiditis
E. Riedel's thyreoiditis
9. A 55-year-old woman died from uraemia. A post-mortem revealed a parathyroid gland adenoma, bones
deformation of extremities, a vertebral column, and ribs. Bones were soft, with slight porousness. On a cut they had
a motley pattern with multiple cysts. Histological investigation revealed marked bone remodelling and lacunar
resorption of osteoid and fibrous tissues. What is the most likely diagnosis?
A. *Parathyroid osteodystrophy.
B. Chronic osteomyelitis
C. Osteoporosis
D. Fibrous dysplasia
E. Paget's disease
10. A 62-year-old man with a 14 years medical history of a diabetes mellitus presented with significantly worsened
eye vision for the last two years. What characteristic processes resulted in worsening of the eye vision?
A. *Microangiopathy
B. Macroangiopathy
C. Calcinosis of hyaloid body
D. Dimness of cornea E.
11. A 45-years-old male presented to the hospital with enlarged nose, ears, a mandible and feet. What is the most
likely diagnosis?
A. *Acromegaly
B. Nanism
C. Cerebro-hypophyseal cachexia
D. Adiposogenital dystrophy E.
12. A male patient presented with obesity of the trunk and face. He also had a long medical history of an arterial
hypertension. The death resulted from a hemorrhage in a brain. A morphological investigation revealed a basophilic
adenoma of a pituitary body and hyperplasia of adrenals cortex. What is the most likely preliminary diagnosis?
A. * Itsenko-Cushing disease
B. Diabetes mellitus A. Acromegaly

96
D. Pituitary nanism
E. Adiposogenital dystrophy
13. A thyroid gland of a 48-year-old male was removed at surgery department. Histological investigation revealed
various size follicles which were lined by tall columnar epithelium. The latter one was pilled up at places forming
papillary infoldings. Colloid appeared lightly staining, watery and finely vacuolated. The stroma showed increased
accumulation of lymphoid cells.
A. *.Basedow's goiter. В Hashimoto's thyreoiditis
C. Riedel's thyreoiditis
D. De Quervain's thyroiditis
E. Nodular goiter
14. A 45-years-old male presented to the hospital with gradual enlargement of feet, right hand, nose and lips. The
adenoma of a pituitary body was diagnosed. What is the most likely disease?
A. * Acromegaly
B. Nanism
C. Basedow's disease.
D. Addison's disease
E. Diabetes mellitus
15. A histological investigation of a thyroid revealed a considerable infiltration of a gland by lymphocytes with
formation of lymphoid follicles, a destruction of parenchyma, growth of connective tissue fibers. What is the most
likely disease presented in that case?
A. * Hashimoto's goiter
B. Colloid goiter
C. Endemic goiter
D. Diffuse toxic goiter
E. Parenchymatous goiter
16. A 64-year-old patient died of a cachexia. A post-mortem revealed an adenoma of parathyroid glands, bones
deformation of extremities, a vertebral column, and ribs. Bones were soft, with slight porousness and easily
deformed or cat. What of the listed diagnoses is the most likely?
A. *Parathyroid osteodystrophy.
B. Osteopetrosis
C. Chondrodysplasia
D. Osteomyelitis
E. Fibrous dysplasia
17. A histological investigation of a thyroid gland biopsy, presented to pathology, revealed lymphoid structures
with the germinal (growth) centers among the follicles filled with a colloid. What is the most likely disease
presented in that case?
A. * Hashimoto's goiter.
B. Endemic goiter
C. Sporadic goiter
D. Basedow's goiter
E. Riedel's thyreoiditis
18. An autopsy of the man revealed a bronze coloring of a skin and mucosa of an oral cavity. In addition, caseous
masses were defined in adrenals. What is the most likely disease in that case?
A. * Addison's disease
B. Itsenko-Cushing syndrome
C. Basedow's disease.
D. Acromegaly
E. Pheochromocytoma
19. A 64-year-old man died with symptoms of a hypoglycemic coma. A postmortem revealed reduced, dense
pancreas. Histological investigation of pancreas showed the growth of a connecting tissue, an atrophy of Langerhans
islets. What disease caused such changes in a pancreas?
A. *Diabetes mellitus.
B. Mucoviscidosis.
C. Acute pancreatitis.
D. Cancer of pancreas.
E. Hypoplasia of pancreas.
20. A 64-year-old man, with a medical history of a diabetes mellitus, presented augmented symptoms of renal
failure. What disease caused such changes in a pancreas?
A. *GIomerulosclerosis.
B. Necrosis of renal tubular epithelium.
C. Amyloidosis.
D. Proliferative glomerulonephritis
E. Membranous nephropathy.

97
21. A morphological investigation of a thyroid removed due to thyrotoxicosis struma revealed a diffuse infiltration
of a gland by lymphocytes with a destruction of parenchyma and an area of connective tissue. What is the most
likely diagnose?
A. * Hashimoto's thyreoiditis
B. Anaphylactic reaction
C. Thyrotoxicosis
D. Cancer of thyroid
E. Adenoma
22. A patient presented to surgery with symptoms of hypothyrosis. Macroscopic investigation of the. removed
thyroid gland revealed its considerable enlargement, dense-elastic texture and lobulated surface. Histological
investigation showed the diffuse infiltration of lymphocytes and plasmocytes of gland's parenchyma with formation
of lymphoid follicles. The latter ones had hyperplastic germinal centers with atypia and a metaplasia of a follicular
epithelium. In addition, the sclerotic alterations of a parenchyma were also determined. What is the most likely
diagnose?
A. *Autoimmune thyroiditis
B. Diffuse toxic goiter
C. Colloid goiter
D. Nodular goiter
E. De Quervain's thyroiditis
23. At operation of excision of a diffusive thyroid gland struma a patient died from acute adrenals insufficiency. An
autopsy, besides the atrophy of adrenals, revealed a left ventricle hypertrophy in a heart, a hypertrophy and an
intracellular edema of cardiomyocytes, a serous edema and lymphoid infiltration of a myocardial stroma. What is
the figurative name of a heart pathology presented at post-mortem?
A. *Thyrotoxyc heart
B. Tabby cat heart
C. Cor bovinum
D. Cor pulmonary
E. Brown atrophy of heart
24. An autopsy of a 40-year-old woman revealed the obesity, chiefly of the trunk and face ("moonface" and
"buffalo hump"), strias on a skin of hips and abdomen. She had in her medical records an osteoporosis with
spontaneous fractures of bones, an amenorrhea, an arterial hypertension, a hyperglycemia. What of the listed
diagnoses is the most likely?
A. * Itsenko-Cushing syndrome
B. Addison's disease
C. Friderichsen-Waterhouse syndrome
D. Conn's syndrome
E. Sipple's syndrome
25. A renal puncture biopsy was taken from a patient with elevated blood sugar levels and high amounts of glucose
in his urine. Histological investigation revealed extension of mesangium with focal accumulation of membranous
substances and periglomerular sclerosis of some glomerules. In addition, there were found a hyalinosis and plasmatic
infiltration of arterioles; lymphocytes, histiocytes and some leucocytes infiltration of a stroma; glycogen deposits within
nephrocytes of narrow segment. What is the most likely diagnosis?
A. *Diabetic glomerulonephritis
B. Arteriolosclerotic nephrocirrosis
C. Pyelonephritis
D. Acute glomerulonephritis
E. Subacute glomerulonephritis
PATHOLOGY OF PREGNANCY
1. A 29-year-old woman presented to maternity department with termination of pregnancy at 20 weeks of gestation. The
fetus and fetal membranes were delivered with some blood clots. Histological investigation revealed fetal membranes,
chorion villi and decidual tissue. Name the described pathology of pregnancy.
A. *Spontaneous complete abortion
B. Premature birth
C. Hydatidiform mole
D. Invasive hydatidiform mole
E. Artificial abortion
2. A young woman presented to the hospital with an acute pain. Her uterine tube was removed at gynecology department.
Macroscopic investigation of a tube revealed its local dilation at the middle third and blood clots accumulations within the
lumen. A histological research showed chorion villi, surrounded by areas of erythrocytes with some leucocytes. What is
the most likely diagnosis?
A. *Tubal pregnancy
B. Acute purulent salpingitis
C. Hemorrhage in tube of the uteri
D. Hemorrhagic salpingitis

98
E. Tuberculous salpingitis
3. A 35-year-old woman had an operation of uterine tube removal at gynecology department. A histological research of a
tube's wall reveled chorion villi and decidual cells. What is the most likely diagnosis?
A. *Tubal pregnancy
B. Placental polyp
C. Choriocarcinoma
D. Paper-doll fetus
E. Lithopedion
4. A- post-mortem of a pregnant woman revealed a brain edema, hemorrhagic pneumonia, motley liver with a
hemorrhage. Microscopic study showed disseminated blood clotting of vessels, plural fine necroses, and
hemorrhages in internal organs. In addition, in kidneys there were found cortical necroses and a fibrinoid necrosis of a
tubules epithelium within a nephron. What is the most likely diagnosis?
A. *Eclampsia
B. Pre-eclampsia
C. Septicopyemia
D. Septicemia
E. Acute renal insufficiency
5. A 25-year-old woman with a medical history of recent childbirth presented to gynecology department with
metrorrhagia. The histological investigation of a curettage material revealed a pathologic growth, consisted of fibrin clots,
chorion villi and decidual tissue, undergoing organization. What is the most likely diagnosis?
A. * Placental polyp
B. Glandular hyperplasia of the endometrium
C. Hydatidiform mole
D. Choriocarcinoma
E. Endometritis
DISEASES OF THE REPRODUCTIVE SYSTEM
1. A 45-year-old woman with a medical history of a menstrual cycle disturbance had curettage of the uterine cavity to
obtain endometrial tissue for histopathologic analysis. Microscopic investigation revealed the increased quantity of
the endometrial glands, their form irregularity. Some of glands were dilated or had a saw-like pattern. What is the most
likely diagnosis?
A. * Cystic glandular hyperplasia of endometrium
B. Placental polyp
C. Atypical hyperplasia of endometrium
D. Glandular polyp of endometrium
E. Endometrial adenocarcinoma
2. An autopsy of an elderly man revealed enlarged, soft, elastic, slightly nodular prostate. A gross investigation of a cut
section showed separate nodes parted by layers of a connective tissue. The compressed surrounding prostatic tissue created
a plane of cleavage about them. A microscopic study found the increase of glands quantity. There were variable number of
prostatic nodes and glands in them.
A. * Glandular nodular hyperplasia
B. Fibrpmuscular (stromal) hyperplasia
C. Mixed nodular hyperplasia
D. Adenocarcinoma
E. Undifferentiated carcinoma
3. A 30-year-old woman, with a history of a chronic endocervicitis, passed routine inspection at the gynecologist with no
complaints. A histological investigation of a biopsy, taken from the pars vaginalis neck of the uteri, revealed the high
cylindrical epithelium, secreting mucus and branched out glands underneath. What is the most likely diagnose?
A. * Endocervicosis
B. Squamons metaplasia
C. Glandular metaplasia
D. LeukopJakia
E. Erythroplakia
4. A 68-year-old man presented with difficulty of a miction. The prostate was operatively removed. A
microscopical investigation revealed an increase of glandular and muscular elements numbers. A lobular structure
of prostate was also altered. What process in a prostate is the most probable?
A. * Mixed nodular hyperplasia
B. Glandular hyperplasia
C. Fibromuscular hyperplasia
D. Prostatitis
E. Adenocarcinoma
5. A 42-year-old woman presented with acyclic, plentiful, uterine bleedings. A histological investigation of a
curettage material revealed increased quantity of glands and their cystic dilation. What is the most likely diagnosis?
A. * Hyperplasia of endometrium
B. Atrophy of endometrium
C. Hypertrophy of endometrium

99
D. Metaplasia
E. Organization
6. A 42-year-old woman presented with dysfunctional uterine bleeding. A histological investigation of a
curettage material revealed increased quantity of coiled glands and their cystic dilation. What is the most likely
diagnosis?
A. * Cystic glandular hyperplasia
B. Atrophy
C. Metaplasia
D. Displasia
E. Hypertrophic vegetations
7. A microscopic investigation of an endometrium revealed coiled extended glands with 'saw-' and a 'spin-like'
pattern. A stromal proliferation with hyperplasia of its cells was also determined. What is the most likely diagnosis?
A. * Glandular hyperplasia of endometrium
B. Acute endometritis
C. Leiomyoma
D. Hydatidiform mole
E. Placental polyp
8. A histological investigation of curettage of the cervical canal of the uteri revealed a prismatic epithelium and a
significant amount of gland-like structures. They developed from the prismatic epithelium cambial elements,
originated from the neck of a uterus. What is the most likely diagnose?
A. *Proliferative endocervicosis
B. Simple endocervicosis
C. Endocervicosis, stage of healing
D. Cervical adenomatosis
E. Cervical polyps
9. A 52-year-old woman, with a medical history of the menstrual cycle disturbances and hemorrhages,
had curettage of the uterine cavity. Histological investigation revealed the increased quantity of the endometrial
glands. Many glands had a morphological pattern of a proliferation phase, clumped together and had intraglandular
papillary protrusions. Some glandular cells exhibited the signs of nuclear and cytoplasm polymorphism. What is
the most likely diagnosis?
A. * Atypical hyperplasia of endometrium
B. Cystic glandular hyperplasia of endometrium
C. Glandular hyperplasia of endometrium
D. Glandular polyp of endometrium
E. Adenocarcinoma of endometrium
10. A physical examination of a 68-year-old man, with complains to an acute ischuria (retention or suppression of
the urine), revealed the nodular enlargement of a prostate. A microscopical investigation showed that the majority of
knots were constructed of polymorphic glandular structures, with some retention cysts. The latter ones had thick
secreta. What is the most likely diagnose?
A. *Glandular hyperplasia of prostate
B. Fibromuscular hyperplasia of prostate
C. Hyperplasia of prostate mixed form
D. Adenoma of prostate
E. Carcinoma of prostate
11. A 24-year-old woman, on a 3rd day after childbirth, presented with a diffusive edema and painful palpation of
the right breast. Physical examination revealed hyperemia of the breast and a fervescence. Histological research of
a gland's tissue showed diffusive leucocytes infiltration of stroma, an interstitial edema and hyperemia of
vessels. What is the most likely diagnose?
A. *Acute phlegmonous mastitis
B. Acute apostematous mastitis
C. Acute serous mastitis
D. Chronic purulent mastitis E
12. A 48-year-old woman with a grease bloody flux from her genital tracts had a diagnostic curettage of her uterus
cavity. Microscopical research of the uterus curettage material revealed a thinning of a mucosal layer, reduction of
the endometrial glands number, a fibrosis of a stroma and some lymphoid cells infiltration. What is the most likely
diagnosis?
A. *Chronic atrophic endometritis
B. Acute purulent endometritis D Chronic cystic endometritis
E. Chronic hypertrophic endometritis
13. A gynecologic investigation of the 36-year-old woman revealed bright red brilliant maculae on a uterus
neck. They bleed easily at a touch. Biopsy investigation showed that a tissue sample was covered by a
cylindrical epithelium with papillary growths. There was also a growth of glands within the thickness of a uterus
neck tissue. What pathology of a uterus neck was diagnosed?

100
A. * Pseudoerosion
B. Cervical erosion
C. Endocervicitis
D. Glandular hyperplasia
E. Leukoplakia
14. A 23-year-old woman presents with her menstrual cycle disorder went trough a transvaginal puncture of
Douglas' cul-de-sac resulted in aspiration of blood and some other material. A microscopic investigation of the
puncture fluid revealed blood, some decidual cells and chorion villi. What is the most likely diagnosis?
A. *Ectopic tubal pregnancy
B. Salpingitis
C. Rupture of Fallopian tube
D. Hydatidiform mole
E. Carcinoma uteri
PERINATAL PATHOLOGY
1. A 33-year-old woman gave birth to a dead fetus with gestational age more than
43 weeks. The fetal skin was dry and chap. An autopsy revealed a general
hypotrophy and nuclei of ossification (Beclard's nuclei) in a proximal epiphysis of
tibial and humeral bones. Amniotic fluid, an umbilical cord and placental
membranes were dyed by meconium. Name the period of the described perinatal
pathology?
A The antenatal period
B. The prenatal period
C. The intranatal period
D. The postnatal period
E. The progenesis period

GAMETOPATHY
1. A 3-month-old child died of pneumonia. A macroscopical investigation revealed upward slant of palpebral
fissures of the eyes, a sinking down dorsum of nose, dolichouranic, a low locating of small auricles. An autopsy
revealed a defect of a heart and the main vessels' development. A genetic study showed a chromosome 21 trisomy.
What of the listed diagnoses is the most likely?
A * Down's syndrome
B. Patau's syndrome
C. Shereshevsky-Turner's syndrome (gonadal dysgenesis)
D. Edwards' syndrome E.
AVITAMINOSIS.
1. A 3-year-old child, with history of alimentary dyspepsia, presented with progressing gingival hemorrhage,
petechial hemorrhages on a skin and a mouth mucosa. There was also a peridental infection. The
described changes characteristic for:
A * Avitaminosis С
B. Avitaminosis D
C. Avitaminosis Bl
D. Avitaminosis B6
E. Avitaminosis A
2. A 9-month-old child presented with retarded dentition, the teething order disturbance, a reconfiguration of
the upper jaw in a horizontal direction (dolichouranic). Microscopical investigation of his teeth revealed the
irregular mineralization of the enamel and crumpled enamel prisms. Some of them were rich with vacuoles. There
were also expansion of a predentin zone and a presence of solitary denticles. What is the most likely disease in that
case?
A *Rachitis, early stage
B. Rachitis, late stage
C. Osteomalacia
D. A gout
E. Hypervitaminosis D
3. A 1, 5-year-old child presented with focal thickenings of ribs, wrists, and a curvature of legs. A stomatologist
pointed to retarded dentition, the teething order disturbance, the irregular mineralization of the enamel and
dentine, a reconfiguration of the upper jaw in a horizontal direction (dolichouranic). What is the most likely disease
presented in that case?
A * Rachitis
B. A dystrophic calcification
C. A metabolic calcification
D. A metastatic calcification
E. Osteomalacia

101
INFECTION DISEASES
1. A post-mortem of a patient, with a history of a drug abuse, revealed red-purple papular nodules and blotches on
the skin of his lower extremities (Kaposi's sarcoma). In addition, acute pneumocystosis carinii pneumonia
was also determined. For what disease the given symptoms are characteristic?
A. *AIDS
B. Influenza
C. Measles
D. Diphtheria
E. Anthrax
2. A 48-year-old woman died of pulmonary insufficiency. A post-mortem revealed the enlarged lung with the dark
red and pinkish-yellow colour areas ("the big motley lung") and necrotic tracheobronchitis. What is the preliminary
diagnosis in that case?
A. * Influenzal pneumonia
B. Croupous pneumonia
C. Caseous pneumonia
D. Fibrosing alveolitis
E. Measles virus pneumonia
3. A 44-year-old man died of pulmonary insufficiency and severe intoxication. A post-mortem investigation of a
lungs cut surface revealed a motley pattern with plural focal hemorrhages and the zones of emphysema. Histological
study showed a hemorrhagic bronchopneumonia with abscesses formation. A cytoplasm of bronchial epithelium
cells had eosinophilic and basophilic inclusions. What is the most likely diagnosis?
A. * Influenza
B. Parainfluenza
C. Adenoviral infection
D. Respiratory syncytial infection
E. Staphylococcal bronchopneumonia
4. A young man presented with rash in the form of roseolas and small hemorrhages (petechias) on a skin of a
stomach, thorax and petechial rash on eyes conjunctiva. Lesion of his brain soon resulted in his death. A
microscopical investigation revealed in destructive-proliferative endo-trombovasculitis in medullar oblongata and
pons of a brain, also in a skin, kidneys and myocardium. What is the most likely disease?
A. *Epidemic typhus
B. Sepsis
C. Periarteritis nodosa
D. Systemic Lupus Erythematosus
E. Brucellosis
5. A 67-year-old man died of pulmonary insufficiency. A post-mortem investigation of a lungs cut
surface revealed multiple, bright, red, deaerated 2,5 cm foci with irregular form and inaccurate borders. A
microscopic study showed erythrocytes and some lymphocytes within alveoli. The described changes are
characteristic for:
A. * Influenzal pneumonia
B. Croupous pneumonia
C. Hemorrhagic infarction of the lung
D. Measles virus pneumonia
E. Aspiration pneumonia
6. A 42-year-old man, with a history of an acute respiratory disease, died of pulmonary insufficiency. A post-
mortem investigation revealed fibrinous -hemorrhagic inflammation in a larynx and trachea mucosa,
destructive panbronchitis and enlarged lungs. Multiple abscesses, hemorrhages and necroses created a motley
appearance of a pulmonal tissue. What of the listed diagnoses the most likely?
A. * Influenza
B. Parainfluenza
C. Respiratory syncytial infection
D. Measles
E. Adenoviral infection
7. A post-mortem of a 40-year-old patient revealed a pneumocystosis carinii pneumonia, Kaposi's sarcoma and B-
cell lymphoma. Medical records suggested that he actively practiced unsafe sexual intercourse with multiple
partners at his life. What is the most likely diagnosis?
A *A human immunodeficiency virus infection, AIDS'Stage
B. A human immunodeficiency virus infection, pre- AIDS stage
C. A secondary immunodeficiency as a result of primary B-cell lymphoma
D. A secondary immunodeficiency as a result of Kaposi's sarcoma E.
8. A 38-year-old man presented with an acute fever, elevated temperature to 40°C, a headache, a cough and
dyspnea. He died on the 5th day of a disease. An autopsy revealed enlarged lungs with a motley pattern of a
pulmonary tissue - "the big motley lung". For what disease the described signs are characteristic?

102
A.* Influenza
B. Adenoviral infection
C. Croupous pneumonia
D. Respiratory syncytial infection
E. Bronchiectasis
9. A 35-year-old narcomaniac (drug abuser) with a history of HTV-infection died at the hospital. A post-mortem
revealed that both lungs were dense, dark-red-grey color, with little air. Histological study showed that interalveolar
septs were infiltrated by lymphocytes. Some alveolocytes were transformed to the big cells, with a centrally located
round nucleus with a light rim of cytoplasm ("an owl's eye"). What opportunistic infection caused pneumonia in that
case?
A *Cytomegalovirus
B. Pneumocystis carinii
C. Atypical mycobacterium
D. Herpes-virus
E. Toxoplasma
10. A 48-year-old male presented with a sustained high fever, severe headache, a dyspnea, palpitation. A physical
investigation revealed a pediculosis, a rash in the form of roseolas and small hemorrhages (petechias) on his thorax
skin. In addition, decubituses of legs and foot gangrene were also determined. He died of a heart failure. A
histological study showed in the medullar oblongata a hyperemia, stasis, perivascular plasmocytes infiltrates and a
proliferation of a microglia (Popov's granuloma). What is the most likely diagnosis? A *Epidemic typhus
B. Antrax meningocephalitis
C. Meningococcal meningitis
D. Typhoid fever
E. Rubella
HA patient died as a result of a heart failure. Macroscopically traces of a preexisted rash in the form of maculae and
spots were recognized. A gross investigation revealed decubituses at the areas of breeches and spinous processes of
vertebra. Microscopical study showed a destructive-proliferative endo-trombovasculitis and Popov's granulomas in
the central nervous system, in a skin, and adrenals. An interstitial myocarditis was diagnosed in his heart. What is
the most likely disease? A * Epidemic typhus
B. Q fever
C. Typhoid fever
D. Periarteritis nodosa
E. HIV-infection
12. A post-mortem of a woman revealed a septicopyemia with metastatic abscesses of lungs and kidneys;
pyosalpinx, a purulent peritonitis, a cachexia and lymphadenopathy. Medical records indicated HIV- positive
blood test results within Jast 5 years. What clinical period of AIDS corresponds with the given pathomorphologic
signs?
A. *Acquired immunodeficiency syndrome (AIDS)
B. Incubation period
C. Persisten generalised lymphadenopathy
D. Pre-AIDS (AIDS -related complex)
E. Reactive lymphadenopathy
13. A gross investigation of a brain at a post-mortem revealed the edema, a plethora, and fine hemorrhage in
medulla oblongata. A microscopical investigation showed a chromatolysis, hydropy and a necrosis of nervous cells.
In addition, the eosinophilic intracytoplasmic formations (little bodies the Babesh- Negri bodies) were defined in
pyramidal neurons of the hippocampus. What diagnosis answers the described morphological implications?
A. * Rabies
B. Meningococcal meningitis
C. Encephalitis
D. Encephalomyelitis
E. Brucellosis
14. A post-mortem investigation of a 42-year-old man revealed serous -hemorrhagic inflammation in a
larynx and trachea mucosa, a focal pneumonia, areas of atelectases and acute lungs emphysema. Histological
research showed a vacuolar dystrophy and loss of cilia within the epithelium layer, an exfoliating of cells with
oxyphile inclusions. Such changes in lungs are caused:
A. * Influenza viruses
B. Parainfluenza viruses
C. Measles virus
D. Adenoviruses
E. RS-viruses
15. A 66-year-old man presented with acute tracheitis and bronchitis, soon resulted in bronchopneumonia. He died
on the 12th day at the hospital from the pulmonal-heart failure. An autopsy revealed fibrinous -hemorrhagic
inflammation in a larynx and trachea mucosa, enlarged lungs. On a cut section lungs had a motley pattern due to

103
areas of pneumonia, hemorrhages in lung parenchyma, acute abscesses and atelectases. In addition, circulative
disturbances and dystrophies were determined in internal organs. What is the most likely diagnosis?
A. *Influenza, severe form
B. Influenza, moderately severe form
C. Parainfluenza
D. Respiratory syncytial infection
E. Adenoviral infection
16. A post-mortem of a patient, who died of a HIV- infection, revealed alterations of his brain. A histological
investigation determined in subcortical white matter, midbrain and a brain stem fine, perivascular necroses,
microglial nodules with multinuclear giant cells, focal gliosis and fibrosis. Name the most likely lesion of the central
nervous system?
A. *Subacute meningocephalitis
B. Metabolic encephalopathy
C. Cytomegaloviral encephalitis
D. Primary lymphoma of CNS
E. Vacuolar myelipathy
17. A patient presented to his physician with expressed immunodeficiency, lymphopenia with change of parity
T-helpers to T- suppressors, the lesion of a skin of the lower extremities. The cutaneous pathology was characterized
by plural tumorous nodules, dome-shaped purple plaques, involved in skin breakdown with resulting fungating
lesions. A histological investigation of the skin's biopsy showed the neoplasm of blood vessels, a dilatation of
capillaries, slit-like blood-filled vascular spaces. What is the most likely diagnosis?
A. *Kaposi's sarcoma
B. Basalioma
C. Derraatomycosis
D. Phoma of skin
E. Inflammatory dermatitis
ISA man, with a history of frequent drinking non- boiled water from the river, has developed vomiting and
explosive 'rice-water' diarrhea. The abdominal pains were absent; the body temperature did not rise. The intestine
biopsy revealed a plethora, a marked swallowing, an edema of enterocytes, and some infiltration of villi by
lymphocytes, plasmocytes, and few leucocytes. What is the most likely disease?
A. *Cholera
B. Salmonellosis
C. Typhoid fever
D. Dysentery
E. Lambliasis
CHILDHOOD INFECTION
1. An autopsy of 3-year-old child revealed hyperemic, turbid arachnoids, which had a form of yellowish-greenish
'cap'. At microscopic investigation a characteristic findings of arachnoids thickening, hyperemia, purulent
with fibrin exudate infiltration was present. What is the most likely diagnosis?
A. *Meningococcal infection
B. Tuberculosis
C. Anthrax
D. Ifluenza
E. Measles
2. A child presented with catarrhal conjunctivitis, pharyngitis, a laryngotracheobronchitis. A
gross investigation of the oral cavity revealed white spot on the buccal mucosa opposite the first molar. In addition,
a blotchy and papular rash appeared on the patient's face, trunk and extremities later. What is the most likely
diagnosis?
A. * Measles
B. Scarlet fever
C. Meningococcal infection
D. Epidemic typhus
E. Influenza
3. A 4-year-old child presented to the hospital with a fever and a sore throat. An oral cavity examination revealed
an edema of a soft palate and grey membranes on tonsils. These membranes left a raw hemorrhagic surface when
forcibly removed. What the most likely diagnosis is among listed below?
A. *Diphtheria fauces
B. Simonovsky-Vensans' angina
C. Lacunar angina
D. Infectious mononucleosis
E. Necrotic angina
4. An autopsy of 11-year-old child revealed plural hemorrhages, mainly in skin of breeches and lower extremities,
serous and mucosas, and also in a brain. Gross investigation determined a focal necrosis and massive hemorrhages

104
in adrenals, a necrotic nephrosis in kidneys, a purulent arthritis, an iridocyclitis and vasculitis. Choose the correct
diagnosis.
A. *Meningococcemia
B. Epidemic typhus
C. Periarteritis nodosa
D. Systemic lupus erythematosus
E. Radiation syndrome
5. A 7-year-old child presented with raised body temperature up to 38,5°, rhinitis, conjunctivitis, and cough. A
physical investigation revealed blotchy and papular skin rash and whitish bran-like rashes in a buccal mucosa. On
the fourth day appeared a dyspnea and wet pulmonary rhonchuses. Soon, difficulty of breath resulted in patient
death from asphyxia. What is the most likely diagnosis?
A. *Measles
B. Meningococcemia
C. Influenza
D. The pulmonary form of a mucoviscidosis
E. Pneumonia
6. A 7-year-old boy presented to the hospital with an acute catarrh in pharynx and tonsils which had extended on a
mouth mucosa, tongue and fauces. A tongue was fiery red (crimson red) with prominent papillae ('raspberry' or
'strawberry' tongue). A gross investigation of tonsils showed superficial necroses. A casting-off necrotic tissue
resulted in ulcers formation. Cervical lymphatic nodules were enlarged. A punctuate erythematous rash was
determined over the trunk, arms and legs. Face was also involved but a small area about the mouth remained
unaffected to produce circumoral pallor. What is the most likely diagnosis?
A. * Scarlet fever
B. Diphtheria
C. Angina
D. Measles
E. Meningococcal nasopharyngitis
7. An autopsy of 4-year-old child revealed hyperemic, yellowish-greenish arachnoids, infiltrated by
purulent and fibrinous exudate resembled a 'cap'. What is the most likely diagnosis?
A. * Meningococcal meningitis
B. Tubercular meningitis
C. Influenzal meningitis
D. Anthrax meningitis
E. Epidemic typhus meningitis
8. An autopsy of a child revealed a necrotic tonsillitis, neck phlegmon, purulent otitis and purulent meningitis.
These alterations are the most typical for:
A. * Septic scarlet fever
B. Meningococcal infection
C. Toxic scarlet fever
D. Fauces diphtherias
E. Otogenic sepsis
9. A child presented with acute a catarrhal nasopharyngitis. In 2 days he died. An autopsy revealed arachnoids
were markedly hyperemic, edematous, infiltrated with turbid, thick, yellowish-greenish liquid. A brain was swelled,
cerebellum tonsils were enlarged in volume, and sulcus of the cerebrum impaction was well defined. The described
changes are the most typical for:
A. * Meningococcal infection
B. Influenza
C. Whooping cough
D. Diphtheria
E. Measles
10. A 5-year-old child presented with raised body temperature, punctulated rash, and conjunctivitis. He died of the
superimposed pneumonia. A histological research of lungs revealed endo - mezo - panbronchitis and giant-cell
pneumonia. For what disease the presented changes are characteristic?
A. *Measles
B. Scarlet fever
C. Croupous pneumonia
D. Diphtheria
E. Smallpox
11. Л 43-year-old male, with a history of kidney transplantation, followed by immunosuppressive therapy, died of
intoxication. A histological investigation of his lungs, kidneys and pancreas revealed strikingly enlarged (giant) cells
with basophilic cytoplasm and large nucleus. The violet intranuclear inclusions surround by a small clear halo,
reminded an eye of an owl. What is the most likely diagnosis?
A. *Cytomegaly

105
B. Tuberculosis
C. Syphilis
D. Lepra
E. Plague
13. 11 -year-old child presented with a body temperature up to 40°C, an acute headache, vomiting, anxiety and a
fever. In four days appeared a hemorrhagic skin rash, an oliguria and adrenal insufficiency, resulted in patient's
death. A bacteriological study of smear preparation from a pharynx revealed a meningococcus. What form of
disease is found out?
A. Meningococcemia
B. Meningococcal meningitis
C. Meningocephalitis
D. Meningococcal nasopharyngitis E.
12. A young 19-year-old man, with an acute onset of disease, died of an infectious-toxic shock. An autopsy revealed
the enlarged tonsils, covered with grey-white membranes, which extended on the palatine arch. In addition, an
edema of the neck soft tissues determined. Histological investigation showed the necrosis of an epithelium of tonsils
and the palatine arch. Tissues underneath were infiltrated by a fibrinous exudate, which formed massive
stratifications on a surface. What is the most likely diagnosis?
A. * Diphtheria
B. Scarlet fever
C. Adenoviral infection
D. Infectious mononucleosis
E. Staphylococcal infection
14. An oral cavity examination of a 4-year-old child, revealed a hyperemia of the fauces and tonsils mucus
membranes. Gross investigation showed enlarged tonsils, covered by dense, whitish-yellowish membranes. They
left a raw surface, with deep defect, when forcibly removed. Soft tissues of a neck were edematous, regional lymph
nodes were enlarged and painful. What the most likely diagnosis is among listed below?
A. * Diphtheria
B. Scarlet fever
C. Measles
D. Parotitis
E. Adenoviral infection
15. An autopsy of 2-year-old child revealed hemorrhagic enanthesis (skin rash), mild hyperemia and edema of
nasopharynx mucous, fine hemorrhages in mucosas and internal organs, severe dystrophy in a liver and
myocardium, an acute necrotic nephrosis,'massive hemorrhages in adrenals. What disease is the most likely to
present such symptoms?
A. * Meningococcal infection
B. Scarlet fever
C. Diphtheria
D. Measles
E. Epidemic typhus
16. A child presented to a hospital with an acute fever, sore throat and swelling of submaxillary (submandibular)
lymph nodes. A gross investigation of fauces revealed the edematous and slightly hyperemic mucous
membrane, enlarged tonsils, covered by the grayish-whitish membranes. They left a raw surface, when forcibly
removed. What is the most likely diagnosis?
A. * Diphtheria
B. Catarrhal angina
C. Scarlet fever
D. Meningococcal infection
E. Measles
17. A 7-year-old girl developed an acute disease. She presented to the hospital with a sore throat, and a fever,
accompanied with a widespread rash. A physical examination revealed severe hyperemia of a pharynx, a 'raspberry'
tongue, and enlarged bright red tonsils with some grayish and yellowish dim areas. These zones also extended on
peritonsillar tissues. Submandibular lymph nodes were enlarged. For what disease the given changes are
characteristic?
A. * Scarlet fever
B. Measles
C. Fauces diphtheria
D. Larynx diphtheria
E. Meningococcal nasopharyngitis
18. A 9-year-old child developed a sharp pain at swallowing and a neck edema. His body temperature had risen to
39°C. A motley-red, punctulated , widespread rash appeared. A physical examination revealed severe hyperemia of
a pharynx ('fiery pharynx') and a 'raspberry' tongue. The enlarged bright red tonsils presented with some grayish
necrotic zones. What is the most likely diagnosis?

106
A. * Scarlet fever
B. Meningococcal nasopharyngitis
C. Diphtheria
D. Influenza
E. Measles
19. A 6-year-old boy developed pain at swallowing and raised body temperature. On a second day of the disease, a
physical investigation revealed the appearance of the fine, densely set, widespread skin rash with a small unaffected
circumoral area. An oral cavity examination showed the bright reddening of a pharynx, enlarged tonsils and
raspberry-red tongue. What is the most likely diagnose?
A. *Scarlet fever.
B. Adenoviral infection.
C. Fauces diphtheria.
D. Measles.
E. Streptococcal angina.
20. An 8-year-old child presented with raised body temperature up to 39° C, rhinitis, conjunctivitis, and cough. A
physical investigation revealed blotchy skin rash and whitish rashes on a buccal mucosa. Suddenly appeared pant
resulted in asphyxia, followed by patient death. What is the most likely diagnosis?
A. *Measles
B. Scarlet fever
C. Diphtheria
D. Meningococcal nasopharyngitis
E. Influenza
21. A 3-year-old girl presented with a sharp pain at the swallowing, a marked neck edema, and the body
temperature raised to 40° C. A gross investigation of tonsils revealed the grayish-yellowish membranes, which left a
raw surface with bleeding ulceration, when forcibly removed. What is the most likely diagnosis?
A. *Diphtheria
B. Scarlet fever
C. Measles
D. Meningococcal nasopharyngitis
E. Influenza
22. A child died of a spontaneous pheumothorax. A post-mortem revealed a collapse and interstitial emphysema of
the right lung, focal atelectases of lungs, a punctual hemorrhage in a brain and a conjunctiva of eyes. What is the
most likely diagnosis?
A.* Whooping cough
B. Bronchopneumonia
C. Measles
D. Diphtheria of respiratory tracts
E. Scarlet fever
23. A child presented to the clinic with paralysis of a phrenic nerve and dysphagia, resulted in his death soon.
An autopsy revealed ulcers of tonsils with a cuticularization, a hemorrhage in adrenals, and flaccidity of a
myocardium. Specify disease for which the described changes and complications are characteristic.
A.* Diphtheria
B. Scarlet fever
C. Agranulocytosis
D. Measles
E. Poliomyelitis
24. A child complains of the general weakness, a pharyngalgia at swallowing and nausea. Objectively, a bright
reddening revealed in fauces; tonsils were enlarged. Later, the fine, densely located maculae, size about papaverous
grain, appeared on a neck, in the top of a back and a breast. Microscopical investigation showed in a mucosa and
tonsils tissue an acute plethora and grayish, dim areas of necroses. On the periphery of the necrotic zones, within the
edema and fibrinous inflammation areas, the chains of streptococcuses were defined. What is the most likely
diagnosis?
A. * Scarlet fever
B. Diphtheria
C. Measles
D. Influenza
E. Meningococcal nasopharyngitis
25. A 7-year-old child presented to the hospital with a punctuated bright-pink rash on hyperemic skin of a forehead,
a neck, in the bottom of abdomen and popliteal fossa. A nasolabial triangle remained pale. Physical investigation of
a stomato-pharynx revealed circumscribed bright red hyperemia of mucosa, 'raspberry' tongue. His tonsils were
friable and edematous; lacunas filled with pus. Lymph nodes of a neck were enlarged, dense and painful. What is the
most likely diagnosis?
A. *Scarlet fever

107
B. Rubella
C. Whooping cough
D. Diphtheria
E. Infectious mononucleosis
26. A child with a history of an acute catarrh of the upper respiratory tract, conjunctivas and blotchy, papular skin
rash, died of pulmonary insufficiency. A histological investigation revealed an interstitial pneumonia with focal
serous macrophageal inflammation and set of giant cells within alveoli walls. What is the most likely diagnosis?
A. *Measles
B. Influenza
C. Parainfluenza
D. Adenoviral infection
E. Cytomegaly
27. A 4-year-old child died of respiratory insufficiency as a result of a respiratory musculation paralysis.
Histological investigation revealed a hyperemia of the moracal part of a spinal cord, the unclear pattern of the grey
substance, punctual hemorrhages and fine sinking down sites of an encephalomalacia. In addition, there was an
inflammation with a neuroglia proliferation around and by the necrotic neurons. What is the most likely diagnosis?
A. *Poliomyelitis
B. Meningococcal infection
C. Cytomegaly
D. Toxoplasmosis
E. Adenoviral infection
BACTERIAL INFECTIONS
1. An autopsy of a 54-year-old man revealed a fibrinous inflammation at colon. What is your diagnosis? A
*Dysentery
B. Amebiasis
C. Typhoid fever
D. Cholera
E. Balantidiasis
2 A patient, the inhabitant of a seaside city in the south of the country, presented to the hospital with severe
diarrheas and vomiting, resulted in his death from dehydration. An autopsy revealed an acute
gastroenteritis with serous-desquamative inflammation in a small bowel. Choose the most likely diagnosis.
A *Cholera
B. Bacterial dysentery
C. Typhoid
D. Amebiasis
E. Salmonellosis
3. A 52-year-old woman died in surgery department of a diffuse purulent peritonitis. A post-mortem revealed in a
distal part of a small intestine enlarged Peyer's patches, which projected into the intestine's lumen. There surface was
covered by brownish-greenish scab. In the centre of some patches were found deep defects, extended to a serous
layer. For what disease described intestinal changes are characteristic? A *Typhoid
B. Dysentery
C. An intestine tuberculosis
D. Crohn's disease
E. Staphylococcal enteritis
4. A 38-year-old man died of diffuse fibrinopurulent peritonitis. A post-mortem revealed an ulcer in a small bowel,
which replicated a form of a Peyer's patch and perforated the intestine's wall. Microscopical investigation showed an
alteration of lymphoid tissue pattern with replacement of lymphocytes by proliferating monocytes, followed by
granulomas formation. The complication of what disease resulted in patient's death? A *Typhoid.
B. Dysenteries.
C. Cholera.
D. Brucellosis
E. Nonspecific ulcerative colitis.
5. An autopsy of a 48-year-old man, with a history of typhoid fever, revealed groups of enlarged lymphoid follicles
in small bowel. The follicles protruded above the mucosal surface; they had a grey-red coloring, juicy appearance
and a surface, which reminded gyri and sulci. Microscopical investigation showed granulomas formation. Specify,
what of the listed stages of typhoid is the most likely?
A *Medullar swelling.
B. Necrotic.
C. Formation of ulcers.
D. Pure ulcers.
E. Healing.
6. An autopsy of a child, with a history of diarrhea, revealed an exicosis and a widespread fibrinous colitis. A
mucosal smear imprint study showed gram-negative rod bacteria. What is your diagnosis?

108
A *Dysentery
B. Cholera
C. Staphylococcal intestinal infection
D. Typhoid
E. Salmonellosis
7. A patient presented with a sustained fever, enlarged and painful spleen. On the 10th day of a disease a rash of flat,
rose-colored spots appeared. On the 21st hospital day he died after developing peritonitis. An autopsy revealed in the
ileum deep ulcers within the area of necrotized group of lymphoid follicles (Peyer's patches). One of ulcers was
perforated; there was a flbrinopurulent diffuse peritonitis. What is the most likely disease? A *Typhoid
B. Dysentery
C. Intestine amebiasis
D. Cholera
E. Salmonellosis
8A post-mortem of a 57-year-old male, with a history of typhoid, revealed a dense, fragile whitish-yellowish color
of the rectus abdominis and a hip muscles. They reminded a stearin candle. The described changes are the result of:
A *Zenker's necrosis
B. Fibrinoid necrosis
C. Caseous necrosis
D. Colliqative necrosis
E. Apoptosis
9. Histological investigation of a rectum revealed large areas of necrosis in mucosa. Necrotic masses were
impregnated by fibrin, resulting in membrane formation. Mucous and submucous on periphery of necrotic
zones were hyperemic, edematous, with hemorrhages and leukocytic infiltration. What is the most likely
diagnosis?
A *Dysentery
B. Cholera
C. Amebiasis
D. Typhoid
E. Salmonellosis
10. A patient died of a diffuse peritonitis. An autopsy revealed in the distal part of a small bowel multiple oval
ulcers with rounded edges, which were parallel to the longitudinal axis of the intestine. A bottom of ulcers was
clean, smooth and glitter. Base of the ulcers presented either by muscular layer of ileum or its serosa. Two ulcers
had perforation apertures 0,5cm in diameter each. What disease resulted on a death of a patient?
A * Typhoid
B. Dysentery
C. Cholera
D. Tuberculosis
E. Epidemic typhus
11. A 44-year-old man presented to a surgery department with diffuse flbrinopurulent peritonitis. An
operation revealed an ulcer of ileum with a perforation. An ulcer closure, followed by abdomen cavity
drainage was performed. Four days later patient died. At post-mortem iliac ulcers of Peyer's patches, parallel to the
longitudinal axis of the intestine, were found. A base of the ulcers was presented by serosa. For what disease such
changes are characteristic? A *Typhoid
B. An intestine tuberculosis
C. Dysentery
D. Intestinal tumours
E. An intestine amebiasis
12 A man, with 4 weeks history of the typhoid, presented with symptoms of 'acute abdomen'. He died soon of
intoxication. At post-mortem a pathologist found a perforation of iliac wall and diffuse purulent peritonitis. For what
stage of disease the described complication is characteristic? A * Stage of pure ulcers
B. Stage of a medullar swelling
C. Necrotic stage
D. Stage of healing of ulcers
E. Bacteriemia
13. A 42-year-old patient, with a history of dysentery, presented with symptoms of paraproctitis. What is the most
probable stage of local changes presented in that case?
A * Formation of ulcers
B. Fibrinous colitis
C. Follicular colitis
D. Catarrhal colitis
E. Stage of healing

109
14. A child presented with diphtheria and died on the 2nd week of the disease from an acute heart failure. A post-
mortem revealed numerous fine centres of a myolysis, accompanied with some perifocal lymphoid infiltration. What
became a cause of death?
A * Myocarditis, caused by bacterial ectotoxin
B. Bacterial myocarditis
C. Septic myocarditis
D. Myocardial infarction
E. Metabolic myocardial necrosis
15 A 39-year-old patient died on the 4th day after operation concerning perforation of a colon wall, resulted in
diffuse purulent peritonitis. An autopsy revealed the mucosa of a colon wall thickened, covered by fibrinous
membrane. Some single ulcers got on different depth. Histological study of a colon wall showed a necrosis of
mucous, a fibrinous inflammation, infiltration by leucocytes with focuses of hemorrhages. What complication of
disease became a cause of patient's death?
A *Dysentery.
B. Typhoid.
C. Nonspecific ulcerative colitis.
D. Crohn's disease
E. Amebiasis.
16 A 45-year-old man presented in the infectious diseases hospital with a profuse diarrhea, exicosis, acute decline of
his body temperature. He died soon of an uremia. An autopsy revealed in his small bowel lumen a colorless liquid in
the form of rice broth and edematous mucosa. Microscopical investigation of a small bowel showed a hyperemia of
vessels, a focal hemorrhage, a desquamation ofenterocytes, a hypersecretion of a beaker (goblet) cells and lympho-
leucocytes infiltration of a mucosal stroma. What is the most likely diagnosis? A *Cholera
B. Salmonellosis
C. Dysentery
D. Typhoid
E. Crohn's disease
17. An autopsy of 47- year-old man revealed in sigmoid and rectum plural red ulcers with irregular form, A mucous
between ulcers was covered by dirty grey membrane. What is the most likely etiology of the disease?
A *Shigella
B. Ameba
C. Mycobacterium tuberculosis
D. Salmonella
E. Staphilococcus
18. The worker of a cattle-breeding farm died in 4, 5 months of the disease beginning. A post-mortem revealed in
his internal organs the granulomas consisted of epithelioid, giant Pirogov- Langhans cells, plasmocytes and
eosinophyles. There were many vessels within a granuloma. A histological investigation showed a productive-
destructive vasculitis in the vessels of internal organs. What is the most likely disease?
A * Brucellosis
B. Anthrax
C. Tuberculosis
D. Epidemic typhus
E. Lepra
19. The worker of a stockbreeding farm presented to the hospital with acute disease and died soon of intoxication.
An autopsy revealed the enlarged, flaccid spleen, which had a dark-cherry colouring of its cut surface. The scrape
of the spleen's pulp was plentiful. Arachnoids of the brain's fornix and base were edematous, impregnated by blood,
had a dark red colouring ("cardinal's hat"). Microscopical investigation found the serous- hemerrhagic inflammation
of brain's membranes and tissues with destruction of fine vessels walls. Diagnose a disease.
A * Anthrax
B. Tularemia
C. Brucellosis
D. Plague
E. Cholera
20. An autopsy of a 53-year-old man revealed few 4-5cm ulcers. The margins were elevated above the mucosa. The
walls of ulcers were covered by the yellowish-grayish crumble masses. The Widal test was positive. Diagnose a
disease?
A * Typhoid
B. Paratyphoid
C. Recurrent typhus
D. Dysentery
E. Crohn's disease

110
21. A histological investigation of a myocardium revealed fatty dystrophy and plural necroses of cardiomyocytes,
few focal cellular interstitial infiltrates. A medical record showed that a child had diphtheria, resulted in the heart
failure. What kind of myocarditis is the most likely?
A * Alterative myocarditis
B. Diffusive exudative myocarditis
C. Focal exudative myocarditis
D. Intersticial myocarditis
E. Granulomatous myocarditis
22. An autopsy of a 45-year-old man, who died on the 5th day of typhoid fever, revealed groups of enlarged,
hyperemic lymphoid follicles in small bowel. The follicles protruded above the mucosal surface; their surface
reminded brain's gyri and sulci. Microscopical investigation showed hyperemia, edema and granulomas formation.
Specify, what period of local changes of typhoid is the most likely?
A * Stage of medullar swelling
B. Necrotic stage
C. Stage of healing
D. Stage of pure ulcers
E. Stage of ulcers' formation
23. An autopsy 54-year-old man revealed markedly dilated lumen of a small bowel filled with a liquid, which
reminded "rice broth". The intestine wall was edematous with multiple petechial hemorrhages. For what infectious
disease the described enteritis is characteristic?
A *Cholera
B. Dysentery
C. Salmonellosis
D. Amebiasis
E. Typhoid
24. A histological investigation of a skin biopsy showed serous-hemorrhagic inflammation and a focus of necrosis.
His medical records revealed the beginning of the disease from a small red maculae formation with the bubble in the
centre, filled by serous-hemorrhagic liquid. Subsequently the central part became black. What disease is the most
probable? A *Carbuncle at a malignant anthrax
B. Actinomycosis of skin
C. Allergic dermatitis
D. Streptococcal carbuncle
E. Chemical dermatitis
25. A 48-year-old patient with severe typhoid developed an acute renal failure, resulted in a lethal outcome of a
disease. An autopsy revealed enlarged and swollen kidneys. On a cut section, the cortex was pale grey, while
pyramids were dark red. A histological investigation showed that in the majority of tubules the lumen was narrowed,
epithelial Cells were enlarged and lost their nuclei; glomeruli were collapsed. A renal stroma was edematous with a
small leukocytic infiltration and fine hemorrhage. What pathology of kidneys presented in that case?
A *Necronephrosis
B. Acute pyelonephritis
C. Acute glomerulonephritis
D. Pyonephrosis
E. Hydronephrosis
26. An autopsy revealed groups of enlarged, hyperemic lymphoid follicles in small bowel. The follicles protruded
above the mucosal surface; their surface reminded brain's gyri and sulci. The cut surface of follicles was succulent;
with gray-red coloring. Microscopical investigation showed monocytes and histiocytes proliferation.
Macrophages formed aggregates (typhoid granulomas). Name a morphological stage of typhoid.
A *Medullar swelling
B. Necrotic
C. Formation of ulcers
D. Pure ulcers
E. Healing
27. A 48-year-old butcher died of a sepsis. A gross investigation of his right cheek revealed a dense, dark red, 6 cm
in diameter, corn-shape infiltrate with a black crust in its centre. The right half of his face and neck were edematous
and solid. A microscopical investigation of a lesion showed the peracute serous- hemorrhagic inflammation with
epidermal and adjacent tissue necrosis within the central zone of an infiltrate. What is the most likely diagnosis?
A * Anthrax
B. Plague
C. Tularemia
D. Phlegmon of a neck
E. Furuncle
28. An autopsy of a 45-year-old man revealed in his small bowel the enlarged, hyperemic groups of lymphoid
follicles. The follicles protruded above the mucosal surface; their surface reminded child's brain gyri and

111
sulci. Microscopical investigation showed monocytes, histiocytes and reticular cells proliferation.
Macrophages formed aggregates (typhoid granulomas), which replaced lymphocytes. For what disease
the described changes are characteristic?
A * Typhoid
B. Cholera
C. Dysentery
D. Salmonellosis
E. Amebiasis
29. A 38-year-old patient was on mountain pastures and then presented to the hospital in severe condition, with
high body temperature. A physical investigation revealed the lymph nodes were markedly enlarged, soldered to
surrounding tissue, motionless, a skin over them was red and very painful. A microscopical study of a lymph node
showed the peracute serous-hemorrhagic inflammation. For what disease it is characteristic?
A * Plague
B. Tularemia
C. Syphilis
D. Brucellosis
E. Anthrax
30. A 39-year-old milkmaid died of cardiovascular insufficiency. A post-mortem reveled in the aortal valve a
polypous -ulcerative endocarditis. A microscopical study of the valve showed the polymorph cellular inflammatory
infiltration, zones of destruction and thrombuses with the organization. In addition, a sclerosis with granulomas,
consisted of randomly located epithelioid, giant, plasmatic, eosinophyie cells were diagnosed within a
stroma of a myocardium. A vasculitis was also determined in the heart vessels. For what disease the described
changes in heart are characteristic?
A * Brucellosis
B. Chronic sepsis
C. Rheumatic disease
D. Systemic lupus erythematosus
E. Systemic scleroderma
31. A 42-year-old woman, who worked at a fur atelier, suddenly died. An autopsy revealed dark red, impregnated
by blood arachnoids of the brain's fornix and base, which reminded a red cap. Microscopical investigation
found the serous-hemorrhagic inflammation of brain's membranes and tissues with necrosis of fine vessels walls
and numerous hemorrhages. What is the most likely diagnosis?
A * Anthrax
B. Cerebral hemorrhage in case of hypertension disease
C. Cerebral hemorrhage due to trauma
D. Meningococcal infection
E. Tubercular leptomeningitis
32. An autopsy of an 8-year-old child revealed in his colon multiple irregular form and various depth defects with
uneven edges. In addition, there were grey-white membranes intimately connected with tissues underneath. What is
the most likely diagnosis? A * Dysentery
B. Salmonellosis
C. Cholera
D. Typhoid
E. Amebiasis
33. An autopsy of a 65-year-old man , who died in week from the beginning of a profuse diarrhea, revealed severe
exicosis, dry all tissues and thick, concentrated blood. A bacteriological study of contents of a small bowel, which
reminded the rice broth, determined vibrioes. What disease resulted in the patient's death?
A * Cholera
B. Dysentery
C. Typhoid
D. Salmonellosis
E. Alimentary toxic infection
34. A male patient, with 5 days history of diarrhea, had a colonoscopy. The investigation revealed a colon mucosa
inflammation with grey-green membranes, intimately connected with tissue underneath. What is the most likely
diagnosis?
A * Dysentery
B. Typhoid
C. Nonspecific ulcerative colitis
D. Salmonellosis
E. Crohn's disease
35. An autopsy of the patient who died of a plague revealed a hemorrhagic syndrome, accompanied with the
hemorrhagic necrosis of a hip skin, a lymphangitis and an inguinal hemorrhagic lymphadenitis. Name the
plague form. A * Dermo-bubonic

112
B. Bubonic
C. Primary septic
D. Primary pulmonic
E. Hemorrhagic
36. A 38-year-old man died of intoxication. A post-mortem revealed in a small bowel an edema of lymphoid
follicles groups. The follicles protruded above the mucosal surface like a soft plaques with irregular surface pattern,
which reminded brain's gyri and sulci. What diagnosis is most probable?
A * Typhoid
B. Dysentery
C. Salmonellosis
D. Acute enteritis
E. Cholera
37. A young man died of intoxication on 4th day after eating the crude eggs. An autopsy revealed the stomach and
small bowel mucosa inflamed, covered by a mucous exudate. In addition, abscesses were found in lungs, brain and
liver. What diagnosis is the most likely? A *Salmonellosis (septic form)
B. Dysentery
C. Salmonellosis (typhoid form)
D. Salmonellosis (intestinal form)
E. Typhoid
38. An autopsy of a 52- year-old man revealed in sigmoid and rectum plural brown-green membranes, hemorrhages,
some blood in the lumen of intestine. Histological study showed a fibrinous colitis. Bacteriological investigation
found S. Sonne. What is the most likely diagnosis? A * Dysentery
B. Cholera
C. Salmonellosis
D. Yersiniosis
E. Crohn's disease
39. A histological research of an autopsy material revealed in a brain tissue a wide zone of proliferating microglia
cells round small vessels. In addition, the marked proliferation of an endothelium, an adventitia and pericytes were
determined in the skin vessels. Perivascular moderate lymphocytes infiltration with single neutrophiles added was
also found. The vessels walls underwent various signs of destruction. Name the causative organism of described
infection disease?
A. * Rickettsia
B. Spirochete
C. Meningococcus
D. Streptococcus
E. Human immunodeficiency virus
40. A 7-year-old child, on the 3ld week of disease, presented with hectic fever, skin and sclera icterus. He died soon
of the hepatorenal failure. An autopsy reveled in a rectum many irregular form defects with uneven edges. These
defects were covered by grey-white membranes, intimately soldered to a tissue underneath. In addition, there was a
purulent thrombophlebitis of the intestinal vessels. Plural pylephlebitic abscesses and fatty dystrophy of hepatocytes
were diagnosed in a liver. An acute necrotic nephrosis was found in kidneys. What disease caused a lethal outcome?
A. *Dysentery
B. Salmonellosis
C. Cholera
D. Typhoid
E. Amebiasis
41. A worker of the agricultural enterprise presented with an acute disease and died soon of intoxication. An autopsy
revealed the enlarged and flaccid spleen with a dark-cherry coloring of a cut surface. There were a superfluous
pulp's scrape from the lien cut surface. Arachnoids of the brain's fornix and base were edematous, impregnated by
blood ('red cap' or 'cardinal's hat'). Microscopical investigation found the serous- hemorrhagic inflammation of
brain's membranes and cerebral tissues. What is the most likely diagnosis?
A. * Anthrax
B. Tularemia
C. Plague
D. Cholera
E. Brucellosis
42. An autopsy at the Forensic Pathology Department of an unknown man, with a history of alcohol intoxication,
revealed in his small bowel the enlarged Peyer's patches, which protruded above the mucosal surface. They were
soft, with irregular surface, which reminded brain's gyri and sulci. The cut surface had a gray-red coloring. For what
disease the described changes are characteristic?
A. * Typhoid
B. Dysentery
C. Lamb lias is

113
D. Amehiasis
E. Enteritis due to Campylobacter
43. A post-mortem of a patient, who died of diffuse fibropurulent peritonitis, revealed in the wall of a small bowel
multiple oval ulcers with rounded edges, which were parallel to the longitudinal axis of the intestine. A base of the
ulcers presented either by muscular layer of ileum or its serosa. Two ulcers had perforation apertures 0,3cm in
diameter each. Define a typhoid stage.
A. * Stage of pure ulcers
B. Stage of "dirty ulcers" formation
C. Necrotic stage
D. Stage of a medullar swelling
E. Stage of healing
44. A 65 -year-old patient addressed to a doctor his complains to frequent urge and painful passage of stool,
with mucous and some blood in feces. Proctosigmoidoscopy revealed marked narrowing of a colon's
lumen, especially in its distal part. A mucosa was edematous, hyperemic, with areas of necrosis and hemorrhages. A
bacteriological study of feces determined the Shigella flexneri. Define the disease?
A. * Dysentery
B. Typhoid
C. Anthrax
D. Crohn's disease
E. Paratyphoid

TUBERCULOSIS
1. A histological investigation of the cervical lymph node revealed the aggregation of epithelioid cells, lymphocytes
and giant Pirogov - Langhans cells. In the centre of a lesion a caseous necrosis was present. Specify the most likely
pathology.
A. * Tuberculosis
B. Rhinoscleroma.
C. Sarcoidosis (Boeck's disease)
D. Glanders
E. Syphilis
2. An autopsy of a 42-year-old man revealed in the second segment of the right lung the focus of consolidation, 5
cm in diameter, surrounded with a thin capsule. The centre was presented by a dense dry crumbling tissue with a
dim surface. Morphological changes in a lung are characteristic for:
A. Tuberculoma
B. Carcinoma of lung
C. Chondromas
D. The tumorous formof a silicosis
E. Postinflammation pneumosclerosis
3. An autopsy of a 63-year-old man revealed in the second segment of the right lung a tubercular panbronchitis,
with the focus of a caseous bronchopneumonia. The focus was surrounded by a wall of epithelioid cells with some
lymphocytes and few giant Pirogov -Langhans cells. What form of a pulmonary tuberculosis presented in this
case?
A. *Acute pulmonary tuberculosis
B. Primary tuberculous complex
C. Growth of primary affect
D. Acute miliary tuberculosis
E. Infiltration tuberculosis
4. A 26-year-old patient arrived in a hospital with complaints to a fever and weakness. One of the enlarged cervical
lymph nodes was taken for histological study. Microscopical investigation showed in lymphoid tissue the centers
of necrosis, surrounded with epithelioid cells, giant multinuclear Pirogov-Langhans cells and lymphocytes. What is
the most likely disease?
A. * Tuberculosis
B. Lymphogranulomatosis (Hodgkin's disease)
C. Lymphatic leukemia
D. Sarcoidosis (Boeck's disease)
E. Syphilis
5. An autopsy of a 44-year-old man revealed tuberculosis. A macroscopical investigation showed in the apex of
the right lung the foci of caseous pneumonia. There were caseous lymphadenitis of enlarged lymph nodes of
mediastinum and plural milium nodules in many organs. Name the described form of tuberculosis?
A. * Primary with mixed form of progressing
B. Primary with gematogenic spreading
C. Primary with lymphogenic spreading
D. Primary with growth of a primary affect

114
E. Gematogenic
6. A 7-year-old boy with a history of tonsillitis presented with markedly enlarged paratracheal, bifurcate and
cervical lymph nodes. A microscopical investigation of the removed cervical lymph node revealed the centers of a
necrosis surrounded with lymphocytes, epithelioid cells and Pirogov-Langhans cells. Specify the most likely
pathology.
A. *Tuberculosis
B. Sarcoidosis (Boeck's disease)
C. Rhinoscleroma.
D. Glanders
E. Syphilis
7. A 40- year-old man died of a pulmonary bleeding. An autopsy revealed in the upper lobe of the right lung a
cavity, 4cm in diameter, with dense walls and a rough internal surface, filled with blood clots. A microscopical
study of the wall showed the inner layer, presented with fell to pieces leucocytes; the middle layer formed by
epithelioid, lymphoid and giant multinuclear cells with nuclei located like a horseshoe. The changes found out in a
lung are characteristic for:
A. Tuberculous cavern (cavity)
B. Bronchiectatic cavern (cavity)
C. Abscess of lungs
D. Congenital cyst
E. Infarct of lung with septic disintegration
8. A post-mortem of a 43-year-old man revealed in the 3 segment of the right lung under the pleura an area of
consolidation, 1,5 cm in diameter, with the accurate borders, surrounded with a whitish fibrous tissue. A gross
investigation of a cut section showed the white-yellow crumbling foci. Presence of the described focus is
characteristic for:
A. Encapsulation of primary affect
B. Peripheral carcinoma
C. Chondroma
D. Fibroma
E. Infarct of lung with organization
9. A 48- year-old man died of a pulmonary -heart failure. An autopsy revealed pneumosclerosis, emphysema of
lungs, and hypertrophy of a right ventricle of heart. In both lungs, mainly under pleura, there were plural foci, 1 cm
in diameter. Histological investigation of the foci showed a necrosis zone in the centre and a wall of epithelioid
cells, lymphocytes with some macrophages and plasmocytes on periphery. Giant Pirogov -Langhans cells were also
defined. The small numbers of vascular capillaries were present on a periphery of the foci. What is the most likely
disease?
A. Gematogenic tuberculosis.
B. Actinomycosis of lungs.
C. Sarcoidosis (Boeck's disease)
D. Syphilis.
E. Silicosis.
10. A 56-year-old man with a long history of fibrous - cavernous pulmonary tuberculosis died of a chronic
pulmonary and heart failure. Within last months the expressed proteinuria was observed. A post-mortem revealed
enlarged, firm kidneys, with a waxy surface. What changes developed in kidneys at this form of tuberculosis?
A. Amyloidosis of kidney
B. Tuberculosis of kidney
C. Glomerulonephritis
D. Nephrolithiasis
E. Necrotic nephrosis
] 1. A child died of a diffuse peritonitis. An autopsy revealed the primary intestinal tubercular complex with primary
affect, a lymphangitis and regional caseous lymphadenitis. In addition, an ulcer of an empty intestine with
perforation was found within the primary affect focus. What is the most likely way of tuberculosis infection in
presented case?
A. Alimentary (nutritional)
B. Transplacental
C. Aero genie
D. Mixed
E. Contact
12. A 24-year-old man presented with enlarged cervical lymphatic node. A histological investigation revealed the
growth of epithelioid, lymphoid cells and macrophages with nuclei in the form of a horseshoe. In some centers the
aggregates of unstructured light pink color masses with nuclei fragments were determined. For what disease the
described changes are characteristic?
A. Tuberculosis
B. Lymphogranulomatosis (Hodgkin's disease)

115
C. Actinomycosis
D. Metastasis of tumor
E. Syphilis
13. A patient with a history of fibrous - cavernous tuberculosis died of a renal failure. A post-mortem revealed a
urine smell, left ventricle hypertrophy, fibrinous pericarditis, and fibrinous - hemorrhagic enterocolitis. Kidneys
were small, reduced in sizes, dense, with plural foci of shrinkage. Histological investigation of the slides with
Congo-Red staining showed pink masses within glomuleruli and vessels walls. In addition, destruction and an
atrophy of the majority of nephrons, accompanied with a nephrosclerosis were determined. What is the most likely
pathology of kidneys?
A. Amyloid- contracted kidneys
B. Contracted kidney
C. Arteriolosclerotic kidney
D. Atherosclerotic contracted kidneys
E. Pielonephritic contracted kidneys
14. An autopsy of a 7-year-old child revealed in the apex of the right lung, under the pleura, a focus of caseous
necrosis, 15 mm in diameter. There were also enlarged bifurcational lymph nodes with small necrotic foci. A
microscopical investigation of the pulmonal focus and the lymph node's lesion showed necrotic masses surrounded
by epithelioid cells, lymphocytes and solitary giant multinuclear Pirogov- Langhans' cells. What is the most likely
disease?
A. *Primary tuberculosis
B. Gematogenic tuberculosis with the lesions of lungs
C. Gematogenic generalized tuberculosis
D. Secondary inflammatory tuberculosis
E. Secondary fibre-inflammatory tuberculosis
15. A post-mortem of a 58-year-old patient revealed plural pathological cavities in both lungs. Microscopical study
showed the inner layer of one cavity presented with necrotic masses and diffuse neutrophil infiltration; the middle
one contained an infiltrate of epithelioid cells, lymphocytes and multinuclear giant cells. The external layer
consisted of a mature connective tissue. Diagnose the form of secondary tuberculosis.
A. *Fibrocavernous tuberculosis
B. Acute inflammatory
C. Fibre-inflammatory
D. Acute cavernous
E. Cirrhotic tuberculosis
16. An autopsy of a 74- year-old man revealed the enlargement and deformation of the right knee joint. A
histological investigation determined the massive foci of a caseous necrosis, surrounded by peripheral mantle of
epithelioid, lymphoid cells and some multinucleated giant cells.
What is the most likely disease?
A. *Tuberculous arthritis
B. Syphilitic arthritis
C. Rheumatic arthritis
D. Gonorrheal arthritis
E. Deforming osteoarthrosis
17. A 52- year-old man with a long history of tuberculosis died of a bleeding from lungs. An autopsy revealed in
lungs some oval and round cavities. The walls of cavities were presented with necrotic masses and a pulmonal
tissue. What is the most likely form of tuberculosis presented in that case?
A. * Acute cavernous
B. Fibrocavernous
C. Tuberculoma
D. Caseous pneumonia
E. Acute inflammatory
18. A post-mortem of a 58-year-old patient, with a history of tuberculosis, revealed in the upper lobe of the right
lung a cavity, 3x2cm in sizes, which connected with a bronchus. A cavity had dense walls and three layers. The
internal layer was pyogenic; the middle one was presented with tubercular granulation tissue and the external
coating of connective tissue. What of the listed diagnoses the most likely?
A. * Fibrocavernous tuberculosis.
B. Fibrous focal tuberculosis
C. Tuberculoma.
D. Acute focal tuberculosis.
E. Acute cavernous tuberculosis
19. A 42-year-old patient complained of a voice timbre change. The larynx biopsy was taken. A histological
investigation revealed the aggregates of large, oblong form cells with light nuclei, rich with a cytoplasm. The nuclei
were located on periphery in the form of a horseshoe. The described histological changes are characteristic for:
A. *Tuberculosis

116
B. Leprosy
C. Syphilis
D. Rhinoscleroma
E. Lymphogranulomatosis (Hodgkin's disease)
20. A histological investigation of the enlarged cervical lymph nodes revealed the nodules consisted mainly of flat,
slightly extended cells with pale nucleus. There were also giant cells with spherical-oval form, light pink cytoplasm
and with nuclei located on a cell periphery. The centre of some nodules was presented with the unstructured, light
pink colored masses. The described changes are characteristic
for:
A. Tuberculosis
B. Lymphogranulomatosis (Hodgkin's disease)
C. Sarcoidosis (Boeck's disease)
D. Brucellosis
E. Nonspecific hyperplasic lymphadenitis
21. A histological investigation of the enlarged supraclavicular lymph nodes, removed from a 40-year-old woman,
revealed an alteration of their structure with aggregates of slightly extended cells with a light nucleus. The
multinuclear, large, oval-round form cells were also determined. These cells were rich with a cytoplasm and their
nuclei were located on periphery in the form of a paling. The lymphoid cells created the next layer. The unstructured
pinkish masses were seen within the centers of lesions. What the described changes are characteristic for?
A. Tuberculous lymphadenitis
B. Metastasis of tumor
C. Lymphogranulomatosis (Hodgkin's disease)
D. Hyperplasic lymphadenitis
E. Sarcoidosis (Boeck's disease)
22. A lung lobe was removed at Surgery Department due to preliminary diagnosed purulent inflammation of a cyst.
A gross investigation of the operative material revealed within the 8 segment a dense focus with irregular roundish
form, 1,8 cm in diameter. On a cut, it had white-yellow coloring with a dim surface and fine sites of osteal density.
The described macroscopical picture characteristic for:
A. * Tuberculous affect with healing
B. Peripheral carcinoma of lung
C. Chondromas
D. Carnification
E. Chronic pneumonia
23. A 52-year-old man with a long history of a tubercular prostatitis died of a meningocephalitis. An autopsy
revealed in arachnoids of the basis and lateral surfaces of a brain, in a spleen, kidneys and liver a considerable
quantity of dense grey color nodules 0, 5-lmm in diameter. Histological investigation of nodules showed epithelioid,
lymphoid and some giant cells with nuclei located on a cell periphery in the form of a horseshoe. What is the most
likely diagnosis?
A. *Acute milliary tuberculosis
B. Acute tuberculous sepsis
C. Chronic milliary tuberculosis
D. Disseminated tuberculosis
E. Septicopyemia
24. A post-mortem of a 40-year-old patient, with a 10 years history of tuberculosis, revealed a cavity in the 1 and П
segments of his right lung. It had dense walls. The inner layer was rough, with beams formed by the obliterated
bronchi and the thrombosed vessels. The middle and lower lobes were firm, with the yellow foci on a cut surface.
What form of tuberculosis is the most likely in that case?
A. * Fibrocavernous
B. Acute cavernous
C. Fibrous inflammatory
D. Tuberculoma
E. Cirrhotic
25. A 40-year-old prisoner died in penitentiary of tuberculosis. An autopsy revealed a deformation and reduction of
the both lungs apexes, plural cavities with dense, 2-3 mm thick walls in the upper lobes of both lungs. In addition,
disseminated foci of a caseous necrosis, 5mm-2 cm in diameter, were found in the lower lobes of lungs. Diagnose
the tuberculosis form.
A. * Secondary fibrocavernous
B. Secondary fibre-inflammatory
C. Gematogenic inflammatory with lesion of lungs
D. Primary, growth of primary affect
E. Secondary cirrhotic
26. A 10-year-old girl arrived in traumatological unit with symptoms of pathological fracture of the
right femur. A histological investigation of an operational material revealed in a bone marrow of a femur

117
fragment, a plural foci of a caseous necrosis with peripheral mantle of epithelioid cells, lymphocytes and solitary
Pirogov-Langhans' giant cells. Zones of a caseous necrosis extended on adjoining sites of a bone tissue, resulting in
osseous destruction. What is the most likely disease?
A. *Tuberculous osteomyelitis
B. Tuberculous spondylitis
C. Tuberculous koxitis
D. General(common) macrofocal tuberculosis
E. Secondary acute focal tuberculosis
27. An autopsy of a 48-year-old man revealed in the first segment of the right lung the round formation, 5 cm in
diameter, surrounded with a thin connective tissue layer. The centre was presented by write, brittle masses.
Diagnose the form of secondary tuberculosis.
A. * Tuberculoma.
B. Caseous pneumonia
C. Acute cavernous tuberculosis
D. Acute inflammatory tuberculosis
E. Fibrocavernous tuberculosis
28. A post-mortem of the 17-year-old girl, who died of a pulmonary insufficiency, revealed confluent zones of a
caseous necrosis in the lower lobe of the right lung. The caseous necrosis was also determined in broncho-
pulmonal, bronchial and bifurcational lymph nodes. What of the listed diagnoses the most likely?
A. *Growth primary affect at primary tuberculosis
B. Gematogenic form of progress of primary tuberculosis
C. Gematogenic tuberculosis with lesion of lungs
D. Tuberculoma
E. Caseous pneumonia at the secondary tuberculosis
29. A 50-year-old man with a history of tuberculosis died of a chronic pulmonary and heart failure. A post-mortem
revealed a lobar lesion of the right lung. The upper lobe of the right lung was enlarged, dense, with fibrinous
membranes on pleura. A gross investigation of a cut surface showed the yellow brittle tissue. What is the most likely
form of secondary tuberculosis described in that case?
A. *Caseous pneumonia
B. Fibrous-focal tuberculosis
C. Infiltrative tuberculosis.
D. Tuberculoma
E. Acute focal tuberculosis
30. An autopsy of a man with a history of tuberculosis revealed in the second segment of the right lung the white-
grey focus, 3 cm in diameter, surrounded with a capsule. Microscopic study showed a focus of necrosis with a
capsule and lack of a perifocal inflammation. Name the tuberculosis form.
A. * Tuberculoma
B. Acute cavernous tuberculosis
C. Fibrocavernous tuberculosis.
D. Cirrhotic tuberculosis
E. Caseous pneumonia
31. A died woman had a chronic pulmonal abscess in her medical records. An autopsy revealed in 2nd segment of
the right lung a roundish cavity, 5cm in the size. The internal surface of a cavity was created by caseous masses; the
external one - by a dense pulmonal tissue. A pyogenic membrane was absent. What is the most likely pathology?
A. * Acute cavernous tuberculosis
B. Fibrocavernous tuberculosis.
C. Acute abscess.
D. Chronic abscess.
E. Primary pulmonary cavity (cavern).
32. A lungs roentgenograrn investigation revealed a dark patch. During a diagnostic express biopsy of a lymph node
of a bronchus was removed. A histological study showed a caseous tissue necrosis, round which lied epithelioid and
lymphoid layers with few multinuclear large cells (Langhans' cells). Specify the cause of the lymphadenitis.
A. * Tuberculosis
B. Pneumonia
C. Syphilis
D. Metastases of cancer
E. Adenoviral infection
33. A 68-year-old woman, with a 20 years history of fibrous - cavernous pulmonary tuberculosis, presented to
the Department of Nephrology with symptoms of uremia, Intravital test on amyloid in kidneys appeared
positive. What is the most likely form of an amyloidosis presented in this case?
A. * Secondary systemic
B. Primary systemic
C. Limited (local)

118
D. Familial congenital
E. Senile (gerontal)
34. An 8-year-old child, with no disease in his medical history, presented to the hospital with a dyspnea. An x-ray
inspection showed the air and a liquid in a pleural cavity. During operation in the right lung the subpleural cavity,
2,4 cm in diameter was found. It had an irregular form with the residual caseous masses inside. The lesion was
connected with the pleural cavity. The similar caseous masses were found at a cut of the enlarged radical lymph
nodes. A histological research of a cavity's wall determined lymphocytes, epithelioid cells and
multinuclear giant cells. How correctly to name a cavity?
A. * Primary pulmonary cavity (cavern) at tuberculosis
B. Abscess of lungs
C. Empyema of pleura
D. Cavity (cavern) at acute cavernous tuberculosis
E. Cavity at fibrocavernous tuberculosis.
35. The girl within last 3 months ate the crude cow milk. She arrived in the hospital with the "acute abdomen"
symptoms. An operation revealed in a caecum a circular ulcer with perforation. A histological study of the ulcer
edges' determined necrotic masses, lymphocytes, epithelioid cells, and few multinucleated giant cells
What is your diagnosis?
A. * Primary intestinal affect at tuberculosis
B. Nonspecific ulcerous colitis
C. Amebiasis
D. Carcinoma of blind gut (cecum)
E. Dysentery
36. A histological study of the enlarged cervical lymph nodes revealed nodules with giant, spherical-oval form
cells. These cells had a light pink cytoplasm and nuclei located on a periphery. The found out changes are
characteristic for:
A. *Tuberculosis
B. Lymphogranulomatosis (Hodgkin's disease)
C. Sarcoidosis (Boeck's disease)
D. Brucellosis
E. Nonspecific hyperplastic lymphadenitis
37. A 63-year-old man with a long history of a tubercular prostatitis died of a meningocephalitis. An autopsy
revealed in arachnoids of the basis and lateral surfaces of a brain, in a spleen, kidneys and liver a considerable
quantity of dense grey color nodules 0, 5-lmm in diameter. Histological investigation of nodules showed epithelioid,
lymphoid and some giant cells with nuclei located on a cell periphery in the form of a horseshoe. What is the most
likely diagnosis?
A. *Acute military tuberculosis
B. Chronic milliary tuberculosis
C. Acuter tubercular sepsis
D. Macrofocal disseminated tuberculosis
E. Septicopyemia
38. A man died of liver cirrhosis. An autopsy revealed in the 1-2 segment of the right lung three grey-yellow, dense
foci, 1-1,5см in diameter with a dim surface. A histological research showed the tubercular endobronchitis, foci of a
caseous bronchopneumonia, surrounded with a wall of epithelioid, lymphoid and giant Pirogov-Langhans cells. For
what form of a tuberculosis the described changes are characteristic?
A. * Acute focal tuberculosis
B. Infiltrative tuberculosis
C. Fibrous-focal tuberculosis
D. Caseous pneumonia
E. Primary pulmonary tuberculous complex
39. An autopsy of a 40-year-old man with a history of AIDS revealed both lung lobes enlarged, dense, air free, with
fibrinous membranes on pleura. A gross investigation of a cut surface showed the yellow dim pulmonal
tissue. A histological investigation determined in alveoli the seroftbrinous and fibrinous exudate with large foci of
a pulmonal tissue necrosis. For what disease the described changes in lungs are characteristic?
A. * Caseous pneumonia
B. Infiltrative tuberculosis
C. Croupous pneumonia
D. Lymphogranulomatosis of lungs (Hodgkin's disease)
E. Carcinoma of lung
40. A 47- year-old man died of an acute anemia, resulted from the pulmonary bleeding. An autopsy revealed in the
2 segment of the right lung the cavity, 5-6 cm in the size, which had connection with the bronchus lumen. The cavity
had irregular roundish form and filled with blood. The internal surface of a cavity was rough and covered by a
flabby unstructured yellowish tissue. A wall was thin, presented by the condensed, inflammatorily changed
pulmonary tissue A histological investigation showed that the cavity inside layer consisted of the fused caseous

119
masses with a considerable quantity of segmented leucocytes. What is the most likely pathology presented in that
case?
A. * Acute cavernous tuberculosis
B. Abscess of lung
C. Disintegrating carcinoma of lung
D. Infarction of lung with septic disintegration
E. Bronchiectatic cavity (cavern)
41. A 56-year-old man died of a progressing heart failure. A post-mortem revealed small, dense lungs; the upper
lobes were deformed, penetrated by a cicatrical tissue, with the encapsulated foci of a caseous necrosis, 0, 2-0,5cm
in diameter. In the upper and middle lobes there were cavities, 3-4 and 4-5cm in the size, with dense walls and fiber
internal surface, filled with the opaque yellow-green liquid. The surrounding pulmonary tissue was sclerotic. The
heart was enlarged at the expense of the hypertrophied right ventricle. Define the form of secondary pulmonary
tuberculosis
A. * Cirrhotic tuberculosis
B. Fibrous-focal tuberculosis.
C. Infiltrative tuberculosis
D. Acute cavernous tuberculosis
E. Fibrocavernous tuberculosis
42. A post-mortem of a 72-year-old vagabond, with marked kypho-scoliosis, revealed the partially destructed
thoracal vertebral bodies due to formation of cavities (sequesters), filled with white-yellow brittle masses. A
histological investigation determined within altered vertebras big foci of a caseous necrosis, surrounded with
solitary giant multinucleated Pirogov-Langhans' cells, epithelioid cells and lymphocytes. Similar changes were
diagnosed at mediastinum lymph nodes and disseminated in lungs and kidneys. What is the most likely diagnosis?
A. *Tuberculous osteomyelitis
B. Purulent osteomyelitis
C. Syphilitic osteomyelitis
D. Osteoporosis
E. Osteoarthrosis
43. An ultrasound investigation of the young 17-year-old man's right kidney revealed pyeloectasis, accompanied
with disturbance of outflow of urine by urethra. Microscopical investigation of the renal biopsy showed a
diffusive infiltration of the interstitium by lymphocytes and hystiocytes. In the cortical and medullar zones there
were the foci of a destruction with formation of the cavities, surrounded by peripheral mantle of epithelioid ,
lymphoid cells and some multinucleated Pirogov-Langhans' giant cells. What disease is the most probable?
A. * Tuberculosis of kidney
B. Suppurative (apostematous) nephritis
C. Nonspecific tubular-interstitial nephritis
D. Polycystic kidney
E. Chronic pyelonephritis
44. An autopsy of a child, who died of meningocephalitis revealed under pleura and in pulmonal tissue plural,
punctual, yellow-white lesions which reminded millet grains. Histological investigation showed granulomas with a
necrosis zone in the centre and a wall of epithelioid cells, lymphocytes, macrophages and some giant Pirogov -
Langhans cells. What is the most likely disease?
A. *Primary milliary tuberculosis
B. Secondary tuberculosis
C. Focal pneumonia
D. Croupous pneumonia
45. A medical record of a dead child reveled meningeal symptoms in his medical history. His X-ray showed a dark
patch in the III segment of the right lung and enlarged radical lymph nodes. At autopsy, in cerebral arachnoids, the
millet grains-like nodules were found. Microscopical study of a nodule determined a focus of a caseous necrosis
with wall of epithelioid, lymphoid cells with giant cells between them. These giant cells had big nuclei, situated on
periphery in the form of a half moon. What is the most likely kind of meningitis?
A. *Tuberculous
B. Syphilitic
C. Brucellous
D. Influenza!
E. Meningococcai
SYPHILIS
1. A 10-year-old boy presented with Hutchinson's triad: parenchymatous keratitis, sensorineural deathess and
deformed Hutchinson's teeth (the barrel form), along with saddle-nose deformity.
For what illness the presented changes are characteristic?
A. * Syphilis.
B. Toxoplasmosis
C. Leprosy.

120
D. Tuberculosis.
E. Opisthorchiasis.
2. An autopsy of a 36-year-old man revealed in his liver s roundish formation, 0,5 cm in diameter. Microscopical
investigation showed in the centre of lesion necrotic masses, surrounded by the granulation tissue with some
plasmatic and lymphoid cells. There were also signs of the vasculitis in the blood vessels. What is the most likely
diagnosis in that case?
A. *Solitary gumma of liver
B. Solitary adenoma of liver.
C. Solitary leproma of liver.
D. Chronic abscess of liver.
E. Carcinoma of liver.
3. An autopsy of a 54-year-old man reveled above valvular rupture of an aorta with resulted in a cardiac tamponade.
Histological investigation of ascending part of an aorta showed in its external and middle layers inflammatory
infiltrates. They consisted of lymphoid, plasmatic and epithelioid cells. There were also recognized the necrosis
centers and proliferation of a vessels' wall. Changes in an aorta are characteristic for:
A. * Syphilitic aortitis
B. Septic aortitis
C. Rheumatic aortitis
D. Atherosclerosis
E. Hypertension diseases
4. A 20-year old woman presented with enlarged, not painful, condensed inguinal lymph nodes. In her genital
mucosa there was a small ulcer with firm edges and the "varnished" grayish color floor. What is the most likely
diagnosis?
A. * Syphilis
B. Tuberculosis.
C. Leprosy.
D. Trophic ulcer.
E. Gonorrhea.
5. A biopsy of cervix uteri was taken from the 34-year-old woman. Histological investigation revealed the
inflammatory infiltration with involving of fine arteries and veins walls. The infiltrate composed of
plasmocytes, lymphocytes, and epithelioid cells. There were also zones of sclerosis and a tissue hyalinosis. What
is the most likely disease?
A. * Syphilis
B. Tuberculosis
C. Leukoplakia
D. Cervical erosion
E. Condyloma
6. The prepuce was removed from a young man. A microscopical investigation revealed the polymorphic infiltrate
consisting from randomly located plasmatic, lymphoid and epithelioid cells. There was marked vessels' vasculitis.
For what disease the described changes are characteristic?
A. *Syphilis
B. Tuberculosis
C. Actinomycosis
D. Periarteritis nodosa
E. Leprosy
7. A man's oral cavity investigation revealed an oval, cartilaginous density, slightly elevated ulcer at his buccal
mucosa. A meaty, reddened floor was covered with grayish sebaceous deposits. A microscopical investigation
showed lymphocyte infiltration, situated mainly round fine vessels. There was also an endothelium proliferation
within the vessel's walls..What disease has developed in that case?
A. * Syphilis
B. Traumatic ulcer
C. Erosive-ulcerated leukoplakia
D. Ulcer-Carcinoma
E. Ulcero-necrotic Vensan's stomatitis
8. A histological study of a biopsy from an oral cavity ulcer revealed the foci of caseous necrosis surrounded with
plasmocytes, epithelioid , lymphoid cells and solitary giant multinucleated Pirogov-Langhans' cells. There were also
signs of endo - and a perivasculitis in fine vessels. Specify the disease.
A. * Syphilis
B. Tuberculosis
C. Leprosy
D. Rhinoscleroma
E. Glanders
SEPSIS

121
1. A 48-year-old patient with a history of croupous pneumonia died of the pulmonary and heart failure. A post-
mortem revealed a hyperplasia of a spleen, a thickening and defects of the aortal valve leaflet with large, up to 2 cm
in diameter, thrombuses attached to tissues underneath. What is the most likely diagnosis?
A. * Polypous-ulcerative endocarditis
B. Acute warty endocarditis
C. Recurrent warty endocarditis
D. Diffuse endocarditis
E. Fibroplastic endocarditis
2. An autopsy of a 48-year-old man revealed the infarct of the left hemisphere of a brain. A macroscopic study also
showed a big septic spleen, immunocomplex glomerulonephritis, ulcers in aortal valve leaflets, covered with
polypous thrombi. The latter ones had colonies of staphilococcuses, determined by microscopic study. What disease
resulted in cerebral thromboembolism?
A. * Septic bacterial endocarditis
B. Septicemia
C. Acute rheumatic valvulitis
D. Septicopyemia
E. Rheumatic thromboendocarditis
3. A 46-year-old man died of an odontogenic sepsis. An autopsy revealed the marked thickening of semilunar aortic
valves. They were whitish, opaque and stiff, with thrombotic masses, 1x1.5 cm in the size on the external surface.
What is the most likely form of an endocarditis?
A. * Polypous-ulcerative endocarditis
B. Diffuse endocarditis
C. Acute warty endocarditis
D. Fibroplastic endocarditis
E. Recurrent warty endocarditis
4. A 32-year-old man with a long history of a drug abuse, presented with a fever, the intoxication and a
pyuria. An echocardioscopy revealed massive superimpositions (vegetations) on the heart valves.
Microbiological blood test showed a presence of a staphylococcus. The patient died of a thromboembolism of a
pulmonary artery. Specify the changes of heart, revealed at post-mortem?
A. * Polypous-ulcerative endocarditis
B. Libman-Sacks endocarditis
C. Diffuse valvulitis
D. Acute warty endocarditis
E. Recurrent warty endocarditis
5. A young woman with a history of the out-of-hospital abortion presented with advanced purulent endomyometritis
resulted in a lethal outcome. A post-mortem revealed numerous abscesses of lungs, pustules under kidneys' capsule
and spleen hyperplasia. What form of a sepsis has developed?
A. * Septicopyemia
B. Septicemia
C. Chroniosepsis
D. Lung sepsis
E. Urosepsis
6. A 30-year-old woman with a history of abortion presented with purulent endometritis resulted in a death from
sepsis. A post-mortem revealed a purulent lymphadenitis, abscesses in lungs, kidneys, a myocardium, a spleen
hyperplasia, and dystrophy of parenchymatous organs. Define the clinico-anatomic form of a sepsis.
A. * Septicopyemia
B. Septicemia
C. Septic endocarditis
D. Granulomatous sepsis
E. Chroniosepsis
7. A 28-year-old woman died in the postnatal period. A post-mortem revealed a purulent endometritis and
thrombophlebitis of a uterus veins, plural abscesses of lungs, kidneys and a spleen. In addition, there were an
apostematous myocarditis and purulent meningitis. What is the most likely kind of a sepsis?
A. * Septicopyemia
B. Septicemia
C. Chroniosepsis
D. The prolonged septic endocarditis
E.
8. A man with a history of a lower extremity wound, presented with persistent pyesis of a lesion, resulted in death
of intoxication. An autopsy revealed a cachexia, dehydration; a brown atrophy of a liver, a myocardium, a spleen
and cross-striated muscles. In addition, there was amyloidosis of kidneys. What of the listed diagnoses is most
probable?
A. * Chroniosepsis

122
B. Septicopyemia
C. Septicemia.
D. Chernogubov's disease.
E. Brucellosis
9. A post-mortem revealed on the external surface of the aortal valve the large, 1-2 cm, brownish-red, easily
crumbling superimpositions (vegetations). They covered ulcerative defects in the valves. What is the most likely
diagnosis?
A. * Polypous-ulcerative endocarditis
B. Recurrent warty endocarditis
C. Acute warty endocarditis
D. Fibroplastic endocarditis
E. Diffuse endocarditis
DISEASE OF THE NERVOUS SYSTEM
1. An autopsy of the elderly woman revealed a massive purulent bronchopneumonia. A brain was
slightly atrophic, gyri were thinned and sulci were deep. A microscopical study of a brain showed perivascular
deposits of amyloid, Hirano bodies and neurofibrillary "tangles".What is the most likely main disease?
A. *Alzheimer's disease
B. Pick's disease
C. Parkinson's disease
D. Disseminated sclerosis
E. Lateral amyotrophic sclerosis
2. An autopsy of the 78-year-old woman revealed a massive purulent bronchopneumonia. There were
numerous grey color plaques, up to 15 mm in diameter, found in the brain and spinal cord. A microscopical study of
brain and spinal cord tissue showed plural perivenular sites of a demyelization with limphoplazmocytes infiltration
and the plural foci of gliosis. Diagnose a basic disease.
A. * Disseminated sclerosis
B. Pick's disease
C. Parkinson's disease
D. Alzheimer's disease
E. Lateral amyotrophic sclerosis
3. A 42- year-old forester, a week later after long stay in spring wood, presented to the hospital with a fever, a
headache, consciousness disturbance and epileptoid attacks. He died in 4 days. An autopsy revealed a brain edema,
plural petechial hemorrhages. A microscopical study showed perivascular and pericellular edema, plural
perivascular, mainly lymphocytic, infiltrates. Diagnose the basic disease.
A. Tick-borne encephalitis
B. Meningococcal infection
C. Poliomyelitis
D. Purulent encephalitis
E. Cerebro-vascula disease
4. An autopsy of the 68-year-old man revealed the atrophy of brain's tissue. The cerebral atrophy mainly involved
the frontal, temporal, and occipital regions. A hydrocephaly was also found. A microscopical study showed atrophic
zones in the cerebral cortex, the senile (neuritic) plaques in hippocampus, neurofibrillary "tangles", damage of
neurons and Hirano bodies. What is the most likely main disease?
A. * Alzheimer's disease
B. Lateral amyotrophic sclerosis
C. Disseminated sclerosis
D. Encephalitis
E. Ganglioneuroma
5. A 75-year-old man, with a history of Alzheimer's disease, died of a bronchopneumonia. A
microscopical study of the cerebral cortex revealed the senile (neuritic) plaques, neurofibrillary "tangles" and
damaged neurons. What formations characterize cytoskeleton pathology of proximal dendrites at the Alzheimer's
disease?
A. *Hirano bodies
B. Councilman's bodies
C. Negri bodies
D. Mallory's bodies
E. Russell bodies
6. An elderly man presented to the hospital with progressive dementia (aphrenia), with marked emotional lability;
thus the focal neurologic symptoms were absent. After his death, a post-mortem revealed the cortex thinning in
frontal, occipital and temporal lobes of the brain. A histological study of a brain's tissue showed the considerable
quantity of senile plaques (especially in a cortex), reduction of neurones' sizes with cytoplasm vacuolation. In
addition, the little Hirano bodies were determined in dendrites. What is the most likely pathology in the central
nervous system?

123
A. * Alzheimer's disease
B. Lateral amyotrophic sclerosis
C. Disseminated sclerosis
D. Tick-borne encephalitis
E. Viral meningoencephalitis
DISEASE OF AN OSTEOMUSCULAR SYSTEM
1. A morphological research of a femur reveled the chronic purulent inflammation of compact substance and a bone
marrow with sequesters formation. What is the most likely disease?
A. * Osteomyelitis
B. Retikulosarcoma
C. Multiple (plasma cell) myeloma (Kahler's disease)
D. Osteoblastoclastoma
E. Periostitis
2. A 6-year-old girl presented to the hospital with suddenly raised temperature and catarrh of respiratory tracts. For
5th day of illness there was a paralysis of her lower extremities muscles, accompanied with respiratory disorders. In
the anterior horns of a spinal cord there was a proliferation of a glia round necrotic neurons. What is the most likely
diagnosis?
A. *Poliomyelitis
B. Measles
C. Diphtheria
D. Meningococcemia
E. Scarlet fever
3. A 48- year-old man presented with advanced staphylococcal purulent periodontitis, resulted in a
purulent inflammation of alveolar process' bone marrow and then a mandible's body. Microscopical investigation
showed thinned osteal beams, the necrosis centers and the osteal sequesters, surrounded with a connective tissue's
capsule. What is the most likely disease?
A. *Chronic osteomyelitis
B. Acute osteomyelitis
C. Parodontome
D. Chronic fibrous periostitis
E. Purulent periostitis
4. A 52-year-old man with a 10 years history of chronic osteomyelitis and 3 years a nephrotic syndrome, died of
uremia. An autopsy revealed dense, white kidneys, with some cicatrices in their cortical layer. They had sebaceous
glitter on a cut surface. Specify a pathology of kidneys which has developed.
A. * Secondary amyloidosis
B. Primary amyloidosis
C. Idiopathic amyloidosis
D. Chronic glomerulonephritis
E. Chronic pyelonephritis
5. A 53- year-old man died of intoxication. An autopsy revealed the enlargement of his hip with some fistulas on a
skin. There was yellow - green thick liquid discharge from the fistulas. The gross investigation showed the
thickened femur with sequesters formation. A microscopical study determined sequestra! cavities, surrounded by
granulation and connective tissue with neutrophil infiltration. The bone-marrow channels were obliterated; the
compact layer was thickened. What is the most likely diagnosis?
A. *Chronic purulent osteomyelitis
B. Acute gematogenic osteomyelitis
C. Tuberculous osteomyelitis
D. Parathyroid osteodystrophy
E. Osteopetrosis
6. A 20-year-old woman presented with a several years history of muscular weakness, resulted in the motion,
talking, chewing and swallowing problems. A physical examination revealed the ptosis of both eyelids.
The thorax roentgenography found a tumorous conglomerate in anterior mediastinum. Histological
investigation of the tumor biopsy determined a thymoma. There were also some dystrophic alterations in skeletal
muscles. Diagnose the disease.
A. * Myasthenia
B. Progressive muscles dystrophy
C. Muscles Erb's dystrophy
D. Dermatomyositis
E. Poliomyelitis
7. A 20-year-old woman with a history of pathological cross- striated muscular weakness and ptosis died of
asphyxia. An autopsy revealed the follicular hyperplasia of a thymus, atrophy and a necrosis of skeletal muscles
with focal lymphocytic infiltrates among the cells. Specify the form of a muscles lesion.
A. * Myasthenia

124
B. Atrophy myositis
C. Progressive myopathy
D. Mussels Leyden's dystrophy
E. Mussels Erb's dystrophy
8. An autopsy of a patient, with a lifetime history of numerous pathological fractures, revealed changes of his long
tubular bones. The femurs and tibia bones were curved or spiral with uneven surface and the obliterated bone-
marrow channels on a cut. There was also a remodeling of a compact cortical layer into the spongy type.
Microscopical investigation showed mosaicity. of osteal structures. There were numerous cavities of the resorptions
combined with osteoblastic lying down of new bone. These were accompanied with chaotic fine-fibered or lamellar
structures of osteal fragments. The arteries were twisted and dilated. What is the most likely diagnosis?
A. * Deforming osteodystrophy
B. Osteopetrosis
C. Parathyroid osteodystrophy
D. Fibrous dysplasia
E. Chronic osteomyelitis
9. A 20-year-old woman presented with weakness of the eye, chewing, speech and swallowing groups of muscles.
Thus, the normal muscles contraction were completely ceased after the vigorous activity and restored again after
the rest. Later, the muscles of extremities and intercostals were also involved in pathological process.
Inadequate ventilation of the lungs resulted in secondary focal pneumonia, followed by patient's death. An autopsy
showed the atrophy of cross-striated muscles, their dystrophy with focal aggregates of lymphocytes in
interstitium. In the enlarged thymic gland the follicular hyperplasia was found out. Specify the most probable
diagnosis:
A. * Myasthenia
B. Verdniga-Goffman's spinal amyotrophy
C. Dyushen's mussels dystrophy
D. Lateral amyotrophic sclerosis.
E. Erb's mussels dystrophy
MYCOSISES.
1. A 48-year-old patient presented to the hospital with deep cerebral coma. It was known, that during lifetime he
experienced periodic attacks of a fever. An autopsy revealed grey-flaky colour of a bone marrow, lymph nodes;
enlarged a liver and a spleen. Histological investigation showed the haemomelanosis and a hemosiderosis in
mentioned organs. What of diagnoses is most probable?
A. * Malaria
B. Hemolytic anemia
C. Addison disease
D. Septicemia
E. Black pox
2. A 54-year-old man presented to the hospital with reddening and edema of the neck's skin, and then resulted in a
small abscess formation. On a cut, a tissue was dense with yellow-green coloring. In purulent masses there were
visible white grains. Histological study showed fungus' druses, plasmatic, xanthome cells, and macrophages.
Specify the most probable kind of a mycosis.
A. *Actinomycosis
B. Aspergillosis
C. Candidosis
D. Sporotrichosis
E. Coccidioidomycosis
3. An autopsy of 48-year-old woman, resident of Middle Asia, with long history of exhausting fever, revealed
enlarged liver and a spleen. They were grey-flaky color. The same color had hyperplastic bone marrow. A cerebral
cortex also had grey-smoky coloring. What is the most likely diagnosis?
A. * Malaria
B. AIDS
C. Epidemic typhus
D. Sepsis
E. Hepatitis
4. A 38-year-old man died of cerebral coma. It was known, that after foreign business trip in one of the African
countries, the icteruses periodically developed. An autopsy revealed the enlarged, dense spleen, with a black pulp. A
liver was also enlarged, hyperemic, and grey-black. A gross investigation of the brain determined brown-grey
coloring of a grey matter and plural fine hemorrhages in white matter. What infectious disease should be suspected?
A. * Malaria
B. Meningococcemia
C. Prion infection
D. Generalized herpes infection
E. Generalized cryptococcosis

125
5. A 69-year-old man presented to the hospital with edema and a painmlness of a skin and soft tissues of his neck.
The phlegmon of a neck and a mediastinitis were diagnosed. The patient died. An autopsy revealed enlarged, dense
left tonsil. On a cut, the tonsil was yellow-green, with set of fine cavities similar to porous structures. Microscopical
investigation determined fine abscesses with basophilic formations in their centers. These formations consisted of
short rhabdoid elements. What is the most likely diagnosis?
A. *Actinomycosis
B. Brucellosis
C. Amebiasis
D. Lambliasis
E. Leishmaniasis

126

Vous aimerez peut-être aussi